Practice Questions: Comprehensive Pharmacology

Lakukan tugas rumah & ujian kamu dengan baik sekarang menggunakan Quizwiz!

-vir

Antiviral

The nurse determines that the patient understood medication instructions about the use of a spacer device when taking inhaled medications after hearing the patient state which of the following as the primary benefit? A. "Now I will not need to breathe in as deeply when taking the inhaler medications." B. "This device will make it so much easier and faster to take my inhaled medications." C. "I will pay less for medication because it will last longer." D. "More of the medication will get down into my lungs to help my breathing."

D. "More of the medication will get down into my lungs to help my breathing." A spacer assists more medication to reach the lungs, with less being deposited in the mouth and the back of the throat.

vecuronium bromide (Norcuron)

skeletal muscle relaxatant ND: impaired communication R/T paralysis of skeletal muscles

613. The nurse is instructing a client regarding intranasal desmopressin. The nurse should tell the client that which occurrence is a side effect of the medication? 1. Headache 2. Vulval pain 3. Runny nose 4. Flushed skin

3 Desmopressin administered by the intranasal route can cause a runny nose. Options 1, 2, and 4 are side effects if the medication is administered by the intravenous route.

793. The nurse is providing instructions to a client who will be self-administering eyedrops. To minimize systemic absorption of the eyedrops, the nurse should instruct the client to take which action? 1. Eat before instilling the drops. 2. Swallow several times after instilling the drops. 3. Blink vigorously to encourage tearing after instilling the drops. 4. Occlude the nasolacrimal duct with a finger after instilling the drops.

4 Applying pressure on the nasolacrimal duct prevents systemic absorption of the medication. Options 1, 2, and 3 will not prevent systemic absorption.

678. A nurse teaches a client about the effects of diphenhydramine (Benadryl), which has been prescribed as a cough suppressant. The nurse determines that the client needs further instructions if the client states that he or she will: 1. Take the medication on an empty stomach. 2. Avoid using alcohol while taking this medication. 3. Use sugarless gum, candy, or oral rinses to decrease dry mouth. 4. Avoid activities requiring mental alertness while taking this medication.

1. Take the medication on an empty stomach.

The nurse is observing an unlicensed assistive personnel (UPA) who is performing morning care for a bedfast client with Huntington disease. Which care measure is most important for the nurse to supervise? A. Oral care B. Bathing C. Foot care D. Catheter care

A. A client with Huntington disease experiences problems with motor skills such as swallowing and is at high risk for aspiration. (B, C, D) do not pose life-threatening consequences.

The nurse is caring for a patient admitted to the hospital with pneumonia. Upon assessment, the nurse notes a temperature of 101.4° F, a productive cough with yellow sputum and a respiratory rate of 20. Which of the following nursing diagnosis is most appropriate based upon this assessment? A. Hyperthermia related to infectious illness B. Ineffective thermoregulation related to chilling C. Ineffective breathing pattern related to pneumonia D. Ineffective airway clearance related to thick secretions

A. Hyperthermia related to infectious illness Because the patient has spiked a temperature and has a diagnosis of pneumonia, the logical nursing diagnosis is hyperthermia related to infectious illness. There is no evidence of a chill, and her breathing pattern is within normal limits at 20 breaths per minute. There is no evidence of ineffective airway clearance from the information given because the patient is expectorating sputum.

A client with hypertension has been receiving ramipril (Altace) 5 mg PO daily for 2 weeks and is scheduled to receive a dose at 0900. At 0830 the client's blood pressure is 120/70. Which action should the nurse take? A. Administer the dose as prescribed. B. Hold the dose and contact the healthcare provider. C. Hold the dose and recheck the blood pressure in 1 hour. D. Check the healthcare provider's prescription to clarify the dose.

A. The BP is WNL and indicates that the medication is working. (B & C) would be indicated if the BP was low (systole below 100). (D) is not required because the dose is within manufacture's recommendations.

A patient with acute exacerbation of COPD needs to receive precise amounts of oxygen. Which of the following types of equipment should the nurse prepare to use? A. Venturi mask B. Partial non-rebreather mask C. Oxygen tent D. Nasal cannula

A. Venturi mask The Venturi mask delivers precise concentrations of oxygen and should be selected whenever this is a priority concern. The other methods are less precise in terms of amount of oxygen delivered.

The nurse is caring for a patient with an acute exacerbation of asthma. Following initial treatment, which of the following findings indicates to the nurse that the patient's respiratory status is improving? A. Wheezing becomes louder B. Vesicular breath sounds decrease C. Aerosol bronchodilators stimulate coughing D. The cough remains nonproductive

A. Wheezing becomes louder The primary problem during an exacerbation of asthma is narrowing of the airway and subsequent diminished air exchange. As the airways begin to dilate, wheezing gets louder because of better air exchange.

To find the infection site associated with acute lymphangitis, the nurse should look _____ to the inflammation. A. distal B. anterior C. proximal D. contralateral

A. distal The nurse should assess distal to swelling to locate the initial site of infection. Examining proximal, contralateral, or anterior to the inflammation does not describe swelling associated with infection.

A nurse is providing care to an adult female patient and observes that the Hb laboratory analysis result is 9 g/dl. Based on this finding, the nurse should expect to observe A. dyspnea. B. bradycardia. C. warm, dry skin. D. activity tolerance without complaint of fatigue.

A. dyspnea.Hb levels are used to determine the severity of anemia. Patients with moderate anemia (Hb 6 to 10 g/dL) may suffer from dyspnea, palpitations, diaphoresis with exertion, and chronic fatigue. Patients who are anemic usually have cool skin related to compensatory mechanism of mild vasoconstriction. Patients who are anemic experience tachycardia because of increased demands placed on the heart to meet overall metabolic requirements. Activity tolerance without complaint is not correct because patients with anemic conditions fatigue readily.

Using light pressure with the index and middle fingers, the nurse cannot palpate any of the patient's superficial lymph nodes. The nurse A. records this finding as normal. B. should reassess the lymph nodes using deeper pressure. C. asks the patient about any history of any radiation therapy. D. notifies the health care provider that x-rays of the nodes will be necessary.

A. records this finding as normal. Superficial lymph nodes are evaluated by light palpation, but they are not normally palpable. It may be normal to find small (<1.0 cm), mobile, firm, nontender nodes. Deep lymph nodes are detected radiographically.

The nurse notices clear nasal drainage in a patient newly admitted with facial trauma, including a nasal fracture. The nurse should: A. test the drainage for the presence of glucose. B. suction the nose to maintain airway clearance. C. document the findings and continue monitoring. D. apply a drip pad and reassure the patient this is normal.

A. test the drainage for the presence of glucose. Clear nasal drainage suggests leakage of cerebrospinal fluid (CSF). The drainage should be tested for the presence of glucose, which would indicate the presence of CSF.

-cocci

Berry shaped bacteria.

HPV vaccine

Human Papilloma Virus (HPV2, HPV4) - -Three doses should be given over a 6 month -interval for females at 11 to 12 years of age (minimum age is 9 years). -The second dose should be administered 2 months after the first dose, and the third dose should be administered 6 months after the first dose. -HPV4 may be given to males starting at age 9 years of age.

mechanical receptors

receptors located in lungs, upper airways, chest wall, and diaphragm that are stimulated by irritants, muscle stretching, and alveolar wall distortion.

Cushing Syndrome

results from hypersecreation glucocorticoids in the adrenal cortex often develope diabetes mellitus - monitor serum glucose levels generialized edema low calorie, low carbohydrate, low sodium diet

sildenafil (viagra)

s/s: headache, heartburn, diarrhea, flushing, nosebleeds, parathesias, changes in color vision Contradicted in clients taking nitrates, anticoags, anti HTN

pancreatitis

serium amylase & lipase 2 to 5 times higher than normal hypercalcemia 40 ~ 75% = positive trousseau sign = carpal spasm severe boring pain

seconel

sleep aide 15g=1g

rhinoplasty

the surgical reconstruction of the nose.

Singulair

used before exercise to prevent breathing problems during exercise (bronchospasm).

Disc

A piece of cartilage between backbones.

Sagittal

A plane that divides the body into right and left portions.

A nurse establishes the presence of a tension pneumothorax when assessment findings reveal a(n) A. absence of lung sounds on the affected side. B. inability to auscultate tracheal breath sounds. C. deviation of the trachea toward the side opposite the pneumothorax. D. shift of the point of maximal impulse (PMI) to the left, with bounding pulses.

C. deviation of the trachea toward the side opposite the pneumothorax. Tension pneumothorax is caused by rapid accumulation of air in the pleural space, causing severely high intrapleural pressure. This results in collapse of the lung, and the mediastinum shifts toward the unaffected side, which is subsequently compressed.

-dipine

Ca+ channel blocker Slows movement of calcium into smooth muscle= arterial dilation & decreased BP Tx: angina, HTN (verapamil & diltiazem may be used for AFIB, A flutter, SVT S/S: Constipation, reflex tachycardia, peripheral edema, toxicity Common meds- nifedipine (procardia), verapamil, diltiazem

Necr/o

Death

-gliptin -glitazone

Diabetes Mellitus

Leukocyte

Eosinophil is a (an)

Hypertrophy

Excessive devolopment.

DNA

Genes are composed of?

Tonsillitis

Inflammation of lymph tissue in the throat.

-graph

Instrument to record.

Supine

Lying on the back.

Sarcoma

Malignant tumor of flesh tissue.

Adipose

Pertaining to fat.

-ptosis

Prolapse

Metabolism

Sum of the chemical processes in a cell.

Coccyx

Tailbone.

Catabolism

The process by which food is burned to realease energy.

Pharynx

Throat.

cirrhosis

Vitamin K1 (AquaMephyton) high calorie, low sodium diet sodium restriction w/ edema fluids restricted to decrease ascites late stage = ascites

pneumothorax

a collection of air or gas in the pleural space causing the lung to collapse.

pancreatic insufficiency

a condition characterized by inadequate production and secretion of pancreatic hormones or enzymes.

7. A client is beginning to take carisoprodol (Soma). Which interventions should the nurse include in the care of this client? (Select all that apply.) a. Ask the client if there is any history of narrow-angle glaucoma. b. Inform the client that muscular pain is usually relieved within 1 week. c. Tell the client to report dizziness and double vision to the health care provider. d. Advise the client to avoid alcohol and other CNS depressants. e. Instruct the client that this drug should not be stopped abruptly.

b. Inform the client that muscular pain is usually relieved within 1 week. c. Tell the client to report dizziness and double vision to the health care provider. d. Advise the client to avoid alcohol and other CNS depressants. e. Instruct the client that this drug should not be stopped abruptly.

5. Nursing interventions for the client taking carbidopa-levodopa for parkinsonism include which? a. Encouraging client to adhere to a high-protein diet b. Informing client that perspiration may be dark and stain clothing c. Advising client that glucose levels should be checked through urine testing d. Warning client that it may take 4 to 5 days before symptoms are controlled

b. Informing client that perspiration may be dark and stain clothing

diaphragmatic breathing

breathing with the use of the diaphragm to achieve maximum inhalation and slow respiratory rate.

Hypoglossal (12th crainal nerve)

difficulty chewing, talking and swallowing

metropolol, labetalol, propanolol

inhibit stimulation of receptor sites= decreased cardiac excitability, CO, myocaridal O2 demand, lower BP by decreasing release of renin in the kidney TX: HTN, angina, tachydysryhmias, HF, MI S/S: Bradycardia, Bradypena, Bronchospasms, decreased BP NI: Monitor DM for hypoglycemia

flail chest

instability of the chest wall resulting from multiple rib fractures.

CPR

just above the xiphoid process on the lower third of the sternum

hypocapnia

low arterial carbon dioxide pressure (PaCO2 < 35 mm Hg); also called hypocarbia.

IV's infusions potential problems

morphine, continous epidural = respiratory depression magnesium continous infusion = hypotension vancomycin intermittent infusion = nephrotoxicity & phlebitits

hypercalcemia

positive trousseau sign = carpal spasm

6.) The burn client is receiving treatments of topical mafenide acetate (Sulfamylon) to the site of injury. The nurse monitors the client, knowing that which of the following indicates that a systemic effect has occurred? 1.Hyperventilation 2.Elevated blood pressure 3.Local pain at the burn site 4.Local rash at the burn site

1.Hyperventilation Rationale: Mafenide acetate is a carbonic anhydrase inhibitor and can suppress renal excretion of acid, thereby causing acidosis. Clients receiving this treatment should be monitored for signs of an acid-base imbalance (hyperventilation). If this occurs, the medication should be discontinued for 1 to 2 days. Options 3 and 4 describe local rather than systemic effects. An elevated blood pressure may be expected from the pain that occurs with a burn injury.

797. A client was just admitted to the hospital to rule out a gastrointestinal (GI) bleed. The client has brought several bottles of medications prescribed by different specialists. During the admission assessment, the client states, "Lately, I have been hearing some roaring sounds in my ears, especially when I am alone." Which medication would the nurse identify as the cause of the client's complaint? 1. Doxycycline 2. Acetylsalicylic acid 3. Atropine sulfate 4. Diltiazem hydrochloride

2 Aspirin is contraindicated for gastrointestinal bleeding and is potentially ototoxic. The client should be advised to notify the prescribing health care provider so the medication can be discontinued and/or a substitute that is less toxic to the ear can be taken instead. Options 1, 3, and 4 do not have effects that are potentially associated with hearing difficulties.

758. The nurse is providing discharge instructions to a client receiving sulfamethoxazole. Which instruction should be included in the list? 1. Restrict fluid intake. 2. Maintain a high fluid intake. 3. If the urine turns dark brown, call the health care provider (HCP) immediately. 4. Decrease the dosage when symptoms are improving to prevent an allergic response.

2 Each dose of sulfamethoxazole should be administered with a full glass of water, and the client should maintain a high fluid intake. The medication is more soluble in alkaline urine. The client should not be instructed to taper or discontinue the dose. Some forms of sulfamethoxazole cause urine to turn dark brown or red. This does not indicate the need to notify the HCP.

729. Intravenous heparin therapy is prescribed for a client. While implementing this prescription, a nurse ensures that which of the following medications is available on the nursing unit? 1. Protamine sulfate 2. Potassium chloride 3. Aminocaproic acid (Amicar) 4. Vitamin K (AquaMEPHYTON)

1. Protamine sulfate

821. The client arrives at the emergency department complaining of back spasms. The client states, "I have been taking two to three aspirin every 4 hours for the last week, and it hasn't helped my back." Aspirin intoxication is suspected, and the nurse assesses the client for which of the following? 1. Tinnitus 2. Diarrhea 3. Constipation 4. Photosensitivity

1. Tinnitus

724. The nurse is monitoring a client who is taking propranolol. Which assessment data indicates a potential serious complication associated with this medication? 1. The development of complaints of insomnia 2. The development of audible expiratory wheezes 3. A baseline blood pressure of 150/80 mm Hg followed by a blood pressure of 138/72 mm Hg after two doses of the medication 4. A baseline resting heart rate of 88 beats/minute followed by a resting heart rate of 72 beats/minute after two doses of the medication

2 Audible expiratory wheezes may indicate a serious adverse reaction, bronchospasm. β-Blockers may induce this reaction, particularly in clients with chronic obstructive pulmonary disease or asthma. Normal decreases in blood pressure and heart rate are expected. Insomnia is a frequent mild side effect and should be monitored.

874. The nurse caring for a client who is taking an aminoglycoside should monitor the client for which adverse effects of the medication? Select all that apply. 1. Seizures 2. Ototoxicity 3. Renal toxicity 4. Dysrhythmias 5. Hepatotoxicity

2, 3, 4 Aminoglycosides are administered to inhibit the growth of bacteria. Adverse effects of this medication include confusion, ototoxicity, renal toxicity, gastrointestinal irritation, palpitations (dysrhythmias), blood pressure changes, and hypersensitivity reactions. Therefore, the remaining options are incorrect.

875. Ketoconazole is prescribed for a client with a diagnosis of candidiasis. Which interventions should the nurse include when administering this medication? Select all that apply. 1. Restrict fluid intake. 2. Monitor liver function studies. 3. Instruct the client to avoid alcohol. 4. Administer the medication with an antacid. 5. Instruct the client to avoid exposure to the sun. 6. Administer the medication on an empty stomach.

2, 3, 5 Ketoconazole is an antifungal medication. There is no reason for the client to restrict fluid intake; in fact, this could be harmful to the client. The medication is hepatotoxic, and the nurse monitors liver function. It is administered with food (not on an empty stomach) and antacids are avoided for 2 hours after taking the medication to ensure absorption. The client is also instructed to avoid alcohol. In addition, the client is instructed to avoid exposure to the sun because the medication increases photosensitivity.

824. The nurse has given medication instructions to the client receiving phenytoin (Dilantin). The nurse determines that the client has an adequate understanding if the client states that: 1. "Alcohol is not contraindicated while taking this medication." 2. "Good oral hygiene is needed, including brushing and flossing." 3. "The medication dose may be self-adjusted, depending on side effects." 4. "The morning dose of the medication should be taken before a serum drug level is drawn."

2. "Good oral hygiene is needed, including brushing and flossing."

194.) A nurse provides instructions to a client regarding the use of tretinoin (Retin-A). Which statement by the client indicates the need for further instructions? 1. "Optimal results will be seen after 6 weeks." 2. "I should apply a very thin layer to my skin." 3. "I should wash my hands thoroughly after applying the medication." 4. "I should cleanse my skin thoroughly before applying the medication."

2. "I should apply a very thin layer to my skin." Rationale: Tretinoin is applied liberally to the skin. The hands are washed thoroughly immediately after applying. Therapeutic results should be seen after 2 to 3 weeks but may not be optimal until after 6 weeks. The skin needs to be cleansed thoroughly before applying the medication.

64.) Nalidixic acid (NegGram) is prescribed for a client with a urinary tract infection. On review of the client's record, the nurse notes that the client is taking warfarin sodium (Coumadin) daily. Which prescription should the nurse anticipate for this client? 1. Discontinuation of warfarin sodium (Coumadin) 2. A decrease in the warfarin sodium (Coumadin) dosage 3. An increase in the warfarin sodium (Coumadin) dosage 4. A decrease in the usual dose of nalidixic acid (NegGram)

2. A decrease in the warfarin sodium (Coumadin) dosage Rationale: Nalidixic acid can intensify the effects of oral anticoagulants by displacing these agents from binding sites on plasma protein. When an oral anticoagulant is combined with nalidixic acid, a decrease in the anticoagulant dosage may be needed.

Chapter 61- cardio 717. A client with atrial fibrillation is receiving a continuous heparin infusion at 1000 units/hr. The nurse would determine that the client is receiving the therapeutic effect based on which of the following results? 1. Prothrombin time of 12.5 seconds 2. Activated partial thromboplastin time of 60 seconds 3. Activated partial thromboplastin time of 28 seconds 4. Activated partial thromboplastin time longer than 120 seconds

2. Activated partial thromboplastin time of 60 seconds

84.) Baclofen (Lioresal) is prescribed for the client with multiple sclerosis. The nurse assists in planning care, knowing that the primary therapeutic effect of this medication is which of the following? 1. Increased muscle tone 2. Decreased muscle spasms 3. Increased range of motion 4. Decreased local pain and tenderness

2. Decreased muscle spasms Rationale: Baclofen is a skeletal muscle relaxant and central nervous system depressant and acts at the spinal cord level to decrease the frequency and amplitude of muscle spasms in clients with spinal cord injuries or diseases and in clients with multiple sclerosis. Options 1, 3, and 4 are incorrect.

732. The nurse is monitoring a client who is taking digoxin (Lanoxin) for adverse effects. Which findings are characteristic of digoxin toxicity. Select all that apply. 1. Tremors 2. Diarrhea 3. Irritability 4. Blurred vision 5. Nausea and vomiting

2. Diarrhea 4. Blurred vision 5. Nausea and vomiting

133.) A nurse is monitoring a client receiving desmopressin acetate (DDAVP) for adverse effects to the medication. Which of the following indicates the presence of an adverse effect? 1. Insomnia 2. Drowsiness 3. Weight loss 4. Increased urination

2. Drowsiness Rationale: Water intoxication (overhydration) or hyponatremia is an adverse effect to desmopressin. Early signs include drowsiness, listlessness, and headache. Decreased urination, rapid weight gain, confusion, seizures, and coma also may occur in overhydration. **Recall that this medication is used to treat diabetes insipidus to eliminate weight loss and increased urination.**

162.) Carbamazepine (Tegretol) is prescribed for a client with a diagnosis of psychomotor seizures. The nurse reviews the client's health history, knowing that this medication is contraindicated if which of the following disorders is present? 1. Headaches 2. Liver disease 3. Hypothyroidism 4. Diabetes mellitus

2. Liver disease Rationale: Carbamazepine (Tegretol) is contraindicated in liver disease, and liver function tests are routinely prescribed for baseline purposes and are monitored during therapy. It is also contraindicated if the client has a history of blood dyscrasias. It is not contraindicated in the conditions noted in the incorrect options.

172.) A nurse provides dietary instructions to a client who will be taking warfarin sodium (Coumadin). The nurse tells the client to avoid which food item? 1. Grapes 2. Spinach 3. Watermelon 4. Cottage cheese

2. Spinach Rationale: Warfarin sodium is an anticoagulant. Anticoagulant medications act by antagonizing the action of vitamin K, which is needed for clotting. When a client is taking an anticoagulant, foods high in vitamin K often are omitted from the diet. Vitamin K-rich foods include green, leafy vegetables, fish, liver, coffee, and tea.

188.) The nurse should anticipate that the most likely medication to be prescribed prophylactically for a child with spina bifida (myelomeningocele) who has a neurogenic bladder would be: 1. Prednisone 2. Sulfisoxazole 3. Furosemide (Lasix) 4. Intravenous immune globulin (IVIG)

2. Sulfisoxazole Rationale: A neurogenic bladder prevents the bladder from completely emptying because of the decrease in muscle tone. The most likely medication to be prescribed to prevent urinary tract infection would be an antibiotic. A common prescribed medication is sulfisoxazole. Prednisone relieves allergic reactions and inflammation rather than preventing infection. Furosemide promotes diuresis and decreases edema caused by congestive heart failure. IVIG assists with antibody production in immunocompromised clients.

647. An older client recently has been taking cimetidine. The nurse monitors the client for which most frequent central nervous system side effect of this medication? 1. Tremors 2. Dizziness 3. Confusion 4. Hallucinations

3 Cimetidine (Tagamet) is a histamine H2-receptor antagonist. Older clients are especially susceptible to central nervous system side effects of cimetidine. The most frequent of these is confusion. Less common central nervous system side effects include headache, dizziness, drowsiness, and hallucinations.

356. The nurse is monitoring a client who is receiving oxytocin to induce labor. Which assessment finding would cause the nurse to immediately discontinue the oxytocin infusion? 1. Fatigue 2. Drowsiness 3. Uterine hyperstimulation 4. Early decelerations of the fetal heart rate

3 Oxytocin stimulates uterine contractions and is a common pharmacological method to induce labor. High-dose protocols have been associated with more uterine hyperstimulation and more cesarean births related to fetal stress. Some health care providers prescribe the administration of oxytocin in 10-minute pulsed infusions rather than as a continuous infusion. This pulsed method, which is more like endogenous secretion of oxytocin, is reported to be effective for labor induction and requires significantly less oxytocin use. Adverse effects associated with administration of the medication are hyperstimulation of uterine contractions and nonreassuring fetal heart rate patterns. Oxytocin infusion must be stopped when any signs of uterine hyperstimulation are present. Drowsiness and fatigue may be caused by the labor experience. Early decelerations of the fetal heart rate are a reassuring sign and do not indicate fetal distress.

680. The nurse is preparing to administer a dose of naloxone hydrochloride intravenously to a client with an intravenous opioid overdose. Which supportive medical equipment should the nurse plan to have at the client's bedside if needed? 1. Nasogastric tube 2. Paracentesis tray 3. Resuscitation equipment 4. Central line insertion tray

3 The nurse administering naloxone for suspected opioid overdose should have resuscitation equipment readily available to support naloxone therapy if it is needed. Other adjuncts that may be needed include oxygen, mechanical ventilator, and vasopressors.

954. A hospitalized client is started on phenelzine for the treatment of depression. The nurse should instruct the client that which foods are acceptable to consume while taking this medication? Select all that apply. 1. Figs 2. Yogurt 3. Crackers 4. Aged cheese 5. Tossed salad 6. Oatmeal raisin cookies

3, 5 Phenelzine is a monoamine oxidase inhibitor (MAOI). The client should avoid ingesting foods that are high in tyramine. Ingestion of these foods could trigger a potentially fatal hypertensive crisis. Foods to avoid include yogurt; aged cheeses; smoked or processed meats; red wines; and fruits such as avocados, raisins, or figs.

731. A client is admitted with pulmonary embolism and is to be treated with streptokinase (Streptase). A nurse would report which of the following assessments to the physician before initiating this therapy? 1. Adventitious breath sounds 2. Temperature of 99.4° F orally 3. Blood pressure of 198/110 mm Hg 4. Respiratory rate of 28 breaths/min

3. Blood pressure of 198/110 mm Hg

819. The client with trigeminal neuralgia tells the nurse that acetaminophen (Tylenol) is taken daily for the relief of generalized discomfort. Which laboratory value would indicate toxicity associated with the medication? 1. Sodium level of 140 mEq/L 2. Prothrombin time of 12 seconds 3. Direct bilirubin level of 2 mg/dL 4. Platelet count of 400,000/mm3

3. Direct bilirubin level of 2 mg/dL

154.) A nurse is reinforcing dietary instructions to a client who has been prescribed cyclosporine (Sandimmune). Which food item would the nurse instruct the client to avoid? 1. Red meats 2. Orange juice 3. Grapefruit juice 4. Green, leafy vegetables

3. Grapefruit juice Rationale: A compound present in grapefruit juice inhibits metabolism of cyclosporine. As a result, the consumption of grapefruit juice can raise cyclosporine levels by 50% to 100%, thereby greatly increasing the risk of toxicity. Grapefruit juice needs to be avoided. Red meats, orange juice, and green leafy vegetables are acceptable to consume.

724. A nurse is planning to administer hydrochlorothiazide (HydroDIURIL) to a client. The nurse understands that which of the following are concerns related to the administration of this medication? 1. Hypouricemia, hyperkalemia 2. Increased risk of osteoporosis 3. Hypokalemia, hyperglycemia, sulfa allergy 4. Hyperkalemia, hypoglycemia, penicillin allergy

3. Hypokalemia, hyperglycemia, sulfa allergy

60.) A nurse is planning to administer hydrochlorothiazide (HydroDIURIL) to a client. The nurse understands that which of the following are concerns related to the administration of this medication? 1. Hypouricemia, hyperkalemia 2. Increased risk of osteoporosis 3. Hypokalemia, hyperglycemia, sulfa allergy 4. Hyperkalemia, hypoglycemia, penicillin allergy

3. Hypokalemia, hyperglycemia, sulfa allergy Rationale: Thiazide diuretics such as hydrochlorothiazide are sulfa-based medications, and a client with a sulfa allergy is at risk for an allergic reaction. Also, clients are at risk for hypokalemia, hyperglycemia, hypercalcemia, hyperlipidemia, and hyperuricemia.

241.) A client with a history of simple partial seizures is taking clorazepate (Tranxene), and asks the nurse if there is a risk of addiction. The nurse's response is based on the understanding that clorazepate: 1. Is not habit forming, either physically or psychologically 2. Leads to physical tolerance, but only after 10 or more years of therapy 3. Leads to physical and psychological dependence with prolonged high-dose therapy 4. Can result in psychological dependence only, because of the nature of the medication

3. Leads to physical and psychological dependence with prolonged high-dose therapy Rationale: Clorazepate is classified as an anticonvulsant, antianxiety agent, and sedative-hypnotic (benzodiazepine). One of the concerns with clorazepate therapy is that the medication can lead to physical or psychological dependence with prolonged therapy at high doses. For this reason, the amount of medication that is readily available to the client at any one time is restricted. **Eliminate options 2 and 4 first because of the closed-ended word "only"**

681. Zafirlukast (Accolate) is prescribed for a client with bronchial asthma. Which laboratory test does the nurse expect to be prescribed before the administration of this medication? 1. Platelet count 2. Neutrophil count 3. Liver function tests 4. Complete blood count

3. Liver function tests

111.) A client arrives at the health care clinic and tells the nurse that he has been doubling his daily dosage of bupropion hydrochloride (Wellbutrin) to help him get better faster. The nurse understands that the client is now at risk for which of the following? 1. Insomnia 2. Weight gain 3. Seizure activity 4. Orthostatic hypotension

3. Seizure activity Rationale: Bupropion does not cause significant orthostatic blood pressure changes. Seizure activity is common in dosages greater than 450 mg daily. Bupropion frequently causes a drop in body weight. Insomnia is a side effect, but seizure activity causes a greater client risk.

211.) A client with epilepsy is taking the prescribed dose of phenytoin (Dilantin) to control seizures. A phenytoin blood level is drawn, and the results reveal a level of 35 mcg/ml. Which of the following symptoms would be expected as a result of this laboratory result? 1. Nystagmus 2. Tachycardia 3. Slurred speech 4. No symptoms, because this is a normal therapeutic level

3. Slurred speech Rationale: The therapeutic phenytoin level is 10 to 20 mcg/mL. At a level higher than 20 mcg/mL, involuntary movements of the eyeballs (nystagmus) appear. At a level higher than 30 mcg/mL, ataxia and slurred speech occur.

46.) A postoperative client has received a dose of naloxone hydrochloride for respiratory depression shortly after transfer to the nursing unit from the postanesthesia care unit. After administration of the medication, the nurse checks the client for: 1. Pupillary changes 2. Scattered lung wheezes 3. Sudden increase in pain 4. Sudden episodes of diarrhea

3. Sudden increase in pain Rationale: Naloxone hydrochloride is an antidote to opioids and may also be given to the postoperative client to treat respiratory depression. When given to the postoperative client for respiratory depression, it may also reverse the effects of analgesics. Therefore, the nurse must check the client for a sudden increase in the level of pain experienced. Options 1, 2, and 4 are not associated with this medication.

Know about Transdermal patch

-• Apply at the same time once each day, preferably in the morning. Keep patch on for 12 to 14 hr each day. • Remove the patch at night to reduce the risk of developing tolerance to nitroglycerin. Be medication-free a minimum of 10 to 12 hr each day (usually at night). • Do not cut patches to ensure appropriate dosage. • Place the patch on a hairless area of skin (chest, back, or abdomen) and rotate sites to prevent skin irritation. • Wash skin with soap and water and dry thoroughly before applying new patch.

850. Allopurinol is prescribed for a client and the nurse provides medication instructions to the client. Which instruction should the nurse provide? 1. Drink 3000 mL of fluid a day. 2. Take the medication on an empty stomach. 3. The effect of the medication will occur immediately. 4. Any swelling of the lips is a normal expected response.

1 Clients taking allopurinol are encouraged to drink 3000 mL of fluid a day. A full therapeutic effect may take 1 week or longer. Allopurinol is to be given with, or immediately after, meals or milk. A client who develops a rash, irritation of the eyes, or swelling of the lips or mouth should contact the health care provider because this may indicate hypersensitivity.

602. Glimepiride is prescribed for a client with DM. The nurse instructs the client to avoid consuming which food while taking this medication? 1. Alcohol 2. Organ meats 3. Whole-grain cereals 4. Carbonated beverages

1 When alcohol is combined with glimepiride, a disulfiram-like reaction may occur. This syndrome includes flushing, palpitations, and nausea. Alcohol can also potentiate the hypoglycemic effects of the medication. Clients need to be instructed to avoid alcohol consumption while taking this medication. The items in options 2, 3, and 4 do not need to be avoided.

653. A histamine (H2)-receptor antagonist will be prescribed for a client. The nurse understands that which medications are H2 receptor antagonists? Select all that apply. 1. Nizatidine 2. Ranitidine 3. Famotidine 4. Cimetidine 5. Esomeprazole 6. Lansoprazole

1, 2, 3, 4 H2-receptor antagonists suppress secretion of gastric acid, alleviate symptoms of heartburn, and assist in preventing complications of peptic ulcer disease. These medications also suppress gastric acid secretions and are used in active ulcer disease, erosive esophagitis, and pathological hypersecretory conditions. The other medications listed are proton pump inhibitors

358. The nurse is monitoring a client in preterm labor who is receiving intravenous magnesium sulfate. The nurse should monitor for which adverse effects of this medication? Select all that apply. 1. Flushing 2. Hypertension 3. Increased urine output 4. Depressed respirations 5. Extreme muscle weakness 6. Hyperactive deep tendon reflexes

1, 4, 5 Magnesium sulfate is a central nervous system depressant and relaxes smooth muscle, including the uterus. It is used to halt preterm labor contractions and is used for preeclamptic clients to prevent seizures. Adverse effects include flushing, depressed respirations, depressed deep tendon reflexes, hypotension, extreme muscle weakness, decreased urine output, pulmonary edema, and elevated serum magnesium levels.

116.) A nurse administers a dose of scopolamine (Transderm-Scop) to a postoperative client. The nurse tells the client to expect which of the following side effects of this medication? 1. Dry mouth 2. Diaphoresis 3. Excessive urination 4. Pupillary constriction

1. Dry mouth Rationale: Scopolamine is an anticholinergic medication for the prevention of nausea and vomiting that causes the frequent side effects of dry mouth, urinary retention, decreased sweating, and dilation of the pupils. The other options describe the opposite effects of cholinergic-blocking agents and therefore are incorrect.

825. A client with trigeminal neuralgia is being treated with carbamazepine, 400 mg orally daily. Which value indicates that the client is experiencing an adverse effect to the medication? 1. Uric acid level, 5 mg/dL 2. Sodium level, 140 mEq/L 3. Blood urea nitrogen level, 15 mg/dL 4. White blood cell count, 3000 cells/mm3

4 Adverse effects of carbamazepine appear as blood dyscrasias, including aplastic anemia, agranulocytosis, thrombocytopenia, and leukopenia; cardiovascular disturbances including thrombophlebitis and dysrhythmias; and dermatological effects. The low white blood cell count reflects agranulocytosis. The laboratory values in options 1, 2, and 3 are normal values.

559. A client with small cell lung cancer is being treated with etoposide. The nurse monitors the client during administration, knowing that which adverse effect is specifically associated with this medication? 1. Alopecia 2. Chest pain 3. Pulmonary fibrosis 4. Orthostatic hypotension

4 An adverse effect specific to etoposife is orthostatic hypotension. Etoposide should be administered slowly over 30-60 minutes to avoid hypotension. The client's blood pressure is monitored during the infusion. Hair loss occurs with nearly all the antineoplastic medications. Chest pain and pulmonary fibrosis are unrelated to this medication

819. Carbidopa-levodopa is prescribed for a client with Parkinson's disease. The nurse monitors the client for side/adverse effects to the medication. Which finding indicates that the client is experiencing an adverse effect? 1. Pruritus 2. Tachycardia 3. Hypertension 4. Impaired voluntary movements

4 Dyskinesia and impaired voluntary movement may occur with high levodopa dosages. Nausea, anorexia, dizziness, orthostatic hypotension, bradycardia, and akinesia are frequent side effects of the medication.

556. Chemotherapy dosage is frequently based on total body surface area; so it is important for the nurse to perform which assessment before administering chemotherapy? 1. Measure the client's abdominal girth 2. Calculate the client's body mass index 3. Ask the client about his or her weight and height 4. Measure the client's current weight and height

4 To ensure that the client receives optimal doses of chemotherapy, dosing is usually based on the total body surface area, which requires a current accurate height and weight for BSA calculations (before each administration). Asking the client about his or her height and weight may lead to inaccuracies in determining a true BSA and dosage. Calculating body mass index and abdominal girth will not provide the data needed.

118.) A nurse is caring for an older client with a diagnosis of myasthenia gravis and has reinforced self-care instructions. Which statement by the client indicates that further teaching is necessary? 1. "I rest each afternoon after my walk." 2. "I cough and deep breathe many times during the day." 3. "If I get abdominal cramps and diarrhea, I should call my doctor." 4. "I can change the time of my medication on the mornings that I feel strong."

4. "I can change the time of my medication on the mornings that I feel strong." Rationale: The client with myasthenia gravis should be taught that timing of anticholinesterase medication is critical. It is important to instruct the client to administer the medication on time to maintain a chemical balance at the neuromuscular junction. If not given on time, the client may become too weak to swallow. Options 1, 2, and 3 include the necessary information that the client needs to understand to maintain health with this neurological degenerative disease.

725. A home health care nurse is visiting a client with elevated triglyceride levels and a serum cholesterol level of 398 mg/dL. The client is taking cholestyramine (Questran). Which of the following statements, if made by the client, indicates the need for further education? 1. "Constipation and bloating might be a problem." 2. "I'll continue to watch my diet and reduce my fats." 3. "Walking a mile each day will help the whole process." 4. "I'll continue my nicotinic acid from the health food store."

4. "I'll continue my nicotinic acid from the health food store."

690. The nurse has just administered the first dose of omalizumab (Xolair) to a client. Which statement by the client would alert the nurse that the client may be experiencing a life threatening adverse reaction? 1. "I have a severe headache." 2. "My feet are quite swollen." 3. "I am nauseated and may vomit." 4. "My lips and tongue are swollen."

4. "My lips and tongue are swollen."

A male client who has never smoked but has had COPD for the past 5 years is now being assessed for cancer of the lung. The nurse knows that he is most likely to develop which type of lung cancer? A. Adenocarcinoma B. Oat-cell carcinoma C. Malignant melanoma D. Squamous-cell carcinoma

A. is the only lung cancer not related to cigarette smoking related to lung scarring and fibrosis from preexisting pulmonary diseases such as TB and COPD. (B& D) are related to smoking. (C) is a skin cancer

A 75-year-old obese patient who is snoring loudly and having periods of apnea several times each night is most likely experiencing A. narcolepsy. B. sleep apnea. C. sleep deprivation. D. paroxysmal nocturnal dyspnea.

B. sleep apnea. Sleep apnea is most common in obese patients. Typical symptoms include snoring and periods of apnea. Narcolepsy is when a patient falls asleep unexpectedly. Sleep deprivation could result from sleep apnea. Paroxysmal nocturnal dyspnea occurs when a patient has shortness of breath during the night.

If a patient with blood type O Rh- is given AB Rh- blood, the nurse would expect A. the patient's Rh factor to react with the RBCs of the donor blood. B. no adverse reaction because the patient has no antibodies against the donor blood. C. the anti-A and anti-B antibodies in the patient's blood to hemolyze the donor blood. D. the anti-A and anti-B antibodies in the donor blood to hemolyze the patient's blood.

C. the anti-A and anti-B antibodies in the patient's blood to hemolyze the donor blood. A patient with O Rh+ blood has no A or B antigens on the red cell but does have anti-A and anti-B antibodies in the blood and has an Rh antigen. AB Rh- blood has both A and B antigens on the red cell but no Rh antigen and no anti-A or anti-B antibodies. If the AB Rh- blood is given to the patient with O Rh+ blood, the antibodies in the patient's blood will react with the antigens in the donor blood, causing hemolysis of the donor cells. There will be no Rh reaction because the donor blood has no Rh antigen.

-asone, -solone - onide Pred- Cort-

Corticosteroid prevent inflammatory response S/S: Hyperglycemia, peptic ulcer, fluid retention (increased appetite), withdrawal symptoms, euphoria, insomnia, psychotic behavior NI: admin w/ meals, DO NOT take with NSAIDS, teach DO NOT stop abruptly Common meds- prednisone (deltasone), betamethasone (celestone), hydrocortisone sodium succinate (Solu-cortef), Methylprednisolone sodium succinate (solu-medrol), fluticasone propionate (advair, flovent)

-scope

Instrument to visually examine.

Viscera

Internal organs.

blunt trama to back of head

LOC assessment most important

(biguanide) Metformin

Oral hypoglycemic Used in conjunction with diet & exercise; type II NI: teach s/s of hypoglycemia, HbA1C metformin (glucophage): withhold 48 hrs before/after test w/ contrast

-ide

Oral hypoglycemic Used in conjunction with diet & exercise; type II NI: teach s/s of hypoglycemia, HbA1C metformin (glucophage): withhold 48 hrs before/after test w/ contrast

Labs for patients taking hydrothiazide

Periodic determination of serum electrolytes to detect possible electrolyte imbalance should be done at appropriate intervals.

Histologist

Person who studies tissues.

Karyotype

Picture of nuclear structures arranged in numerical order.

182.) A client with angina pectoris is experiencing chest pain that radiates down the left arm. The nurse administers a sublingual nitroglycerin tablet to the client. The client's pain is unrelieved, and the nurse determines that the client needs another nitroglycerin tablet. Which of the following vital signs is most important for the nurse to check before administering the medication? 1. Temperature 2. Respirations 3. Blood pressure 4. Radial pulse rate

Rationale: Nitroglycerin acts directly on the smooth muscle of the blood vessels, causing relaxation and dilation. As a result, hypotension can occur. The nurse would check the client's blood pressure before administering the second nitroglycerin tablet. Although the respirations and apical pulse may be checked, these vital signs are not affected as a result of this medication. The temperature also is not associated with the administration of this medication.

nifedipine (procardia), verapamil, diltiazem

Slows movement of calcium into smooth muscle= arterial dilation & decreased BP Tx: angina, HTN (verapamil & diltiazem may be used for AFIB, A flutter, SVT S/S: Constipation, reflex tachycardia, peripheral edema, toxicity

Larynx

Structure in the trachea.

-stomy

Surgical creation of a permanent opening to the outside of the body.

Amniocentesis

Surgical puncture to remove fluid from the sac around the embryo.

amitripytyline (elavil)

TCA S/S: anticholenergic effects, sedation, toxicity NI: DO NOT admin with MAOIs, avoid alcohol, contradicted in clients w/ seizures

Penicillin

TX: pneumonia, upper respiratory infections, septicemia, endocarditis, rheumatic fever, GYN infections NI: hypersensitivity w/ poss. anaphylaxis

Mediastinum

The space in the chest between the lungs.

obstructive sleep apnea

a condition characterized by partial or complete upper airway obstruction during sleep, causing apnea and hypopnea.

chylothorax

a condition marked by lymphatic fluid in the pleural space caused by a leak in the thoracic duct.

O2 toxicity

a condition of oxygen overdosage caused by prolonged exposure to a high levels of oxygen; may inactivate pulmonary surfactant and lead to development of acute respiratory distress syndrome.

7. A client is taking lithium. The nurse should be aware of the importance of which nursing intervention(s)? (Select all that apply.) a. Observe the client for motor tremors. b. Monitor the client for orthostatic hypotension. c. Draw lithium blood levels immediately after a dose. d. Advise the client to drink 750 mL/day of fluid in hot weather. e. Advise the client to avoid caffeinated foods and beverages, such as coffee, tea, colas, and chocolate. f. Teach the client to take lithium with meals to decrease gastric irritation.

a. Observe the client for motor tremors. b. Monitor the client for orthostatic hypotension. e. Advise the client to avoid caffeinated foods and beverages, such as coffee, tea, colas, and chocolate. f. Teach the client to take lithium with meals to decrease gastric irritation.

2. The nurse teaches the client receiving atropine to expect which side effect? a. Diarrhea b. Bradycardia c. Blurred vision d. Frequent urination

c. Blurred vision

4.Atenolol (Tenormin) is prescribed for a client. The nurse realizes that this drug is a beta-adrenergic blocker and that this drug classification is contraindicated for clients with which condition? a. Hypothyroidism b. Angina pectoris c. Cardiogenic shock d. Liver dysfunction

c. Cardiogenic shock

tidal volume

volume of air exchanged with each breath.

SIADH inappropriate antidiuretic hormone secreation

water retention & dilutional hyponatremia POC=quiet enviroment, deep tendon reflex assessment, neurologic checks, daily weights 1kg=1L

Patient identifiers

-Medical record number -home telephone number

What food should you increase when taking Lasix?

-increased amounts of potassium-rich foods (e.g., bananas, prunes, raisins, and orange juice)

604. The HCP prescribed exenatide for a client with type 1 DM who takes insulin. The nurse should plan to take which most appropriate intervention? 1. Withhold the medication and call the HCP, questioning the prescription for the client 2. Administer the medication within 60 minutes before the morning and evening meal 3. Monitor the client for GI side effects after administering the medication 4. Withdraw the insulin from the prefilled pen into an insulin syringe to prepare for administration

1 Exenatide (Byetta) is an incretin mimetic used for type 2 DM only. It is not recommended for clients taking insulin. Hence, the nurse should withhold the medication and question the HCP regarding this prescription. Although options 2 and 3 are correct statements about the medication, in this situation the medication should not be administered. The medication is packaged in prefilled pens ready for injection without the need for drawing it up into another syringe.

956. The nurse is performing a follow-up teaching session with a client discharged 1 month ago. The client is taking fluoxetine. What information would be important for the nurse to obtain during this client visit regarding the side/adverse effects of the medication? 1. Cardiovascular symptoms 2. Gastrointestinal dysfunctions 3. Problems with mouth dryness 4. Problems with excessive sweating

2 Buspirone (Buspar) is not recommended for the treatment of paranoid thought disorders, drug or alcohol withdrawal, or schizophrenia. Buspirone (Buspar) most often is indicated for the treatment of anxiety.

851. Colchicine is prescribed for a client with a diagnosis of gout. The nurse reviews the client's record, knowing that this medication would be used with caution in which disorder? 1. Myxedema 2. Kidney disease 3. Hypothyroidism 4. Diabetes mellitus

2 Colchicine is used with caution in older clients, debilitated clients, and clients with cardiac, kidney, or gastrointestinal disease. The disorders in options 1, 3, and 4 are not concerns with administration of this medication.

601. The home care nurse visits a client recently diagnosed with DM who is taking Humulin NPH insulin daily. The client asks the nurse how to store the unopened vials of insulin. The nurse should tell the client to take which action? 1. Freeze the insulin 2. Refrigerate the insulin 3. Store the insulin in a dark, dry place 4. Keep the insulin at room temperature

2 Insulin in unopened vials should be stored under refrigeration until needed. Vials should not be frozen. When stored unopened under refrigeration, insulin can be used up to the expiration date on the vial. Options 1, 3, and 4 are incorrect.

732. Intravenous heparin therapy is prescribed for a client. While implementing this prescription, the nurse ensures that which medication is available on the nursing unit? 1. Vitamin K 2. Protamine sulfate 3. Potassium chloride 4. Aminocaproic acid

2 The antidote to heparin is protamine sulfate; it should be readily available for use if excessive bleeding or hemorrhage should occur. Vitamin K is an antidote for warfarin sodium. Potassium chloride is administered for a potassium deficit. Aminocaproic acid is the antidote for thrombolytic therapy.

953. The nurse is teaching a client who is being started on imipramine about the medication. The nurse should inform the client to expect maximum desired effects at what time period following initiation of the medication? 1. In 2 months 2. In 2 to 3 weeks 3. During the first week 4. During the sixth week of administration

2 The maximum therapeutic effects of imipramine (Tofranil) may not occur for 2 to 3 weeks after antidepressant therapy has been initiated. Options 1, 3, and 4 are incorrect.

823. The nurse is caring for a client with severe back pain. Codeine sulfate has been prescribed for the client. Which of the following does the nurse specifically include in the plan of care while the client is taking this medication? 1. Monitor fluid balance. 2. Monitor bowel activity. 3. Monitor peripheral pulses. 4. Monitor for hypertension.

2. Monitor bowel activity.

47.) A client has been taking isoniazid (INH) for 2 months. The client complains to a nurse about numbness, paresthesias, and tingling in the extremities. The nurse interprets that the client is experiencing: 1. Hypercalcemia 2. Peripheral neuritis 3. Small blood vessel spasm 4. Impaired peripheral circulation

2. Peripheral neuritis Rationale: A common side effect of the TB drug INH is peripheral neuritis. This is manifested by numbness, tingling, and paresthesias in the extremities. This side effect can be minimized by pyridoxine (vitamin B6) intake. Options 1, 3, and 4 are incorrect.

956. A hospitalized client has begun taking bupropion (Wellbutrin) as an antidepressant agent. A nurse monitors this client for which side effect indicating that the client is taking an excessive amount of medication? 1. Constipation 2. Seizure activity 3. Increased weight 4. Dizziness when getting upright

2. Seizure activity

951. A nurse notes that a client with schizophrenia and receiving an antipsychotic medication is moving her mouth, protruding her tongue, and grimacing as she watches television. The nurse determines that the client is experiencing: 1. Parkinsonism 2. Tardive dyskinesia 3. Hypertensive crisis 4. Neuroleptic malignant syndrome

2. Tardive dyskinesia

765. The nurse is providing dietary instructions to a client who has been prescribed cyclosporine. Which food item should the nurse instruct the client to exclude from the diet? 1. Red meats 2. Orange juice 3. Grapefruit juice 4. Green leafy vegetables

3 A compound present in grapefruit juice inhibits metabolism of cyclosporine. As a result, consumption of grapefruit juice can raise cyclosporine levels by 50% to 100%, thereby greatly increasing the risk of toxicity

880. The nurse is assigned to care for a client with cytomegalovirus retinitis and acquired immunodeficiency syndrome who is receiving foscarnet, an antiviral medication. The nurse should monitor the results of which laboratory study while the client is taking this medication? 1. CD4 cell count 2. Lymphocyte count 3. Serum albumin level 4. Serum creatinine level

4 Foscarnet is toxic to the kidneys. The serum creatinine level is monitored before therapy, two or three times per week during induction therapy, and at least weekly during maintenance therapy. Foscarnet also may cause decreased levels of calcium, magnesium, phosphorus, and potassium. Thus, these levels also are measured with the same frequency.

A client diagnosed with angina pectoris complains of chest pain while ambulating in the hallway. Which action should the nurse implement first? A. Support the client to a sitting position. B. Ask the client to walk slowly back to the room. C. Administer a sublingual nitroglycerin tablet. D. Provide oxygen via nasal cannula.

A. Assist in safely repositioning and then administer (C & D). Then the client can be escorted back to the room via wheelchair or stretcher (B).

While obtaining the admission assessment data, which of the following characteristics would a nurse expect a patient with anemia to report? A. Palpitations B. Blurred vision C. Increased appetite D. Feeling of warm flushing sensation

A. Palpitations Patients experiencing moderate anemia (hemoglobin [Hb] 6 to 10 g/dL) may experience dyspnea (shortness of breath), palpitations, diaphoresis (profound perspiration) with exertion, and chronic fatigue. Blurred vision is associated in patients experiencing profound anemia states. Anorexia is common in patients with severe anemia, as well. Patients with anemia often appear pale and complain of feeling cold because of compensatory vasoconstriction of the subcutaneous capillaries.

41. Select all that apply. During initial assessment, a nurse should record which of the following manifestations of respiratory distress? A. Tachypnea B. Nasal flaring C. Thready pulse D. Panting or grunting E. Use of intercostal muscles F. An inspiratory-to-expiratory ratio of 1:2

AD Manifestations of respiratory distress include tachypnea, grunting and panting on respiration, central cyanosis, use of accessory muscles, and flaring nares.

527. The clinic nurse is performing an admission assessment on a client and notes that the client is taking azelaic acid. Because of the medication prescription, the nurse would suspect that the client is being treated for which condition? 1. Acne 2. Eczema 3. Hair loss 4. Herpes simplex

Answer: 1 Azelaic acid is a topical medication used to treat mild to moderate acne. The acid appears to work by suppressing the growth of Propionibacterium acnes and by ddecreasing the proliferation of keratinocytes. Options 2, 3, and 4 are incorrect.

-arin

Anticoagulant inhibit clotting factors (warfarin = factors VII, IX, X) TX: evolving stroke, pulmonary embolism, massive deep vein thrombosis, cardiac cath, MI, DIC S/S: hemorrhage, heparin induced thrombocytopenia, toxicity/overdose Common meds- warfarin (coumadin) {admin once daily, avoid NSAIDs & aspirin}, enoxaparin (lovenox)

-statin

Antilipidemic aid in lowering LDL & increasing HDL S/S: muscle aches, hepatotoxicity, myopathy, rhabdomyolysis, peripheral neruopathy NI: take in evening, monitor renal and liver function, low fat/high fiber diet, drug interactions: digoxin, warfarin, NSAIDs, etc. Common meds- lovastatin (mevacor)

Seconal 0.1 gram PRN at bedtime is prescribed for rest. The scored tablets are labeled grain 1.5 per tablet. How many tablets should the nurse plan to administer? A. 1/2 tablet B. 1 tablet C. 1 1/2 tablet D. 2 tablets

B. 15 gr = 1 g, 0.1 x 15 = 1.5 grains

The patient has an order for albuterol 5 mg via nebulizer. Available is a solution containing 2 mg/ml. How many milliliters should the nurse use to prepare the patient's dose? A. 0.2 B. 2.5 C. 3.75 D. 5.0

B. 2.5

What is the correct location for the placement of the hand for manual chest compressions during CPR on the adult client. A. Just above the xiphoid process on the upper third of the sternum. B. Below the xiphoid process midway between the sternum and the umbilicus. C. Just about the xiphoid process on the lower third of the sternum. D. Below the xiphoid process midway between the sternum and the first rib.

C.

Chronic

Continuing over a long period of time.

The nurse determines that the patient is not experiencing adverse effects of albuterol (Proventil) after noting which of the following patient vital signs? A. Oxygen saturation 96% B. Respiratory rate of 18 C. Temperature of 98.4° F D. Pulse rate of 76

D. Pulse rate of 76 Albuterol is a β2-agonist that can sometimes cause adverse cardiovascular effects. These would include tachycardia and angina. A pulse rate of 76 indicates that the patient did not experience tachycardia as an adverse effect.

The nurse assesses a patient with shortness of breath for evidence of long-standing hypoxemia by inspecting: A. Chest excursion B. Spinal curvatures C. The respiratory pattern D. The fingernail and its base

D. The fingernail and its base Clubbing, a sign of long-standing hypoxemia, is evidenced by an increase in the angle between the base of the nail and the fingernail to 180 degrees or more, usually accompanied by an increase in the depth, bulk, and sponginess of the end of the finger.

Hyperkalemia

ECG=tall spiked T wave, prolonged QT intervial, widening QRS complex are all signs of hyperkalemia; tumor lysis syndrome

Craniotomy

Incision of the skull.

Otalgia

Pain in the ear.

levodopa (Sinemet)

Parkinsons Disease lessen tremors increases amount of levodopa to CNS (dopamine to the brain) s/s toxicity=dyskinesia, hallucinations, psychosis

Mitochondria

Part of a cell where catabolism primarily occurs.

Endoplasmic Reticulum

Part of the cell where formation of proteins occurs.

Laryngectomy

Removal of the voice box.

-oid

Resembling

-pram, -ine

SSRIs S/S: weight gain, fatigue, sexual dysfunction, drowsiness NI: avoid alcohol, do not discontinue abrubptly, monitor for serotonin syndrome! (agitation, confusion, hallucinations) within first 72 hrs

TSS

Staphlococcus aures produce a toxin that can enter the blood stream through vaginal mucosa. wash hands before and change tampon frequently 4-6 hours

Angioplasty

Surgical repair of blood vessel.

gentamicin sulfate

TX: pneumonia, meningitis, septicemia NI: high risk for ototoxicity, nephrotoxicity, monitor creatinine & BUN

-therapy

Treatment.

-tyline

Tricyclic antidepressant S/S: anticholenergic effects, sedation, toxicity NI: DO NOT admin with MAOIs, avoid alcohol, contradicted in clients w/ seizures Common meds- amitripytyline (elavil)

asthma

a chronic inflammatory lung disease that results in airflow obstruction; characterized by recurring episodes of paroxysmal dyspnea, wheezing on expiration and/or inspiration caused by constriction of the bronchi, coughing, and viscous mucoid bronchial secretions.

trigger

a substance, object, or agent that initiates or stimulates an action; in asthma, any stimuli that initiates the IgE

sundowning

agitated behavior in the evening observe for tirdness at the end of the day

neuro function

altered neuro function =

tuberculosis

an infectious disease caused by Mycobacterium tuberculosis; usually involves the lungs but also occurs in the larynx, kidneys, bones, adrenal glands, lymph nodes, and meninges and can be disseminated throughout the body.

4. A client received spinal anesthesia. Which is most important for the nurse to monitor? a. Loss of consciousness b. Hangover effects and dependence c. Hypotension and headaches d. Excitement or delirium

c. Hypotension and headaches

Patient reports IV discomfort, what is your first action?

color and temperature

esophagogastromy

esophageal cancer risk for infection = meticulious oral care should be provided several times a day prior to surgery

hypercapnia

greater than normal amounts of carbon dioxide in the blood (PaCO2 > 45 mm Hg); also called hypercarbia.

Magnesium Sulphate

hypomagnesemia reccomended for torsadesde pointes a form of polymorphic ventrical tackycardia associated with a prolonged QT intervial that occurs with hypomagnesemia

methotrexate (Mexate)

immunosuppressant can cause bone marrow depression rheumatoid arthritis lab=hemaglobin decrease =adverse side effect

pleurisy (pleuritis)

inflammation of the pleura.

NG Tube

no drainage in 2 hours client nausated = reposition client on side

vibration

pressing on the chest with the flat of the hands while repeatedly tensing the hand and arm muscles to facilitate movement of secretions to larger airways.

chest percussion

rhythmic percussion of a patient's chest with cupped hands to loosen retained respiratory secretions.

Medication for Schizophrenia

risperidone, Risperdal

Fosomax

same as-Alendronate is used for treating osteoporosis in men and postmenopausal women.

crackle

short, low-pitched sounds consisting of discontinuous bubbling caused by air passing through airway intermittently occluded by mucus, unstable bronchial wall, or fold of mucosa; evident on inspiration and, at times, expiration; similar sound to blowing through a straw under water.

dyspnea

shortness of breath; difficulty breathing that may be caused by certain heart conditions, strenuous exercise, or anxiety.

older adults

stooped posture results in upper torso becomming center of gravity

tracheostomy

surgical opening into the trachea through which an indwelling tube may be inserted.

panlobular emphysema

type of emphysema involving distention and destruction of the entire primary respiratory lobule; usually associated with "1-antitrypsin deficiency; also called chronic hypertrophic, diffuse, generalized, panlobular, or vesicular emphysema.

centrilobular emphysema

type of emphysema often associated with chronic bronchitis in which respiratory bronchioles enlarge, the walls are destroyed, and the bronchioles become confluent; characterized by enlargement of air spaces in the proximal part of the acinus, primarily at the level of the respiratory bronchioles.

Bell palsy (7th crainal nerve)

unilateral facial weakness and paralysis

angina pectoris

when walking=1.assist to seated position; 2. sublingual nitroglycerin; 3.oxygen 4. wheelchair to room

953. A client who has been taking buspirone (BuSpar) for 1 month returns to the clinic for a follow-up assessment. A nurse determines that the medication is effective if the absence of which manifestation has occurred? 1. Paranoid thought process 2. Rapid heartbeat or anxiety 3. Alcohol withdrawal symptoms 4. Thought broadcasting or delusions

2. Rapid heartbeat or anxiety

570. The nurse is monitoring the laboratory results of a client receiving an antineoplastic medication by the intravenous route. The nurse plans to initiate bleeding precautions if which laboratory result is noted? 1. A clotting time of 10 minutes 2. An ammonia level of 20 mg/dL 3. A platelet count of 50,000 cells/min 4. A white blood cell count of 5000 cells/mm3

3 Bleeding precautions need to be initiated when the platelet count decreases. The normal paltelet count is 150,000-450,000 cell/mm3. When the platelet count decreased, the client is at risk for bleeding. The normal white blood cell count is 4500-11,000 cellsm/mm3. When the white blood cell count drops, neutropenic precautions need to be implemented. The normal clotting time is 8-15 minutes. The normal ammonia value is 10-80 mcg/dL.

615. Prednisone is prescribed for a client with DM who is taking Humulin NPH insulin daily. Which prescription change does the nurse anticipate during therapy with the prednisone? 1. An additional dose of prednisone daily 2. A decreased amount of daily Humulin NPH insulin 3. An increased amount of daily Humulin NPH insulin 4. The addition of an oral hypoglycemic medication daily

3 Glucocorticoids can elevate blood glucose levels. Clients with DM may need their dosages of insulin or oral hypoglycemic medications increased during glucocorticoid therapy. Therefore, options 1, 2, and 4 are incorrect.

825. The client with myasthenia gravis has become increasingly weaker. The physician prepares to identify whether the client is reacting to an overdose of the medication (cholinergic crisis) or an increasing severity of the disease (myasthenic crisis). An injection of edrophonium (Tensilon) is administered. Which of the following would indicate that the client is in cholinergic crisis? 1. No change in the condition 2. Complaints of muscle spasms 3. An improvement of the weakness 4. A temporary worsening of the condition

4. A temporary worsening of the condition

76.) Carbidopa-levodopa (Sinemet) is prescribed for a client with Parkinson's disease, and the nurse monitors the client for adverse reactions to the medication. Which of the following indicates that the client is experiencing an adverse reaction? 1. Pruritus 2. Tachycardia 3. Hypertension 4. Impaired voluntary movements

4. Impaired voluntary movements Rationale: Dyskinesia and impaired voluntary movement may occur with high levodopa dosages. Nausea, anorexia, dizziness, orthostatic hypotension, bradycardia, and akinesia (the temporary muscle weakness that lasts 1 minute to 1 hour, also known as the "on-off phenomenon") are frequent side effects of the medication.

225.) A nursing student is assigned to care for a client with a diagnosis of schizophrenia. Haloperidol (Haldol) is prescribed for the client, and the nursing instructor asks the student to describe the action of the medication. Which statement by the nursing student indicates an understanding of the action of this medication? 1. It is a serotonin reuptake blocker. 2. It inhibits the breakdown of released acetylcholine. 3. It blocks the uptake of norepinephrine and serotonin. 4. It blocks the binding of dopamine to the postsynaptic dopamine receptors in the brain.

4. It blocks the binding of dopamine to the postsynaptic dopamine receptors in the brain. Rationale: Haloperidol acts by blocking the binding of dopamine to the postsynaptic dopamine receptors in the brain. Imipramine hydrochloride (Tofranil) blocks the reuptake of norepinephrine and serotonin. Donepezil hydrochloride (Aricept) inhibits the breakdown of released acetylcholine. Fluoxetine hydrochloride (Prozac) is a potent serotonin reuptake blocker.

The nurse is caring for a postoperative patient with sudden onset of respiratory distress. The physician orders a STAT ventilation-perfusion scan. Which of the following explanations should the nurse provide to the patient about the procedure? A. This test involves injection of a radioisotope to outline the blood vessels in the lungs, followed by inhalation of a radioisotope gas. B. This test will use special technology to examine cross sections of the chest with use of a contrast dye. C. This test will use magnetic fields to produce images of the lungs and chest. D. This test involves injecting contrast dye into a blood vessel to outline the blood vessels of the lungs.

A. This test involves injection of a radioisotope to outline the blood vessels in the lungs, followed by inhalation of a radioisotope gas.A ventilation-perfusion scan has two parts. In the perfusion portion, a radioisotope is injected into the blood and the pulmonary vasculature is outlined. In the ventilation part, the patient inhales a radioactive gas that outlines the alveoli.

The nurse is caring for a client with a chest tube to water seal drainage that was inserted 10 days ago because of a ruptured bullae and pneumothorax. Which finding should the nurse report to the healthcare provider before the chest tube is removed? A. Tidal of water in the water seal chamber B. Bilateral muffled breath sounds at bases C. Temperature of 101 degrees F D. Absence of chest tube drainage for 2 days.

A. Tidal in the water seal chamber should be reported to the HPC to show that the chest tube is working properly. (B) may indicate hypoventilation from the chest tube and usually improves when the tube is removed. (C) indicates infection (D) is an expected finding.

Which of the following nursing interventions is of the highest priority in helping a patient expectorate thick secretions related to pneumonia? A. Humidify the oxygen as able B. Increase fluid intake to 3L/day if tolerated. C. Administer cough suppressant q4hr. D. Teach patient to splint the affected area.

B. Increase fluid intake to 3L/day if tolerated. Although several interventions may help the patient expectorate mucus, the highest priority should be on increasing fluid intake, which will liquefy the secretions so that the patient can expectorate them more easily. Humidifying the oxygen is also helpful, but is not the primary intervention. Teaching the patient to splint the affected area may also be helpful, but does not liquefy the secretions so that they can be removed.

Cystocele

Hernia of the urinary bladder.

43. Select all that apply. Which of the following is included in a comprehensive respiratory assessment? A. Pulse oximetry B. Chest auscultation C. Apical radial pulse D. Nail-bed assessment E. Evaluation of respiratory effort F. Rate and character of respirations

ABDEF The total assessment of the respiratory system includes pulse oximetry; auscultation; skin and nail-bed assessment for the detection of cyanosis; and rate, character, and degree of effort of respirations. The apical radial pulse is a cardiac assessment.

525. A client with severe acne is seen in the clinic and the health care provider prescribes isotretinoin. The nurse reviews the client's medication record and would contact the HCP if the client is taking which medication? 1. Vitamin A 2. Digoxin 3. Furosemide 4. Phenytoin

Answer: 1 Isotretinoin is a metabolite of vitamin A and can produce generalized intensification of isotretinoin toxicity. Because of the potential for increased toxicity, vitamin A supplements should be discontinued before isotretinoin therapy. Options 2, 3, and 4 are not contraindicated with the use of isotretinoin.

Garamycin-

Antibiotic that is toxic to the kidney, injected for radiology studies.

-azine - setron

Antiemtic reduce N & V S/S: drowsiness, anticholenergic effects, restlessness, tardive dyskinesia, EPS NI: monitor VS Common meds- promethazine (phenergan), metaoclopramide (reglan), ondansertron (zofran)

wheezes

a form of rhonchus characterized by continuous high-pitched squeaking sound caused by rapid vibration of bronchial walls.

pneumoconiosis

a general term for lung diseases caused by inhalation and retention of dust particles.

compliance

a measure of the ease of expansion of the lungs and thorax.

Chapter 21 1. It is important for the nurse teaching the client regarding secobarbital (Seconal) to include which information about secobarbital? a. It is a short-acting drug that may cause one to awaken early in the morning. b. It is an intermediate-acting drug that frequently causes REM rebound. c. It is an intermediate-acting drug that frequently causes a hangover effect. d. It is a long-acting drug that is frequently associated with dependence.

a. It is a short-acting drug that may cause one to awaken early in the morning.

7. For the client who is taking nalbuphine (Nubain), what should the nurse do? (Select all that apply.) a. Monitor any changes in respirations. b. Instruct the client to report bradycardia. c. Administer IV nalbuphine undiluted. d. Explain to the client to expect an excessive amount of urine output. e. Instruct the client to avoid alcohol when taking nalbuphine to avoid respiratory depression.

a. Monitor any changes in respirations. c. Administer IV nalbuphine undiluted. e. Instruct the client to avoid alcohol when taking nalbuphine to avoid respiratory depression.

6. For the client who is taking acetaminophen (Tylenol), what should the nurse do? (Select all that apply.) a. Monitor routine liver enzyme tests. b. Encourage the client to check package labels of OTC drugs to avoid overdosing. c. Teach the diabetic client taking acetaminophen to check blood glucose more frequently. d. Teach the female client that oral contraceptives can increase the effect of acetaminophen. e. Teach the client that caffeine decreases the effects of acetaminophen.

a. Monitor routine liver enzyme tests. b. Encourage the client to check package labels of OTC drugs to avoid overdosing. c. Teach the diabetic client taking acetaminophen to check blood glucose more frequently.

7. A client is taking tacrine (Cognex) to improve cognitive function. What should the nurse teach the client? (Select all that apply.) a. That the client should rise slowly to avoid dizziness b. That obstacles should be removed from pathways to avoid injury c. That the drug dosing schedule should be followed closely d. That the client should be checked frequently for hypertension e. That the client should receive regular liver function tests

a. That the client should rise slowly to avoid dizziness b. That obstacles should be removed from pathways to avoid injury c. That the drug dosing schedule should be followed closely e. That the client should receive regular liver function tests

hemothorax

accumulation of blood in the pleural space.

empyema

an accumulation of purulent exudates in a body cavity, especially the pleural space, as a result of bacterial infection, such as pleurisy or tuberculosis.

cystic fibrosis

an autosomal recessive, multisystem disease characterized by altered function of the exocrine glands involving primarily the lungs, pancreas, and sweat glands.

3. A family member of a client with Alzheimer's disease asks the nurse what causes this disorder. What does the nurse explain is the cause of Alzheimer's disease? a. An excess of acetylcholine b. Neurofibrillary tangles inside neurons c. Degeneration of dopaminergic neurons d. Neuritic plaques that form inside neurons in the cerebellum

b. Neurofibrillary tangles inside neurons

Lisinopril therapeutic effect

blood pressure answer (e.g. 120/80)

permanent pacemaker

changes in pulse rate in rythem may indicate pacer failure dizziness may be due to decreased heart rate leading to decreased cardiac output; should carry a card in wallet with type and serial number of pacemaker; report redness and tenderness - s/s infection

pentobarbital (Neubatal sodium)

contriandicated with liver damage

COPD

contributing factor=smoking

pleural friction rub

creaking or grating sound from roughened, inflamed surfaces of the pleura rubbing together, evident during inspiration, expiration, or both and no change with coughing; usually uncomfortable, especially on deep inspiration.

chest tube

decreased drainage =assess for kinks or dependant loops -do not clamp off

pulmonary hypertension

elevated pulmonary pressure resulting from an increase in pulmonary vascular resistance to blood flow through small arteries and arterioles.

hyperresponsiveness

excessive or exaggerated response to a stimulus; in asthma leads to bronchoconstriction in response to physical, chemical, or pharmacologic stimuli.

hospital-acquired pneumonia

pneumonia occurring 48 hours or longer after hospital admission and not incubating at the time of hospitalization.

cancer

reduce fats increase fruits, vegetables and fiber ie bran flakes, skim milk, orange slices

kidney stone

strain all urine most important encourage urine

trigeminal neuralgia (5th crainal nerve)

sudden stabbing severe pain over the lip and chin

allergic rhinitis

the reaction of the nasal mucosa to a specific allergen.

684. A client has been started on long-term therapy with rifampin (Rifadin). A nurse teaches the client that the medication: 1. Should always be taken with food or antacids 2. Should be double-dosed if one dose is for-gotten 3. Causes orange discoloration of sweat, tears, urine, and feces 4. May be discontinued independently if symptoms are gone in 3 months

3. Causes orange discoloration of sweat, tears, urine, and feces

245.) A client taking carbamazepine (Tegretol) asks the nurse what to do if he misses one dose. The nurse responds that the carbamazepine should be: 1. Withheld until the next scheduled dose 2. Withheld and the health care provider is notified immediately 3. Taken as long as it is not immediately before the next dose 4. Withheld until the next scheduled dose, which should then be doubled

3. Taken as long as it is not immediately before the next dose Rationale: Carbamazepine is an anticonvulsant that should be taken around the clock, precisely as directed. If a dose is omitted, the client should take the dose as soon as it is remembered, as long as it is not immediately before the next dose. The medication should not be double dosed. If more than one dose is omitted, the client should call the health care provider.

731. A client being treated for heart failure is administered intravenous bumetanide. Which outcome indicates the medication has achieved the expected effect? 1. Cough becomes productive of frothy pink sputum 2. The serum potassium level changes from 3.8 to 3.1 mEq/L 3. B-natriuretic peptide (BNP) factor increases from 200 to 262 pg/mL 4. Urine output increases from 10 mL/hour to greater than 50 mL hourly

4 Bumetanide is a diuretic and expected outcomes include increased urine output, decreased crackles, and decreased weight. Options 1, 2, and 3 are incorrect.

364. Rho(D) immune globulin is prescribed for a client after delivery and the nurse provides information to the client about the purpose of the medication. The nurse determines that the woman understands the purpose if the woman states that it will protect her next baby from which condition? 1. Having Rh-positive blood 2. Developing a rubella infection 3. Developing physiological jaundice 4. Being affected by Rh incompatibility

4 Rh incompatibility can occur when an Rh-negative mother becomes sensitized to the Rh antigen. Sensitization may develop when an Rh-negative woman becomes pregnant with a fetus who is Rh positive. During pregnancy and at delivery, some of the fetus's Rh-positive blood can enter the maternal circulation, causing the mother's immune system to form antibodies against Rhpositive blood. Administration of Rho(D) immune globulin (RhoGAM) prevents the mother from developing antibodies against Rhpositive blood by providing passive antibody protection against the Rh antigen.

Digoxin side effects

-Fatigue -Bradycardia -Anorexia -Nausea/Vomiting

Haldol-inform if you are taking ____________ medication.

-benzodiazepine class of anti-anxiety drugs (all ending with "pam") and even, Xanax.

26.) Glimepiride (Amaryl) is prescribed for a client with diabetes mellitus. A nurse reinforces instructions for the client and tells the client to avoid which of the following while taking this medication? 1. Alcohol 2. Organ meats 3. Whole-grain cereals 4. Carbonated beverages

1. Alcohol Rationale: When alcohol is combined with glimepiride (Amaryl), a disulfiram-like reaction may occur. This syndrome includes flushing, palpitations, and nausea. Alcohol can also potentiate the hypoglycemic effects of the medication. Clients need to be instructed to avoid alcohol consumption while taking this medication. The items in options 2, 3, and 4 do not need to be avoided.

826. The nurse is caring for a client with severe back pain. Codeine sulfate has been prescribed for the client. Specific to this medication, which intervention should the nurse include in the plan of care while the client is taking this medication? 1. Monitor radial pulse. 2. Monitor bowel activity. 3. Monitor apical heart rate. 4. Monitor peripheral pulses.

2 While the client is taking codeine sulfate, the nurse would monitor vital signs and assess for hypotension. The nurse also should increase fluid intake, palpate the bladder for urinary retention, auscultate bowel sounds, and monitor the pattern of daily bowel activity and stool consistency because the medication causes constipation. The nurse should monitor respiratory status and initiate deep-breathing and coughing exercises. In addition, the nurse monitors the effectiveness of the pain medication.

9.) The nurse is applying a topical corticosteroid to a client with eczema. The nurse would monitor for the potential for increased systemic absorption of the medication if the medication were being applied to which of the following body areas? 1. Back 2. Axilla 3. Soles of the feet 4. Palms of the hands

2. Axilla Rationale: Topical corticosteroids can be absorbed into the systemic circulation. Absorption is higher from regions where the skin is especially permeable (scalp, axilla, face, eyelids, neck, perineum, genitalia), and lower from regions in which permeability is poor (back, palms, soles).

683. A client is to begin a 6-month course of therapy with isoniazid (INH). A nurse plans to teach the client to: 1. Use alcohol in small amounts only. 2. Report yellow eyes or skin immediately. 3. Increase intake of Swiss or aged cheeses. 4. Avoid vitamin supplements during therapy.

2. Report yellow eyes or skin immediately.

730. A client is receiving thrombolytic therapy with a continuous infusion of streptokinase (Streptase). The client suddenly becomes extremely anxious and complains of itching. A nurse hears stridor and on examination of the client notes generalized urticaria and hypotension. Which of the following should be the priority action of the nurse? 1. Administer oxygen and protamine sulfate. 2. Stop the infusion and call the physician. 3. Cut the infusion rate in half and sit the client up in bed. 4. Administer diphenhydramine (Benadryl) and continue the infusion.

2. Stop the infusion and call the physician.

147.) A client with a prescription to take theophylline (Theo-24) daily has been given medication instructions by the nurse. The nurse determines that the client needs further information about the medication if the client states that he or she will: 1. Drink at least 2 L of fluid per day. 2. Take the daily dose at bedtime. 3. Avoid changing brands of the medication without health care provider (HCP) approval. 4. Avoid over-the-counter (OTC) cough and cold medications unless approved by the HCP.

2. Take the daily dose at bedtime. Rationale: The client taking a single daily dose of theophylline, a xanthine bronchodilator, should take the medication early in the morning. This enables the client to have maximal benefit from the medication during daytime activities. In addition, this medication causes insomnia. The client should take in at least 2 L of fluid per day to decrease viscosity of secretions. The client should check with the physician before changing brands of the medication. The client also checks with the HCP before taking OTC cough, cold, or other respiratory preparations because they could cause interactive effects, increasing the side effects of theophylline and causing dysrhythmias.

726. A client is on nicotinic acid (niacin) for hyperlipidemia and the nurse provides instructions to the client about the medication. Which statement by the client would indicate an understanding of the instructions? 1. "It is not necessary to avoid the use of alcohol." 2. "The medication should be taken with meals to decrease flushing." 3. "Clay-colored stools are a common side effect and should not be of concern." 4. "Ibuprofen (Motrin) taken 30 minutes before the nicotinic acid should decrease the flushing."

4. "Ibuprofen (Motrin) taken 30 minutes before the nicotinic acid should decrease the flushing."

560. A clinic nurse prepares a teaching plan for a client receiving an antineoplastic medication. When implementing the plan, the nurse should make which statement to the client? 1. "You can take aspirin as needed for headache" 2. "You can drink beverages containing alcohol in moderate amounts each evening" 3. "You need to consult with the HCP before receiving immunizations" 4. "It is fine to recive a flu cavvine at the local health fair without HCP approval because the flu is so contagious"

3 Because antineoplastic medications lower the resistance of the body, client must be informed not to receive immunizations without an HCP's approval. Clients also need to avoid contact with individual who have recently received a live virus vaccine. Clients need to avoid aspirin and aspirin-containing products to minimize the risk of bleeding, and they need to avoid alcohol to minimize the risk of toxicity and side/adverse effects.

644. A client has a PRN prescription for loperamide hydrochloride. For which condition should the nurse administer this medication? 1. Constipation 2. Abdominal pain 3. An episode of diarrhea 4. Hematest-positive nasogastric tube drainage

3 Loperamide (Imodium) is an antidiarrheal agent. It is used to manage acute and chronic diarrhea in conditions such as inflammatory bowel disease. Loperamide also can be used to reduce the volume of drainage from an ileostomy. It is not used for the conditions in options 1, 2, and 4.

610. The nurse provides instructions to a client who is taking levothyroxine. The nurse should tell the client to take the medication at which time? 1. With food 2. At lunch time 3. On an empty stomach 4. At bedtime with a snack

3 Oral doses of levothyroxine should be take on an empty stomach to enhance absorption. Dosing should be done in the morning before breakfast.

646. A client has begun medication therapy with pancrelipase. The nurse evaluates that the medication is having the optimal intended benefit if which effect is observed? 1. Weight loss 2. Relief of heartburn 3. Reduction of steatorrhea 4. Absence of abdominal pain

3 Pancrelipase (Pancrease, Creon) is a pancreatic enzyme used in clients with pancreatitis as a digestive aid. The medication should reduce the amount of fatty stools (steatorrhea). Another intended effect could be improved nutritional status. It is not used to treat abdominal pain or heartburn. Its use could result in weight gain but should not result in weight loss if it is aiding in digestion.

734. The nurse should report which assessment finding to the health care provider (HCP) before initiating thrombolytic therapy in a client with pulmonary embolism? 1. Adventitious breath sounds 2. Temperature of 99.4 ° F orally 3. Blood pressure of 198/110 mm Hg 4. Respiratory rate of 28 breaths/minute

3 Thrombolytic therapy is contraindicated in a number of preexisting conditions in which there is a risk of uncontrolled bleeding, similar to the case in anticoagulant therapy. Thrombolytic therapy also is contraindicated in severe uncontrolled hypertension because of the risk of cerebral hemorrhage. Therefore the nurse would report the results of the blood pressure to the HCP before initiating therapy.

726. A client is diagnosed with an ST-segment elevation myocardial infarction (STEMI) and is receiving tissue plasminogen activator, alteplase. Which action is a priority nursing intervention? 1. Monitor for kidney failure. 2. Monitor psychosocial status. 3. Monitor for signs of bleeding. 4. Have heparin sodium available.

3 Tissue plasminogen activator is a thrombolytic. Hemorrhage is a complication of any type of thrombolytic medication. The client is monitored for bleeding. Monitoring for renal failure and monitoring the client's psychosocial status are important but are not the most critical interventions. Heparin may be administered after thrombolytic therapy, but the question is not asking about follow-up medications.

651. A client with a peptic ulcer is diagnosed with a Helicobacter pylori infection. The nurse is teaching the client about the medications prescribed, including clarithromycin, esomeprazole, and moxicillin. Which statement by the client indicates the best understanding of the medication regimen? 1. "My ulcer will heal because these medications will kill the bacteria" 2. "These medications are only taken when I have pain from my ulcer" 3. "The medication will kill the bacteria and stop the acid production" 4. "These medications will coat the ulcer and decrease the acid production in my stomach"

3 Triple therapy for Helicobacter pylori infection usually includes two antibacterial medications and a proton pump inhibitor. Clarithromycin (Biaxin) and amoxicillin (Amoxil) are antibacterials. Esomeprazole (Nexium) is a proton pump inhibitor. These medications will kill the bacteria and decrease acid production.

684. Zafirlukast is prescribed for a client with bronchial asthma. Which laboratory test does the nurse expect to be prescribed before the administration of this medication? 1. Platelet count 2. Neutrophil count 3. Liver function tests 4. Complete blood count

3 Zafirlukast (Accolate) is a leukotriene receptor antagonist used in the prophylaxis and long-term treatment of bronchial asthma. Zafirlukast is used with caution in clients with impaired hepatic function. Liver function laboratory tests should be performed to obtain a baseline, and the levels should be monitored during administration of the medication.

62.) A client is on nicotinic acid (niacin) for hyperlipidemia and the nurse provides instructions to the client about the medication. Which statement by the client would indicate an understanding of the instructions? 1. "It is not necessary to avoid the use of alcohol." 2. "The medication should be taken with meals to decrease flushing." 3. "Clay-colored stools are a common side effect and should not be of concern." 4. "Ibuprofen (Motrin) taken 30 minutes before the nicotinic acid should decrease the flushing."

4. "Ibuprofen (Motrin) taken 30 minutes before the nicotinic acid should decrease the flushing." Rationale: Flushing is a side effect of this medication. Aspirin or a nonsteroidal anti-inflammatory drug can be taken 30 minutes before taking the medication to decrease flushing. Alcohol consumption needs to be avoided because it will enhance this side effect. The medication should be taken with meals, this will decrease gastrointestinal upset. Taking the medication with meals has no effect on the flushing. Clay-colored stools are a sign of hepatic dysfunction and should be immediately reported to the health care provider (HCP).

876. The nurse is caring for a client who has been taking a sulfonamide and should monitor for signs/symptoms of which side/adverse effects of the medication? Select all that apply. 1. Ototoxicity 2. Palpitations 3. Nephrotoxicity 4. Bone marrow depression 5. Gastrointestinal (GI) effects 6. Increased white blood cell (WBC) count

3, 4, 5 Side/adverse effects include nephrotoxicity, bone marrow depression, GI effects, hepatotoxicity, dermatological effects, and some neurological symptoms including headache, dizziness, vertigo, ataxia, depression, and seizures. Options 1, 2, and 6 are unrelated to this medication.

226.) A client receiving lithium carbonate (Lithobid) complains of loose, watery stools and difficulty walking. The nurse would expect the serum lithium level to be which of the following? 1. 0.7 mEq/L 2. 1.0 mEq/L 3. 1.2 mEq/L 4. 1.7 mEq/L

4. 1.7 mEq/L Rationale: The therapeutic serum level of lithium ranges from 0.6 to 1.2 mEq/L. Serum lithium levels above the therapeutic level will produce signs of toxicity.

163.) A client with trigeminal neuralgia tells the nurse that acetaminophen (Tylenol) is taken on a frequent daily basis for relief of generalized discomfort. The nurse reviews the client's laboratory results and determines that which of the following indicates toxicity associated with the medication? 1. Sodium of 140 mEq/L 2. Prothrombin time of 12 seconds 3. Platelet count of 400,000 cells/mm3 4. A direct bilirubin level of 2 mg/dL

4. A direct bilirubin level of 2 mg/dL Rationale: In adults, overdose of acetaminophen (Tylenol) causes liver damage. Option 4 is an indicator of liver function and is the only option that indicates an abnormal laboratory value. The normal direct bilirubin is 0 to 0.4 mg/dL. The normal platelet count is 150,000 to 400,000 cells/mm3. The normal prothrombin time is 10 to 13 seconds. The normal sodium level is 135 to 145 mEq/L.

181.) A client is taking ticlopidine hydrochloride (Ticlid). The nurse tells the client to avoid which of the following while taking this medication? 1. Vitamin C 2. Vitamin D 3. Acetaminophen (Tylenol) 4. Acetylsalicylic acid (aspirin)

4. Acetylsalicylic acid (aspirin) Rationale: Ticlopidine hydrochloride is a platelet aggregation inhibitor. It is used to decrease the risk of thrombotic strokes in clients with precursor symptoms. Because it is an antiplatelet agent, other medications that precipitate or aggravate bleeding should be avoided during its use. Therefore, aspirin or any aspirin-containing product should be avoided.

4.) The camp nurse asks the children preparing to swim in the lake if they have applied sunscreen. The nurse reminds the children that chemical sunscreens are most effective when applied: 1. Immediately before swimming 2. 15 minutes before exposure to the sun 3. Immediately before exposure to the sun 4. At least 30 minutes before exposure to the sun

4. At least 30 minutes before exposure to the sun Rationale: Sunscreens are most effective when applied at least 30 minutes before exposure to the sun so that they can penetrate the skin. All sunscreens should be reapplied after swimming or sweating.

Which abnormal lab finding indicates that a client with diabetes needs further evaluation for diabetic nephropathy? A. Hypokalemia B. Microalbuminauria C. Elevated serum lipids D. Ketonuria

B. Microalbuminuria is the earliest sign of nephropathy and indicates the need for follow-up evaluation. Hyperkalemia (A) is associated with end stage renal disease caused by diabetic nephropathy. (C) may be elevated in end stage renal disease. (D) may signal the onset of DKA.

The patient has an order for albuterol 5 mg via nebulizer. Available is a solution containing 1 mg/ml. How many milliliters should the nurse use to prepare the patient's dose? A. 0.2 B. 2.5 C. 3.75 D. 5.0

D. 5.0

-zosin

HTN/Prostate

status asthmaticus

a severe, life-threatening asthma attack that is refractory to usual treatment and places the patient at risk for developing respiratory failure.

5. The nurse is aware of which fact regarding lorazepam (Ativan)? a. It may cause confusion and blurred vision. b. It has a maximum adult dose of 25 mg/day. c. When combined with cimetidine, it causes plasma levels to be decreased. d. It interferes with the binding of dopamine receptors.

a. It may cause confusion and blurred vision.

acute bronchitis

an inflammation of the lower respiratory tract that is usually due to infection.

epistaxis

nosebleed

Lipitor

-lowers cholesterol in blood, "statins". Reduce LDL and total cholesterol. Raise HDL.

What to understand about Parkinson's Meds?

-they don't cure disease, they slow the process.

609. The nurse is monitoring a client receiving levothyroxine for hypothyroidism. Which findings indicate the presence of a side effect associated with this medication? Select all that apply. 1. Insomnia 2. Weight loss 3. Bradycardia 4. Constipation 5. Mild heat intolerance

1, 2, 5 Insomnia, weight loss, and mild heat intolerance are side effects of levothyroxine. Bradycardia and constipation are not side effects associated with this medication and rather are associated with hypothyroidism, which is the disorder that this medication is prescribed to treat.

115.) A client received 20 units of NPH insulin subcutaneously at 8:00 AM. The nurse should check the client for a potential hypoglycemic reaction at what time? 1. 5:00 PM 2. 10:00 AM 3. 11:00 AM 4. 11:00 PM

1. 5:00 PM Rationale: NPH is intermediate-acting insulin. Its onset of action is 1 to 2½ hours, it peaks in 4 to 12 hours, and its duration of action is 24 hours. Hypoglycemic reactions most likely occur during peak time.

10.) The clinic nurse is performing an admission assessment on a client. The nurse notes that the client is taking azelaic acid (Azelex). Because of the medication prescription, the nurse would suspect that the client is being treated for: 1. Acne 2. Eczema 3. Hair loss 4. Herpes simplex

1. Acne Rationale: Azelaic acid is a topical medication used to treat mild to moderate acne. The acid appears to work by suppressing the growth of Propionibacterium acnes and decreasing the proliferation of keratinocytes. Options 2, 3, and 4 are incorrect.

689. A client has begun therapy with theophylline (Theo-24). A nurse plans to teach the client to limit the intake of which of the following while taking this medication? 1. Coffee, cola, and chocolate 2. Oysters, lobster, and shrimp 3. Melons, oranges, and pineapple 4. Cottage cheese, cream cheese, and dairy creamers

1. Coffee, cola, and chocolate

125.) A nurse is preparing to administer digoxin (Lanoxin), 0.125 mg orally, to a client with heart failure. Which vital sign is most important for the nurse to check before administering the medication? 1. Heart rate 2. Temperature 3. Respirations 4. Blood pressure

1. Heart rate Rationale: Digoxin is a cardiac glycoside that is used to treat heart failure and acts by increasing the force of myocardial contraction. Because bradycardia may be a clinical sign of toxicity, the nurse counts the apical heart rate for 1 full minute before administering the medication. If the pulse rate is less than 60 beats/minute in an adult client, the nurse would withhold the medication and report the pulse rate to the registered nurse, who would then contact the health care provider.

167.) A nurse prepares to reinforce instructions to a client who is taking allopurinol (Zyloprim). The nurse plans to include which of the following in the instructions? 1. Instruct the client to drink 3000 mL of fluid per day. 2. Instruct the client to take the medication on an empty stomach. 3. Inform the client that the effect of the medication will occur immediately. 4. Instruct the client that, if swelling of the lips occurs, this is a normal expected response.

1. Instruct the client to drink 3000 mL of fluid per day. Rationale: Allopurinol (Zyloprim) is an antigout medication used to decrease uric acid levels. Clients taking allopurinol are encouraged to drink 3000 mL of fluid a day. A full therapeutic effect may take 1 week or longer. Allopurinol is to be given with or immediately following meals or milk to prevent gastrointestinal irritation. If the client develops a rash, irritation of the eyes, or swelling of the lips or mouth, he or she should contact the health care provider because this may indicate hypersensitivity.

113.) A nurse is reinforcing discharge instructions to a client receiving sulfisoxazole. Which of the following would be included in the plan of care for instructions? 1. Maintain a high fluid intake. 2. Discontinue the medication when feeling better. 3. If the urine turns dark brown, call the health care provider immediately. 4. Decrease the dosage when symptoms are improving to prevent an allergic response.

1. Maintain a high fluid intake. Rationale: Each dose of sulfisoxazole should be administered with a full glass of water, and the client should maintain a high fluid intake. The medication is more soluble in alkaline urine. The client should not be instructed to taper or discontinue the dose. Some forms of sulfisoxazole cause the urine to turn dark brown or red. This does not indicate the need to notify the health care provider.

29.) A client is taking Humulin NPH insulin daily every morning. The nurse reinforces instructions for the client and tells the client that the most likely time for a hypoglycemic reaction to occur is: 1. 2 to 4 hours after administration 2. 4 to 12 hours after administration 3. 16 to 18 hours after administration 4. 18 to 24 hours after administration

2. 4 to 12 hours after administration Rationale: Humulin NPH is an intermediate-acting insulin. The onset of action is 1.5 hours, it peaks in 4 to 12 hours, and its duration of action is 24 hours. Hypoglycemic reactions most likely occur during peak time.

950. A client receiving tricyclic antidepressants arrives at the mental health clinic. Which observation would indicate that the client is following the medication plan correctly? 1. Client reports not going to work for this past week. 2. Client arrives at the clinic neat and appropriate in appearance. 3. Client complains of not being able to "do anything" anymore. 4. Client reports sleeping 12 hours per night and 3 to 4 hours during the day.

2. Client arrives at the clinic neat and appropriate in appearance.

686. A client with tuberculosis is being started on antituberculosis therapy with isoniazid (INH). Before giving the client the first dose, a nurse ensures that which of the following baseline studies has been completed? 1. Electrolyte levels 2. Liver enzyme levels 3. Serum creatinine level 4. Coagulation times

2. Liver enzyme levels

186.) A nurse prepares to administer sodium polystyrene sulfonate (Kayexalate) to a client. Before administering the medication, the nurse reviews the action of the medication and understands that it: 1. Releases bicarbonate in exchange for primarily sodium ions 2. Releases sodium ions in exchange for primarily potassium ions 3. Releases potassium ions in exchange for primarily sodium ions 4. Releases sodium ions in exchange for primarily bicarbonate ions

2. Releases sodium ions in exchange for primarily potassium ions Rationale: Sodium polystyrene sulfonate is a cation exchange resin used in the treatment of hyperkalemia. The resin either passes through the intestine or is retained in the colon. It releases sodium ions in exchange for primarily potassium ions. The therapeutic effect occurs 2 to 12 hours after oral administration and longer after rectal administration.

216.) A nurse is caring for a client with gout who is taking Colcrys (colchicine). The client has been instructed to restrict the diet to low-purine foods. Which of the following foods should the nurse instruct the client to avoid while taking this medication? 1. Spinach 2. Scallops 3. Potatoes 4. Ice cream

2. Scallops Rationale: Colchicine is a medication used for clients with gout to inhibit the reabsorption of uric acid by the kidney and promote excretion of uric acid in the urine. Uric acid is produced when purine is catabolized. Clients are instructed to modify their diet and limit excessive purine intake. High-purine foods to avoid or limit include organ meats, roe, sardines, scallops, anchovies, broth, mincemeat, herring, shrimp, mackerel, gravy, and yeast.

83.) The client has been on treatment for rheumatoid arthritis for 3 weeks. During the administration of etanercept (Enbrel), it is most important for the nurse to check: 1. The injection site for itching and edema 2. The white blood cell counts and platelet counts 3. Whether the client is experiencing fatigue and joint pain 4. A metallic taste in the mouth, with a loss of appetite

2. The white blood cell counts and platelet counts Rationale: Infection and pancytopenia are side effects of etanercept (Enbrel). Laboratory studies are performed before and during drug treatment. The appearance of abnormal white blood cell counts and abnormal platelet counts can alert the nurse to a potentially life-threatening infection. Injection site itching is a common occurrence following administration. A metallic taste with loss of appetite are not common signs of side effects of this medication.

82.) A client is receiving meperidine hydrochloride (Demerol) for pain. Which of the following are side effects of this medication. Select all that apply. 1. Diarrhea 2. Tremors 3. Drowsiness 4. Hypotension 5. Urinary frequency 6. Increased respiratory rate

2. Tremors 3. Drowsiness 4. Hypotension Rationale: Meperidine hydrochloride is an opioid analgesic. Side effects include respiratory depression, drowsiness, hypotension, constipation, urinary retention, nausea, vomiting, and tremors.

135.) A nurse reinforces medication instructions to a client who is taking levothyroxine (Synthroid). The nurse instructs the client to notify the health care provider (HCP) if which of the following occurs? 1. Fatigue 2. Tremors 3. Cold intolerance 4. Excessively dry skin

2. Tremors Rationale: Excessive doses of levothyroxine (Synthroid) can produce signs and symptoms of hyperthyroidism. These include tachycardia, chest pain, tremors, nervousness, insomnia, hyperthermia, heat intolerance, and sweating. The client should be instructed to notify the HCP if these occur. Options 1, 3, and 4 are signs of hypothyroidism.

238.) Ribavirin (Virazole) is prescribed for the hospitalized child with respiratory syncytial virus (RSV). The nurse prepares to administer this medication via which of the following routes? 1. Orally 2. Via face mask 3. Intravenously 4. Intramuscularly

2. Via face mask Rationale: Ribavirin is an antiviral respiratory medication used mainly in hospitalized children with severe RSV and in high-risk children. Administration is via hood, face mask, or oxygen tent. The medication is most effective if administered within the first 3 days of the infection.

164.) A client receives a prescription for methocarbamol (Robaxin), and the nurse reinforces instructions to the client regarding the medication. Which client statement would indicate a need for further instructions? 1. "My urine may turn brown or green." 2. "This medication is prescribed to help relieve my muscle spasms." 3. "If my vision becomes blurred, I don't need to be concerned about it." 4. "I need to call my doctor if I experience nasal congestion from this medication."

3. "If my vision becomes blurred, I don't need to be concerned about it." Rationale: The client needs to be told that the urine may turn brown, black, or green. Other adverse effects include blurred vision, nasal congestion, urticaria, and rash. The client needs to be instructed that, if these adverse effects occur, the health care provider needs to be notified. The medication is used to relieve muscle spasms.

219.) A health care provider instructs a client with rheumatoid arthritis to take ibuprofen (Motrin). The nurse reinforces the instructions, knowing that the normal adult dose for this client is which of the following? 1. 100 mg orally twice a day 2. 200 mg orally twice a day 3. 400 mg orally three times a day 4. 1000 mg orally four times a day

3. 400 mg orally three times a day Rationale: For acute or chronic rheumatoid arthritis or osteoarthritis, the normal oral adult dose is 400 to 800 mg three or four times daily.

71.) After kidney transplantation, cyclosporine (Sand immune) is prescribed for a client. Which laboratory result would indicate an adverse effect from the use of this medication? 1. Decreased creatinine level 2. Decreased hemoglobin level 3. Elevated blood urea nitrogen level 4. Decreased white blood cell count

3. Elevated blood urea nitrogen level Rationale: Nephrotoxicity can occur from the use of cyclosporine (Sandimmune). Nephrotoxicity is evaluated by monitoring for elevated blood urea nitrogen (BUN) and serum creatinine levels. Cyclosporine is an immunosuppressant but does not depress the bone marrow.

5.) Mafenide acetate (Sulfamylon) is prescribed for the client with a burn injury. When applying the medication, the client complains of local discomfort and burning. Which of the following is the most appropriate nursing action? 1. Notifying the registered nurse 2. Discontinuing the medication 3. Informing the client that this is normal 4. Applying a thinner film than prescribed to the burn site

3. Informing the client that this is normal Rationale: Mafenide acetate is bacteriostatic for gram-negative and gram-positive organisms and is used to treat burns to reduce bacteria present in avascular tissues. The client should be informed that the medication will cause local discomfort and burning and that this is a normal reaction; therefore options 1, 2, and 4 are incorrect

243.) A hospitalized client is having the dosage of clonazepam (Klonopin) adjusted. The nurse should plan to: 1. Weigh the client daily. 2. Observe for ecchymosis. 3. Institute seizure precautions. 4. Monitor blood glucose levels.

3. Institute seizure precautions. Rationale: Clonazepam is a benzodiazepine used as an anticonvulsant. During initial therapy and during periods of dosage adjustment, the nurse should initiate seizure precautions for the client. Options 1, 2, and 4 are not associated with the use of this medication.

52.) A client with tuberculosis is being started on antituberculosis therapy with isoniazid (INH). Before giving the client the first dose, a nurse ensures that which of the following baseline studies has been completed? 1. Electrolyte levels 2. Coagulation times 3. Liver enzyme levels 4. Serum creatinine level

3. Liver enzyme levels Rationale: INH therapy can cause an elevation of hepatic enzyme levels and hepatitis. Therefore, liver enzyme levels are monitored when therapy is initiated and during the first 3 months of therapy. They may be monitored longer in the client who is greater than age 50 or abuses alcohol.

723. A client is diagnosed with an acute myocardial infarction and is receiving tissue plasminogen activator, alteplase (Activase, tPA). Which of the following is a priority nursing intervention? 1. Monitor for renal failure. 2. Monitor psychosocial status. 3. Monitor for signs of bleeding. 4. Have heparin sodium available.

3. Monitor for signs of bleeding

820. The client is taking the prescribed dose of phenytoin (Dilantin) to control seizures. Results of a phenytoin blood level study reveal a level of 35 mcg/mL. Which of the following symptoms would be expected as a result of this laboratory result? 1. Hypotension 2. Tachycardia 3. Slurred speech 4. No symptoms, because this is a normal therapeutic level

3. Slurred speech

187.) A clinic nurse prepares to administer an MMR (measles, mumps, rubella) vaccine to a child. How is this vaccine best administered? 1. Intramuscularly in the deltoid muscle 2. Subcutaneously in the gluteal muscle 3. Subcutaneously in the outer aspect of the upper arm 4. Intramuscularly in the anterolateral aspect of the thigh

3. Subcutaneously in the outer aspect of the upper arm Rationale: The MMR vaccine is administered subcutaneously in the outer aspect of the upper arm. The gluteal muscle is most often used for intramuscular injections. The MMR vaccine is not administered by the intramuscular route.

22.) A nurse is caring for a client after thyroidectomy and notes that calcium gluconate is prescribed for the client. The nurse determines that this medication has been prescribed to: 1. Treat thyroid storm. 2. Prevent cardiac irritability. 3. Treat hypocalcemic tetany. 4. Stimulate the release of parathyroid hormone.

3. Treat hypocalcemic tetany. Rationale: Hypocalcemia can develop after thyroidectomy if the parathyroid glands are accidentally removed or injured during surgery. Manifestations develop 1 to 7 days after surgery. If the client develops numbness and tingling around the mouth, fingertips, or toes or muscle spasms or twitching, the health care provider is notified immediately. Calcium gluconate should be kept at the bedside.

27.) Sildenafil (Viagra) is prescribed to treat a client with erectile dysfunction. A nurse reviews the client's medical record and would question the prescription if which of the following is noted in the client's history? 1. Neuralgia 2. Insomnia 3. Use of nitroglycerin 4. Use of multivitamins

3. Use of nitroglycerin Rationale: Sildenafil (Viagra) enhances the vasodilating effect of nitric oxide in the corpus cavernosum of the penis, thus sustaining an erection. Because of the effect of the medication, it is contraindicated with concurrent use of organic nitrates and nitroglycerin. Sildenafil is not contraindicated with the use of vitamins. Neuralgia and insomnia are side effects of the medication.

945. A client with schizophrenia has been started on medication therapy with clozapine (Clozaril). A nurse assesses the results of which laboratory study to monitor for adverse effects from this medication? 1. Platelet count 2. Blood glucose level 3. White blood cell count 4. Liver function studies

3. White blood cell count

648. A client with a gastric ulcer has a prescription for sucralfate 1 g by mouth four times daily. The nurse should schedule the medication for which times? 1. With meals and at bedtime 2. Every 6 hours around the clock 3. One hour after meals and at bedtime 4. One hour before meals and at bedtime

4 Sucralfate (Carafate) is a gastric protectant. The medication should be scheduled for administration 1 hour before meals and at bedtime. The medication is timed to allow it to form a protective coating over the ulcer before food intake stimulates gastric acid production and mechanical irritation. The other options are incorrect.

19.) Tamoxifen is prescribed for the client with metastatic breast carcinoma. The nurse understands that the primary action of this medication is to: 1. Increase DNA and RNA synthesis. 2. Promote the biosynthesis of nucleic acids. 3. Increase estrogen concentration and estrogen response. 4. Compete with estradiol for binding to estrogen in tissues containing high concentrations of receptors.

4. Compete with estradiol for binding to estrogen in tissues containing high concentrations of receptors. Rationale: Tamoxifen is an antineoplastic medication that competes with estradiol for binding to estrogen in tissues containing high concentrations of receptors. Tamoxifen is used to treat metastatic breast carcinoma in women and men. Tamoxifen is also effective in delaying the recurrence of cancer following mastectomy. Tamoxifen reduces DNA synthesis and estrogen response.

157.) A client receiving nitrofurantoin (Macrodantin) calls the health care provider's office complaining of side effects related to the medication. Which side effect indicates the need to stop treatment with this medication? 1. Nausea 2. Diarrhea 3. Anorexia 4. Cough and chest pain

4. Cough and chest pain Rationale: Gastrointestinal (GI) effects are the most frequent adverse reactions to this medication and can be minimized by administering the medication with milk or meals. Pulmonary reactions, manifested as dyspnea, chest pain, chills, fever, cough, and the presence of alveolar infiltrates on the x-ray, would indicate the need to stop the treatment. These symptoms resolve in 2 to 4 days following discontinuation of this medication. **Eliminate options 1, 2, and 3 because they are similar GI-related side effects. Also, use the ABCs— airway, breathing, and circulation**

159.) A nurse is caring for a client receiving morphine sulfate subcutaneously for pain. Because morphine sulfate has been prescribed for this client, which nursing action would be included in the plan of care? 1. Encourage fluid intake. 2. Monitor the client's temperature. 3. Maintain the client in a supine position. 4. Encourage the client to cough and deep breathe.

4. Encourage the client to cough and deep breathe. Rationale: Morphine sulfate suppresses the cough reflex. Clients need to be encouraged to cough and deep breathe to prevent pneumonia. **ABCs—airway, breathing, and circulation**

36.) The client has a PRN prescription for ondansetron (Zofran). For which condition should this medication be administered to the postoperative client? 1. Paralytic ileus 2. Incisional pain 3. Urinary retention 4. Nausea and vomiting

4. Nausea and vomiting Rationale: Ondansetron is an antiemetic used to treat postoperative nausea and vomiting, as well as nausea and vomiting associated with chemotherapy. The other options are incorrect.

184.) A nurse preparing a client for surgery reviews the client's medication record. The client is to be nothing per mouth (NPO) after midnight. Which of the following medications, if noted on the client's record, should the nurse question? 1. Cyclobenzaprine (Flexeril) 2. Alendronate (Fosamax) 3. Allopurinol (Zyloprim) 4. Prednisone

4. Prednisone Rationale: Prednisone is a corticosteroid that can cause adrenal atrophy, which reduces the body's ability to withstand stress. Before and during surgery, dosages may be temporarily increased. Cyclobenzaprine is a skeletal muscle relaxant. Alendronate is a bone-resorption inhibitor. Allopurinol is an antigout medication.

144.) A client is taking docusate sodium (Colace). The nurse monitors which of the following to determine whether the client is having a therapeutic effect from this medication? 1. Abdominal pain 2. Reduction in steatorrhea 3. Hematest-negative stools 4. Regular bowel movements

4. Regular bowel movements Rationale: Docusate sodium is a stool softener that promotes the absorption of water into the stool, producing a softer consistency of stool. The intended effect is relief or prevention of constipation. The medication does not relieve abdominal pain, stop gastrointestinal (GI) bleeding, or decrease the amount of fat in the stools.

44.) A client is receiving acetylcysteine (Mucomyst), 20% solution diluted in 0.9% normal saline by nebulizer. The nurse should have which item available for possible use after giving this medication? 1. Ambu bag 2. Intubation tray 3. Nasogastric tube 4. Suction equipment

4. Suction equipment Rationale: Acetylcysteine can be given orally or by nasogastric tube to treat acetaminophen overdose, or it may be given by inhalation for use as a mucolytic. The nurse administering this medication as a mucolytic should have suction equipment available in case the client cannot manage to clear the increased volume of liquefied secretions.

822. A client with trigeminal neuralgia is being treated with carbamazepine (Tegretol), 400 mg orally daily. Which of the following indicates that the client is experiencing a side effect to the medication? 1. Uric acid level, 5 mg/dL 2. Sodium level, 140 mEq/L 3. Blood urea nitrogen level, 15 mg/dL 4. White blood cell count, 3000/mm3

4. White blood cell count, 3000/mm3

-cycline, -floxacin

Antibiotic

-olol

Beta Blocker inhibit stimulation of receptor sites= decreased cardiac excitability, CO, myocaridal O2 demand, lower BP by decreasing release of renin in the kidney TX: HTN, angina, tachydysryhmias, HF, MI S/S: Bradycardia, Bradypena, Bronchospasms, decreased BP NI: Monitor DM for hypoglycemia Common meds- metropolol, labetalol, propanolol

-prazole

Proton pump inhibitor S/S: D,V, N, can increase risk for fractures,, pneumonia, & acid rebound NI: DO NOT crush, chew, break, notify PROVIDER if GI bleeding! Common meds- omepazole (prilosec)

Liver

RUQ contain the _______.

omepazole (prilosec)

S/S: D,V, N, can increase risk for fractures,, pneumonia, & acid rebound NI: DO NOT crush, chew, break, notify PROVIDER if GI bleeding!

diphenhydramine (benadryl), loratadine (claratin), cetirizine (zyrtec), fexofenadrine (allegra)

S/S: anticholenergic effects (cant see, spit, pee, poop), drowsiness NI: use cautiously pts w/ HTN, PUD, urinary retention, assess hypokalemia, BP, Advise to take @ night

Arteriole

Small artery.

-ase

Thrombolytic dissolves clots TX: acute MI, DVT, massive PE, ischemic stroke S/S: serious bleeding risks from recent wounds, puncture sites, weakened vessels, hypotension NI: Must take 4-6 hrs of onset Common meds- alteplase (activase, tPA)

Myelogram

X-ray record of the spinal cord.

surfactant

a lipoprotein that lowers the surface tension in the alveoli, reduces the amount of pressure needed to inflate the alveoli, and decreases the tendency of the alveoli to collapse.

8. The nurse should know that which drugs are used to treat migraine attacks? a. Triptans b. Anticonvulsants c. Tricyclic antidepressants d. Beta-adrenergic blockers

a. Triptans

What medication to administer with Tylenol overdose?

acetylcysteine (Mucomyst) must be given IV

208.) A client with myasthenia gravis verbalizes complaints of feeling much weaker than normal. The health care provider plans to implement a diagnostic test to determine if the client is experiencing a myasthenic crisis and administers edrophonium (Enlon). Which of the following would indicate that the client is experiencing a myasthenic crisis? 1. Increasing weakness 2. No change in the condition 3. An increase in muscle spasms 4. A temporary improvement in the condition

auto-define "A client with myasthen..." Rationale: Edrophonium (Enlon) is administered to determine whether the client is reacting to an overdose of a medication (cholinergic crisis) or to an increasing severity of the disease (myasthenic crisis). When the edrophonium (Enlon) injection is given and the condition improves temporarily, the client is in myasthenic crisis. This is known as a positive test. Increasing weakness would occur in cholinergic crisis. Options 2 and 3 would not occur in either crisis.

nasal polyps

benign mucous membrane masses that form slowly in response to repeated inflammation of the sinus or nasal mucosa and project into the nasal cavity.

2. Several children are admitted for diagnosis with possible attention deficit/hyperactivity disorder. Which is most important for the nurse to observe? a. A girl who is lethargic b. A girl who lacks impulsivity c. A boy with smooth coordination d. A boy with an inability to complete tasks

d. A boy with an inability to complete tasks

adventitious sounds

extra breath sounds that are not normally heard, such as crackles, rhonchi, wheezes, and pleural friction rubs.

elastic recoil

the tendency for the lungs to recoil or reduce in volume after being stretched or expanded.

Macrodantin medication

used to treat or prevent certain urinary tract infections

766. Tacrolimus is prescribed for a client. Which disorder, if noted in the client's record, would indicate that the medication needs to be administered with caution? 1. Pancreatitis 2. Ulcerative colitis 3. Diabetes insipidus 4. Coronary artery disease

1 Tacrolimus (Prograf) is used with caution in immunosuppressed clients and in clients with renal, hepatic, or pancreatic function impairment. Tacrolimus is contraindicated in clients with hypersensitivity to this medication or hypersensitivity to cyclosporine.

854. The nurse is analyzing the laboratory studies on a client receiving dantrolene sodium. Which laboratory test would identify an adverse effect associated with the administration of this medication? 1. Platelet count 2. Creatinine level 3. Liver function tests 4. Blood urea nitrogen level

3 Dose-related liver damage is the most serious adverse effect of dantrolene. To reduce the risk of liver damage, liver function tests should be performed before treatment and throughout the treatment interval. Dantrolene is administered at the lowest effective dosage for the shortest time necessary.

emphysema

an abnormal condition of the pulmonary system, characterized by overinflation and destructive changes in alveolar walls.

221.) A nurse is reviewing the health care provider's prescriptions for an adult client who has been admitted to the hospital following a back injury. Carisoprodol (Soma) is prescribed for the client to relieve the muscle spasms; the health care provider has prescribed 350 mg to be administered four times a day. When preparing to give this medication, the nurse determines that this dosage is: 1. The normal adult dosage 2. A lower than normal dosage 3. A higher than normal dosage 4. A dosage requiring further clarification

1. The normal adult dosage Rationale: The normal adult dosage for carisoprodol is 350 mg orally three or four times daily.

41.) The client has been taking omeprazole (Prilosec) for 4 weeks. The ambulatory care nurse evaluates that the client is receiving optimal intended effect of the medication if the client reports the absence of which symptom? 1. Diarrhea 2. Heartburn 3. Flatulence 4. Constipation

2. Heartburn Rationale: Omeprazole is a proton pump inhibitor classified as an antiulcer agent. The intended effect of the medication is relief of pain from gastric irritation, often called heartburn by clients. Omeprazole is not used to treat the conditions identified in options 1, 3, and 4.

879. Amikacin is prescribed for a client with a bacterial infection. The nurse instructs the client to contact the health care provider (HCP) immediately if which occurs? 1. Nausea 2. Lethargy 3. Hearing loss 4. Muscle aches

3 Amikacin (Amikin) is an aminoglycoside. Side/adverse effects of aminoglycosides include ototoxicity (hearing problems), confusion, disorientation, gastrointestinal irritation, palpitations, blood pressure changes, nephrotoxicity, and hypersensitivity. The nurse instructs the client to report hearing loss to the HCP immediately. Lethargy and muscle aches are not associated with the use of this medication. It is not necessary to contact the HCP immediately if nausea occurs. If nausea persists or results in vomiting, the HCP should be notified.

229.) A client who is taking lithium carbonate (Lithobid) is scheduled for surgery. The nurse informs the client that: 1. The medication will be discontinued a week before the surgery and resumed 1 week postoperatively. 2. The medication is to be taken until the day of surgery and resumed by injection immediately postoperatively. 3. The medication will be discontinued 1 to 2 days before the surgery and resumed as soon as full oral intake is allowed. 4. The medication will be discontinued several days before surgery and resumed by injection in the immediate postoperative period.

3. The medication will be discontinued 1 to 2 days before the surgery and resumed as soon as full oral intake is allowed. Rationale: The client who is on lithium carbonate must be off the medication for 1 to 2 days before a scheduled surgical procedure and can resume the medication when full oral intake is prescribed after the surgery. **lithium carbonate is an oral medication and is not given as an injection**

166.) Alendronate (Fosamax) is prescribed for a client with osteoporosis. The client taking this medication is instructed to: 1. Take the medication at bedtime. 2. Take the medication in the morning with breakfast. 3. Lie down for 30 minutes after taking the medication. 4. Take the medication with a full glass of water after rising in the morning.

4. Take the medication with a full glass of water after rising in the morning. Rationale: Precautions need to be taken with administration of alendronate to prevent gastrointestinal side effects (especially esophageal irritation) and to increase absorption of the medication. The medication needs to be taken with a full glass of water after rising in the morning. The client should not eat or drink anything for 30 minutes following administration and should not lie down after taking the medication.

3. For the client receiving periodic morphine IV push, which is most critical for the nurse to monitor? a. Fever b. Diarrhea c. Respirations d. White blood cell count

c. Respirations

2. The nurse realizes that some herbs interact with selective serotonin reuptake inhibitors (SSRIs). Which herb interaction may cause serotonin syndrome? a. feverfew b. ma-huang c. St. John's wort d. gingko biloba

c. St. John's wort

4. A client with multiple sclerosis is in the chronic progressive phase. The nurse should expect which drug to be most helpful at this time? a. Interferon β-1a (Avonex, Rebif) b. Glucocorticoids c. Azathioprine (Imuran) d. Cyclophosphamide (Cytoxan)

d. Cyclophosphamide (Cytoxan)

chronic pancreatitis

pulmonary disease state characterized by the presence of airflow obstruction caused by chronic bronchitis or emphysema; clinical use of the term indicates the presence of chronic bronchitis and/or emphysema; includes asthma, chronic bronchiectasis, chronic bronchitis, and emphysema.

dry powder inhaler

dry powdered drug delivered by inhalation.

Food to avoid when taking Lithium

-salty foods -alcoholic beverages

Skin cell

An epithelial cell is a (an)

4. A client is taking clonazepam (Klonopin) for absence (petit mal) seizures. Which value(s) should the nurse report as outside the therapeutic range for clonazepam? (Select all that apply.) a. 5 mcg/mL b. 15 mcg/mL c. 60 ng/mL d. 120 ng/mL

a. 5 mcg/mL b. 15 mcg/mL d. 120 ng/mL

3.A client is prescribed metoprolol (Lopressor) to treat hypertension. It is important for the nurse to monitor the client for which condition? a. Bradycardia b. Hypertension c. Ankle edema d. Decreased respirations

a. Bradycardia

2. The nurse understands that antipsychotics act in which way? a. By blocking actions of dopamine b. By blocking actions of epinephrine c. By promoting prostaglandin synthesis d. By enhancing the action of gamma-aminobutyric acid

a. By blocking actions of dopamine

719. A client who is receiving digoxin (Lanoxin) daily has a serum potassium level of 3 mEq/L and is complaining of anorexia. A physician prescribes a digoxin level to rule out digoxin toxicity. A nurse checks the results, knowing that which of the following is the therapeutic serum level (range) for digoxin? 1. 0.5 to 2 ng/mL 2. 1.2 to 2.8 ng/mL 3. 3 to 5 ng/mL 4. 3.5 to 5.5 ng/mL

1. 0.5 to 2 ng/mL

4. A client is taking tranylcypromine sulfate (Parnate) for depression. What advice should the nurse include in the teaching plan for this medication? a. Warn of severe hypotension. b. Avoid beer and cheddar cheese. c. Encourage ginseng and ephedra. d. Encourage fruit such as bananas.

b. Avoid beer and cheddar cheese.

2. For the client receiving pyridostigmine administration, the nurse should monitor for which adverse reaction? a. Hypertension b. Bronchospasm c. Thrombocytopenia d. Stevens-Johnson syndrome

b. Bronchospasm

826. Meperidine hydrochloride (Demerol) has been prescribed for a client to treat pain. Select the side effects of this medication. Select all that apply. 1. Diarrhea 2. Tremors 3. Drowsiness 4. Hypotension 5. Urinary frequency 6. Increased respiratory rate

2. Tremors 3. Drowsiness 4. Hypotension

42. Select all that apply. Which of the following nursing actions can help clear tracheobronchial secretions in a patient with cystic fibrosis? A. Postural drainage B. Suppressing the cough C. Ensuring adequate hydration D. Administering mucolytic aerosols E. Encouraging the patient to lie flat F. Administering water-soluble vitamins

ACD Postural drainage, adequate hydration, and administration of mucolytic aerosols all encourage coughing and the clearing of secretions. A patient with cystic fibrosis will be more comfortable sitting upright.

To prevent atelectasis in an 82-year-old patient with a hip fracture, a nurse should A. supply oxygen. B. suction the upper airway. C. ambulate the patient frequently. D. assist the patient with aggressive coughing and deep breathing.

D. assist the patient with aggressive coughing and deep breathing. Decreased mobility after surgery in older adults creates the possibility of fluid buildup and retention in lung tissue. One of the primary goals of nursing intervention is to prevent atelectasis in a high-risk patient. Aggressive coughing and deep breathing can prevent atelectasis in the postoperative patient.

Acromegaly

Enlargement of extremities after puberty due to pituitary gland problem.

Diaphragm

Muscular wall separating the abdominal and thoracic cavities.

Duloxetine (Cymbalta) Fluoxetine (Provac) Escitalopram (Lexapro) Sertraline (Zoloft)

S/S: weight gain, fatigue, sexual dysfunction, drowsiness NI: avoid alcohol, do not discontinue abrubptly, monitor for serotonin syndrome! (agitation, confusion, hallucinations) within first 72 hrs

-iprazole -apine -idone

Second Generation Antipsychotic (SGA)

Pleural Cavity

Space between the membranes around the lungs.

deviated septum

a deflection of the normally straight nasal septum.

NEUPOGEN (filgrastim)-what is the appropriate route of this med?

administered by subcutaneous injection or IV infusion

7. A client is taking valproic acid (Depakote). The nurse should monitor the client for a which therapeutic serum range? a. 10 to 20 mcg/mL b. 15 to 40 mcg/mL c. 20 to 80 ng/mL d. 40 to 100 mcg/mL

d. 40 to 100 mcg/mL

olpthalmoscopic exam

from a distance of 8-12 inches and slightly to the side, shine the light in the clients pupil; client should focus on distant object to promote dialation, olpthalmoscopic set at 0 lens to begin, should be held in front of the examiners left eye when examining clients right eye and kept within 1 inch of clients eye for optimum visualation

diverticulitits

hard ridgid abdomen & elevated WBC = peritonitis = medical emergency should be reported to PCP immediately s/s left lower quadrant pain; elevated temperature; refusing to eat; nausea

Hepatitis B

health care providers should have Hep B vaccine; transmitted by fecal/oral contamination

PVD peripheral vascular disease

help client dangle legs

RBC Blood transfusion

http://www.atitesting.com/ati_next_gen/FocusedReview/data/datacontext/RM%20AMS%20RN%208.0%20Chp%2044.pdf (prime with normal saline and infuse with sodium chloride).

Valporic Acid lab

liver

rednisone (deltasone), betamethasone (celestone), hydrocortisone sodium succinate (Solu-cortef), Methylprednisolone sodium succinate (solu-medrol), fluticasone propionate (advair, flovent)

prevent inflammatory response S/S: Hyperglycemia, peptic ulcer, fluid retention (increased appetite), withdrawal symptoms, euphoria, insomnia, psychotic behavior NI: admin w/ meals, DO NOT take with NSAIDS, teach DO NOT stop abruptly

CKD chronic kidney disease

prior to hemodialysis lab= hypocalcemia due to hyperphosphatemia, hyperkalemic & hypernatremic

older adults

protein found in urine slightly rises as a result of kidney change or UTI w/asymptomatic bacteriuria and pyuria as a result of incomplete bladder emptying

promethazine (phenergan), metaoclopramide (reglan), ondansertron (zofran)

reduce N & V S/S: drowsiness, anticholenergic effects, restlessness, tardive dyskinesia, EPS NI: monitor VS

Pap smear

should be continued through menapause to test for vaginal and cervical cancer

telemetry

ventrical fibrillation = life threatening start CPR

fremitus

vibration of the chest wall produced by vocalization.

What lab values should a nurse monitor for a patient with chronic renal failure?

■ Urinalysis ☐ Hematuria, proteinuria, and alterations in specific gravity ☐ Serum creatinine - Gradual increase of 1 to 2 mg/dL per every 24 to 48 hr for acute renal failure (ARF) - Gradual increase over months to years for chronic renal failure (CRF) exceeding 4 mg/dL ■ Blood urea nitrogen (BUN) - 80 to 100 mg/dL within 1 week with ARF - Gradual increase with elevated serum creatinine over months to years for CRF - 180-200 mg/dL with (CRF) ■ Serum electrolytes - Decreased sodium (dilutional) and calcium, increased potassium, phosphorus, and magnesium ■ Complete blood count (CBC) - Decreased hemoglobin

Cell Membrane

Allows materials to pass into and out of the cell.

Ischemia

Blood is held back from an area.

esophageal speech

a method of swallowing air, trapping it in the esophagus, and releasing it to create sound.

Lithium report immediately

slurred speech

thoracotomy

surgical opening into the thoracic cavity.

Hypochondriac regions

The upper lateral regions of the abdomen, beneath the ribs.

948. A nurse is administering risperidone (Risperdal) to a client who is scheduled to be discharged. Before discharge, which of the following should the nurse teach the client? 1. Get adequate sunlight. 2. Avoid foods rich in potassium. 3. Continue driving as usual. 4. Get up slowly when changing positions.

4. Get up slowly when changing positions.

Malignant myeloma

Tumor of bone marrow.

-cillin

Penicillin TX: pneumonia, upper respiratory infections, septicemia, endocarditis, rheumatic fever, GYN infections NI: hypersensitivity w/ poss. anaphylaxis

postural drainage

the use of various positions to promote gravity drainage of bronchial secretions; coughing usually expels secretions of the trachea.

meniere syndrome (8th crainal nerve)

tinnitus, vertigo, eharing difficulties

ECG

U wave is positive deflectionfollowing the T wave often present in hypokalemia tall spiked T wave, prolonged QT intervial, widening QRS complex are all signs of hyperkalemia

tracheotomy

a surgical incision into the trachea for the purpose of establishing an airway; performed below a blockage by a foreign body, tumor, or edema of the glottis.

thoracentesis

a surgical procedure done to remove fluid from the pleural space.

pursed-lip breathing

a technique of exhaling against pursed lips to prolong exhalation, preventing bronchiolar collapse and air trapping; done to increase expiratory airway pressure, improve oxygenation of the blood, and help prevent early airway closure.

pulmonary embolism

a thromboembolic occlusion of the pulmonary vasculature resulting from thrombi in the venous circulation or right side of the heart and from other sources, such as amniotic fluid, air, fat, bone marrow, and foreign IV material that travel as emboli until lodging in the pulmonary vessels.

7. A client appears to have had an overdose of phenothiazines. The nurse is aware that the potential treatment for phenothiazine overdose includes which intervention(s)? (Select all that apply.) a. Gastric lavage b. Adequate hydration c. Maintaining an airway d. fluphenazine (Prolixin) e. risperidone (Risperdal) f. Activated charcoal administration

a. Gastric lavage b. Adequate hydration c. Maintaining an airway f. Activated charcoal administration

Chapter 23 1. A client with parkinsonism asks the nurse to explain what causes this condition. The most accurate response by the nurse is that parkinsonism is caused by the degeneration of which? a. Cholinergic neurons b. Dopaminergic neurons c. Acetylcholine neurotransmitters d. Monamine oxidase-B neurotransmitters

b. Dopaminergic neurons

3. A client is taking zolpidem (Ambien) for insomnia. The nurse prepares a care plan that includes monitoring of the client for side effects/adverse reactions of this drug. Which is a side effect of zolpidem? a. Insomnia b. Headache c. Laryngospasm d. Blood dyscrasias

b. Headache

5. A nurse is teaching a client about zolpidem. Which is important for the nurse to include in the teaching of this drug? a. Maximum dose is 20 mg/d b. May lead to psychological dependence c. For older adults, dose is 15 mg at bedtime d. Should only be used for 21 days or less

b. May lead to psychological dependence

769. A client with chronic kidney disease is receiving epoetin alfa. Which laboratory result would indicate a therapeutic effect of the medication? 1. Hematocrit of 32% 2. Platelet count of 400,000 cells/mm3 3. Blood urea nitrogen level of 15 mg/dL 4. White blood cell count of 6000 cells/mm3

1 Epoetin alfa is used to reverse anemia associated with chronic kidney disease. Therapeutic effect is seen when the hematocrit is between 30% and 33%. Options 2, 3, and 4 are not associated with the action of this medication.

Chapter 22 1. The nurse witnesses a client's seizure involving generalized contraction of the body followed by jerkiness of arms and legs. The nurse reports that this is which type of seizure? a. Myoclonic b. Petit mal c. Tonic clonic d. Psychomotor

c. Tonic clonic

3. An antipsychotic agent, fluphenazine (Prolixin), is ordered for a client with psychosis. The nurse knows that this agent can lead to extrapyramidal symptoms (EPS) that may be treated with which medication? a. quetiapine (Seroquel) b. aripiprazole (Abilify) c. benztropine (Cogentin) d. chlorpromazine (Thorazine)

c. benztropine (Cogentin)

Chapter 26 1. The nurse knows that which medication will cause the least gastrointestinal distress? a. aspirin b. ketorolac c. celecoxib d. ibuprofen

c. celecoxib

7. A newborn client is in respiratory distress. The nurse anticipates preparation for which medication to be given? a. modafinil b. armodafinil c. theophylline d. amphetamine

c. theophylline

digitalis (Lanoxin) digoxin

cardiac glycoside can build up toxic levels s/s anexoria, nausea, vomiting, diarrhea, headache, fatigue

hypomagnesemia

chronic alcholic ie HR 160 BP 90/54 give IV magnesium sulphate prolonged QT intervial

rhonchi

continuous rumbling, snoring, or rattling sounds from obstruction of large airways with secretions; most prominent on expiration; change often evident after coughing or suctioning.

ACE inhibitor

cough os a common side effect hypertension do not stop abruptly (rebound hypertension may occur)

Laryngectomy

cuff should be inflated only prior to feeding

alteplase (activase, tPA)

dissolves clots TX: acute MI, DVT, massive PE, ischemic stroke S/S: serious bleeding risks from recent wounds, puncture sites, weakened vessels, hypotension NI: Must take 4-6 hrs of onset

cor pulmonale

hypertrophy of the right side of the heart, with or without heart failure, resulting from pulmonary hypertension.

Aldosteronism

lab =decreased serum level of potassium hypokalemia hypertension

Lymphocyte

A blood cell that produces antibodies.

-pril

ACE inhibitor Block the conversion of angiotensin I to angiotensin II TX: HTN, HF, MI, diabetic nephropathy S/S: Anigoedema, Cough, Electrolyte imbalance (^k+) NI: Monitor K+ levels, BP Common med- catopril, lisinopril, enalapril (vastotec)

The nurse is scheduled to give a dose of salmeterol by metered dose inhaler (MDI). The nurse would administer the right drug by selecting the inhaler with which of the following trade names? A. Vanceril B. Serevent C. AeroBid D. Atrovent

B. Serevent The trade or brand name for salmeterol, an adrenergic bronchodilator, is Serevent.

The most appropriate position to assist a patient with chronic obstructive pulmonary disease (COPD) who is having difficulty breathing would be a A. high Fowler's position without a pillow behind the head. B. semi-Fowler's position with a single pillow behind the head. C. right side-lying position with the head of the bed at 45 degrees' elevation. D. sitting upright and forward position with arms supported on an over-the-bed table.

D. sitting upright and forward position with arms supported on an over-the-bed table.Sitting upright and leaning forward with arms supported on an over-the-bed table would be of most help to this patient, because it allows for expansion of the thoracic cage in all four directions (front, back, and two sides).

When administering oxygen to a patient with COPD with the potential for carbon dioxide narcosis, the nurse should A. never administer oxygen at a rate of more than 2 L/min. B. monitor the patient's use of oxygen to detect oxygen dependency. C. monitor the patient for symptoms of oxygen toxicity, such as paresthesias. D. use ABGs as a guide to determine what FIO2 level meets the patient's needs.

D. use ABGs as a guide to determine what FIO2 level meets the patient's needs. It is critical to start oxygen at low flow rates and then use ABGs as a guide to determine what FIO2 level is sufficient and can be tolerated.

-afil

Erectile dysfunction s/s: headache, heartburn, diarrhea, flushing, nosebleeds, parathesias, changes in color vision Contradicted in clients taking nitrates, anticoags, anti HTN Common meds- sildenafil (viagra)

2.The nurse will monitor the client taking albuterol (Proventil) for which condition? a. Palpitations b. Hypoglycemia c. Bronchospasm d. Uterine contractions

a. Palpitations

tumor lysis syndrome

hyperkalemia may occur =requires insulin to reduce serum potassium = monitor serium potassium and blood glucose levels

692. A client has begun therapy with theophylline. The nurse should plan to teach the client to limit the intake of which items while taking this medication? 1. Coffee, cola, and chocolate 2. Oysters, lobster, and shrimp 3. Melons, oranges, and pineapple 4. Cottage cheese, cream cheese, and dairy creamers

1 Theophylline (Theo-24) is a methylxanthine bronchodilator. The nurse teaches the client to limit the intake of xanthinecontaining foods while taking this medication. These foods include coffee, cola, and chocolate.

neurogenic bladder

infection - from stasis of urine and subsuquent catheterization

767. The nurse is reviewing the laboratory results for a client receiving tacrolimus. Which laboratory result would indicate to the nurse that the client is experiencing an adverse effect of the medication? 1. Blood glucose of 200 mg/dL 2. Potassium level of 3.8 mEq/L 3. Platelet count of 300,000 cells/mm3 4. White blood cell count of 6000 cells/mm3

1 A blood glucose level of 200 mg/dL is significantly elevated above the normal range of 70 to 110 mg/dL and suggests an adverse effect. Other adverse effects include neurotoxicity evidenced by headache, tremor, and insomnia; gastrointestinal effects such as diarrhea, nausea, and vomiting; hypertension; and hyperkalemia.

694. The nurse is caring for a client with a diagnosis of influenza who first began to experience symptoms yesterday. Antiviral therapy is prescribed and the nurse provides instructions to the client about the therapy. Which statement by the client indicates an understanding of the instructions? 1. "I must take the medication exactly as prescribed." 2. "Once I start the medication, I will no longer be contagious." 3. "I will not get any colds or infections while taking this medication." 4. "This medication has minimal side effects and I can return to normal activities."

1 Antiviral medications for influenza must be taken exactly as prescribed. These medications do not prevent the spread of influenza and clients are usually contagious for up to 2 days after the initiation of antiviral medications. Secondary bacterial infections may occur despite antiviral treatment. Side effects occur with these medications and may necessitate change in activities, especially when driving or operating machinery if dizziness occurs.

562. The nurse is reviewing the history and physical examination of a client who will be receiving asparaginase, an antineuoplastic agent. The nurse contacts the HCP before administering the medication if which disorder is documented in the client's history? 1. Pancreatitis 2. Diabetes mellitus 3. Myocardial infarction 4. Chronic obstructive pulmonary disease

1 Asparaginase is contraindicated if hypersensitivity exists, in pancreatitis, or if the client has a history of pancreatitis. The medication impairs pancreatic function and pancreatic function tests should be performed before therapy begins and when a week or more has elapsed between dose administrations. The client needs to be monitored for signs of pancreatitis, which include nausea, vomiting, and abdominal pain. The conditions noted in options 2, 3, and 4 are not contraindicated with this medication.

607. The community health nurse visits a client at home. Prednisone, 10 mg orally daily, has been prescribed for the client and the nurse teaches the client about the medication. Which statement, if made by the client, indicates that further teaching is necessary? 1. "I can take aspirin or my antihistamine if I need it" 2. "I need to take the medication every day at the same time" 3. "I need to avoid coffee, tea, cola, and chocolate in my diet" 4. "If I gain more than 5 pounds a week, I will call my health care provider"

1 Aspirin and other over the counter medications should not be taken unless the client consults with the HCP. The client needs to take the medication at the same time every day and should be instructed not to stop the medication. A slight weight gain as a result of an improved appetite is expected, but after the dosage is stabilized, a weight gain of 5 pounds or more weekly should be reported to the HCP. Caffeine-containing foods and fluids need to be avoided because they may contribute to steroid-ulcer development.

855. Cyclobenzaprine hydrochloride is prescribed for a client for muscle spasms and the nurse is reviewing the client's record. Which disorder, if noted in the record, would indicate a need to contact the health care provider about the administration of this medication? 1. Glaucoma 2. Emphysema 3. Hypothyroidism 4. Diabetes mellitus

1 Because cyclobenzaprine (Flexeril) has anticholinergic effects, it should be used with caution in clients with a history of urinary retention, glaucoma, and increased intraocular pressure. Cyclobenzaprine should be used only for a short time (2 to 3 weeks). The conditions in options 2, 3, and 4 are not a concern with this medication.

792. The nurse prepares a client for an ear irrigation as prescribed by the health care provider. Which action should the nurse take when performing the procedure? 1. Warm the irrigating solution to 98.6° F. 2. Position the client with the affected side up following the irrigation. 3. Direct a slow steady stream of irrigation solution toward the eardrum. 4. Assist the client to turn his or her head so that the ear to be irrigated is facing upward.

1 Before ear irrigation, the nurse should inspect the tympanic membrane to ensure that it is intact. The irrigating solution should be warmed to 98.6 ° F because a solution temperature that is not close to the client's body temperature will cause ear injury, nausea, and vertigo. The affected side should be down following the irrigation to assist in drainage of the fluid. When irrigating, a direct and slow steady stream of irrigation solution is directed toward the wall of the canal, not toward the eardrum. The client is positioned sitting, facing forward with the head in a natural position; if the ear is faced upward, the nurse would not be able to visualize the canal.

770. A client with a urinary tract infection is receiving ciprofloxacin by the intravenous (IV) route. The nurse appropriately administers the medication by performing which action? 1. Infusing slowly over 60 minutes. 2. Infusing in a light-protective bag. 3. Infusing only through a central line. 4. Infusing rapidly as a direct intravenous push medication.

1 Ciprofloxacin (Cipro) is prescribed for treatment of mild, moderate, severe, and complicated infections of the urinary tract, lower respiratory tract, and skin and skin structure. A single dose is administered slowly over 60 minutes to minimize discomfort and vein irritation. Other solutions infusing at the same site need to be temporarily discontinued while the ciprofloxacin is infusing.

681. The nurse teaches a client about the effects of diphenhydramine, which has been prescribed as a cough suppressant. The nurse determines that the client needs further instruction if the client makes which statement? 1. "I will take the medication on an empty stomach." 2. "I won't drink alcohol while taking this medication." 3. "I will use sugarless gum, candy, or oral rinses to decrease dryness in my mouth." 4. "I won't do activities that require mental alertness while taking this medication."

1 Diphenhydramine (Benadryl) has several uses, including as an antihistamine, antitussive, antidyskinetic, and sedativehypnotic. Instructions for use include taking with food or milk to decrease gastrointestinal upset and using oral rinses or sugarless gum or hard candy to minimize dry mouth. Because the medication causes drowsiness, the client should avoid use of alcohol or central nervous system depressants, operating a car, or engaging in other activities requiring mental awareness during use.

359. The nursing instructor asks a nursing student to describe the procedure for administering erythromycin ointment to the eyes of a newborn. Which student statement indicates that further teaching is needed? 1. "I will flush the eyes after instilling the ointment." 2. "I will clean the newborn's eyes before instilling ointment." 3. "I need to administer the eye ointment within 1 hour after delivery." 4. "I will instill the eye ointment into each of the newborn's conjunctival sacs."

1 Eye prophylaxis protects the newborn against Neisseria gonorrhoeae and Chlamydia trachomatis. The eyes are not flushed after instillation of the medication because the flush would wash away the administered medication. Options 2, 3, and 4 are correct statements regarding the procedure for administering eye medication to the newborn.

958. A client taking lithium carbonate reports vomiting, abdominal pain, diarrhea, blurred vision, tinnitus, and tremors. The lithium level is 2.5 mEq/L. This level is indicative of which finding? 1. Toxic 2. Normal 3. Slightly above normal 4. Excessively below normal

1 Maintenance serum levels of lithium are 0.6 to 1.2 mEq/L. Symptoms of toxicity begin to appear at levels of 1.5 to 2 mEq/L. Lithium toxicity requires immediate medical attention with lavage and possible peritoneal dialysis or hemodialysis

824. The client arrives at the emergency department complaining of back spasms. The client states, "I have been taking two to three aspirin every 4 hours for the last week, and it hasn't helped my back." Since aspirin intoxication is suspected, the nurse should assess the client for which manifestation? 1. Tinnitus 2. Diarrhea 3. Constipation 4. Photosensitivity

1 Mild intoxication with acetylsalicylic acid (aspirin) is called salicylism and is experienced commonly when the daily dosage is higher than 4 g. Tinnitus (ringing in the ears) is the most frequent effect noted with intoxication. Hyperventilation may occur because salicylate stimulates the respiratory center. Fever may result, because salicylate interferes with the metabolic pathways coupling oxygen consumption and heat production. Options 2, 3, and 4 are not associated specifically with toxicity.

363. An opioid analgesic is administered to a client in labor. The nurse assigned to care for the client ensures that which medication is readily available if respiratory depression occurs? 1. Naloxone 2. Morphine sulfate 3. Betamethasone 4. Meperidine hydrochloride

1 Opioid analgesics may be prescribed to relieve moderate to severe pain associated with labor. Opioid toxicity can occur and cause respiratory depression. Naloxone is an opioid antagonist, which reverses the effects of opioids and is given for respiratory depression. Morphine sulfate and meperidine hydrochloride are opioid analgesics. Betamethasone is a corticosteroid administered to enhance fetal lung maturity.

616. A client with DM visits a health care clinic. The client's DM previously had been well controlled with glyburide daily, but recently the fasting blood glucose level has been 180 to 200 mg/dl. Which medication if added to the client's regimen, may have contributed to the hyperglycemia? 1. Prednisone 2. Phenelzine 3. Atenolol 4. Allopurinol

1 Prednisone may decrease the effect of oral hypoglycemics, insulin, diuretics, and potassium supplements. Option 2, a monoamine oxidase inhibitor, and option 3, a beta-blocker, have their own intrinsic hypoglycemic activity. Option 4 decreases urinary excretion of sulfonylurea agents, causing increased levels of the oral agents, which can lead to hypoglycemia.

881. A client who is human immunodeficiency virus seropositive has been taking stavudine. The nurse should monitor which most closely while the client is taking this medication? 1. Gait 2. Appetite 3. Level of consciousness 4. Gastrointestinal function

1 Stavudine is an antiretroviral used to manage human immunodeficiency virus infection in clients who do not respond to or who cannot tolerate conventional therapy. The medication can cause peripheral neuropathy, and the nurse should monitor the client's gait closely and ask the client about paresthesia. Options 2, 3, and 4 are unrelated to this medication.

722. A client who is receiving digoxin daily has a serum potassium level of 3 mEq/L and is complaining of anorexia. The health care provider prescribes determination of the serum digoxin level to rule out digoxin toxicity. The nurse checks the results, knowing that which value is the therapeutic serum level (range) for digoxin? 1. 0.5 to 2 ng/mL 2. 1.2 to 2.8 ng/mL 3. 3.0 to 5.0 ng/mL 4. 3.5 to 5.5 ng/mL

1 Therapeutic levels for digoxin range from 0.5 to 2 ng/mL. The ranges in the remaining options are incorrect.

794. A client is prescribed an eyedrop and an eye ointment for the right eye. How should the nurse best administer the medications? 1. Administer the eyedrop first, followed by the eye ointment. 2. Administer the eye ointment first, followed by the eyedrop. 3. Administer the eyedrop, wait 15 minutes, and administer the eye ointment. 4. Administer the eye ointment, wait 15 minutes, and administer the eyedrop.

1 When an eyedrop and an eye ointment are scheduled to be administered at the same time, the eyedrop is administered first. The instillation of two medications is separated by 3 to 5 minutes.

600. The nurse is teaching a client how to mix regular insulin and NPH insulin in the same syringe. Which action, if performed by the client, indicates the need for further teaching? 1. Withdraws the NPH insulin first 2. Withdraws the regular insulin first 3. Injects air into NPH insulin vial first 4. Injects an amount of air equal to the desired dose of insulin into each vial

1 When preparing a mixture of short-acting insulin suc as regular insulin with another insulin preparation, the short-acting insulin is drawn into the syringe first. This sequence will avoid contaminating the vial of short-acting insulin with insulin of another type. Options 2, 3 and 4 identify correct actions for preparing NPH and short-acting insulin

856. In monitoring a client's response to disease-modifying antirheumatic drugs, which assessment findings would the nurse consider acceptable responses? Select all that apply. 1. Symptom control during periods of emotional stress 2. Normal white blood cell, platelet, and neutrophil counts 3. Radiological findings that show no progression of joint degeneration 4. An increased range of motion in the affected joints 3 months into therapy 5. Inflammation and irritation at the injection site 3 days after the injection is given 6. A low-grade temperature on rising in the morning that remains throughout the day

1, 2, 3, 4 Because emotional stress frequently exacerbates the symptoms of rheumatoid arthritis, the absence of symptoms is a positive finding. DMARDs are given to slow the progression of joint degeneration. In addition, an improvement in the range of motion after 3 months of therapy with normal blood work is a positive finding. Temperature elevation and inflammation and irritation at the medication injection site could indicate signs of infection.

606. The home health care nurse is visiting a client who was recently diagnosed with type 2 DM. The client is prescribed repaglinide and metformin and asks the nurse to explain these medications. The nurse should provide which instructions to the client? Select all that apply. 1. Diarrhea may occur secondary to the metformin 2. The repaglinide is not taken if a meal is skipped 3. The repaglinide is taken 30 minutes before eating 4. A simple sugar food item is carried and used to treat mild hypoglycemia episodes 5. Metformin increases hepatic glucose production to prevent hypoglycemia associated with repaglinide 6. Muscle pain is an expected effect of metformin and may be treated with acetaminophen

1, 2, 3, 4 Repaglinide, a rapid-acting oral hypoglycemic agent that stimulates pancreatic insulin secretion, should be taken before meals (approximately 30 minutes before meals) and should be withheld if the client does not eat. Hypoglycemia is a side effect of repaglinide and the client should always be prepared by carrying a simple sugar with her or him at all times. Metformin is an oral hypoglycemic given in combination with repaglinide and works by decreasing hepatic glucose production. A common side effect of metformin is diarrhea. Muscle pain may occur as an adverse effect from metformin but it might signify a more serious condition that warrants HCP notification, not the use of acetaminophen.

791. The nurse is preparing to administer eyedrops. Which interventions should the nurse take to administer the drops? Select all that apply. 1. Wash hands. 2. Put gloves on. 3. Place the drop in the conjunctival sac. 4. Pull the lower lid down against the cheek bone. 5. Instruct the client to squeeze the eyes shut after instilling the eyedrop. 6. Instruct the client to tilt the head forward, open the eyes, and look down.

1, 2, 3, 4 To administer eye medications, the nurse should wash hands and put gloves on. The client is instructed to tilt the head backward, open the eyes, and look up. The nurse pulls the lower lid down against the cheekbone and holds the bottle like a pencil with the tip downward. Holding the bottle, the nurse gently rests the wrist of the hand on the client's cheek and squeezes the bottle gently to allow the drop to fall into the conjunctival sac. The client is instructed to close the eyes gently and not to squeeze the eyes shut to prevent the loss of medication.

566. The nurse is monitoring the intravenous infusion of an antineoplastic medication. During the infusion, the client complains of pain at the insertion site. On inspection of the site, the nurse notes redness and swelling and that the infusion of the medication has slowed in rate. The nurse suspects extravasation and should take which actions? Select all that apply. 1, Stop the infusion 2, Notify the HCP 3. Prepare to apply ice or heat to the site 4. Restart the IV at a distal part of the same vein 5. Prepare to administer a prescribed antidote into the site 6. Increase the flow rate of the solution to flush the skin and subcutaneous tissue

1, 2, 3, 5 Redness and swelling and a slowed infusion idicate signs of extravasation. If the nurse suspects extravasation during the intravenous administration of an antineoplastic medication, the infusion is stopped and the HCP is notified. Ice or heat may be prescribed for application to the site and an antidote may be prescribed to be administered into the site. Increasing the flow rate can increase damage to the tissues. Restarting an IV in the same vein can increase damage to the site and veins.

691. Rifabutin is prescribed for a client with active Mycobacterium avium complex (MAC) disease and tuberculosis. For which side/adverse effects of the medication should the nurse monitor? Select all that apply. 1. Signs of hepatitis 2. Flulike syndrome 3. Low neutrophil count 4. Vitamin B6 deficiency 5. Ocular pain or blurred vision 6. Tingling and numbness of the fingers

1, 2, 3, 5 Rifabutin (Mycobutin) may be prescribed for a client with active Mycobacterium avium complex (MAC) disease and tuberculosis. It inhibits mycobacterial DNA-dependent RNA polymerase and suppresses protein synthesis. Side/adverse effects include rash, gastrointestinal disturbances, neutropenia (low neutrophil count), red-orange-colored body secretions, uveitis (blurred vision and eye pain), myositis, arthralgia, hepatitis, chest pain with dyspnea, and flulike syndrome. Vitamin B6 deficiency and numbness and tingling in the extremities are associated with the use of isoniazid. Ethambutol (Myambutol) causes peripheral neuritis.

605. A client is taking Humulin NPH insulin and regular insulin every morning. The nurse should provide which instructions to the client? Select all that apply. 1. Hypoglycemia may be experienced before dinnertime 2. The insulin dose should be decreased if illness occurs 3. The insulin should be administered at room temperature 4. The insulin vial needs to be shaken vigorously to break up the precipitates 5. The NPH insulin should be drawn into the syringe first, then the regular insulin

1, 3 Humulin NPH is an intermediate-acting insulin. The onset of action is 1.5 hours, it peaks in 4 to 12 hours, and its duration of action is 24 hours. Regular insulin is a short-acting insulin. Depending on the type, the onset of action is 0.5 hours, it peaks in 2 to 5.5 hours, and its duration is 5 to 8 hours. Hypoglycemic reactions most likely occur during peak time. Insulin should be at room temperature when administered. Clients may need their insulin dosages increased during times of illness. Insulin vials should never be shaken vigorously. Regular insulin is always drawn up before NPH.

608. A client with hyperthyroidism has been given methimazole. Which nursing considerations are associated with this medication? Select all that apply. 1. Administer methimazole with food 2. Place the client on a low-calorie, low-protein diet 3. Assess the client for unexplained bruising or bleeding 4. Instruct the client to report side/adverse effects such as sore throat, fever, or headaches 5. Use special radioactive precautions when handling the client's urine for the first 24 hours following initial administration

1, 3, 4 Common side effects of methimazole include nausea, vomiting, and diarrhea. To address these side effects, this medication should be taken with food. Because of the increase in metabolism that occurs in hyperthyroidism, the client should consume a high calorie diet. Antithyroid medications can cause agranulocytosis and the HCP should be notified immediately. Methimazole is not radioactive and should not be stopped abruptly, due to the risk of thyroid storm.

173.) A nurse reviews the medication history of a client admitted to the hospital and notes that the client is taking leflunomide (Arava). During data collection, the nurse asks which question to determine medication effectiveness? 1. "Do you have any joint pain?" 2. "Are you having any diarrhea?" 3. "Do you have frequent headaches?" 4. "Are you experiencing heartburn?"

1. "Do you have any joint pain?" Rationale: Leflunomide is an immunosuppressive agent and has an anti-inflammatory action. The medication provides symptomatic relief of rheumatoid arthritis. Diarrhea can occur as a side effect of the medication. The other options are unrelated to medication effectiveness.

127.) The nurse provides medication instructions to an older hypertensive client who is taking 20 mg of lisinopril (Prinivil, Zestril) orally daily. The nurse evaluates the need for further teaching when the client states which of the following? 1. "I can skip a dose once a week." 2. "I need to change my position slowly." 3. "I take the pill after breakfast each day." 4. "If I get a bad headache, I should call my doctor immediately."

1. "I can skip a dose once a week." Rationale: Lisinopril is an antihypertensive angiotensin-converting enzyme (ACE) inhibitor. The usual dosage range is 20 to 40 mg per day. Adverse effects include headache, dizziness, fatigue, orthostatic hypotension, tachycardia, and angioedema. Specific client teaching points include taking one pill a day, not stopping the medication without consulting the health care provider (HCP), and monitoring for side effects and adverse reactions. The client should notify the HCP if side effects occur.

31.) A community health nurse visits a client at home. Prednisone 10 mg orally daily has been prescribed for the client and the nurse reinforces teaching for the client about the medication. Which statement, if made by the client, indicates that further teaching is necessary? 1. "I can take aspirin or my antihistamine if I need it." 2. "I need to take the medication every day at the same time." 3. "I need to avoid coffee, tea, cola, and chocolate in my diet." 4. "If I gain more than 5 pounds a week, I will call my doctor."

1. "I can take aspirin or my antihistamine if I need it." Rationale: Aspirin and other over-the-counter medications should not be taken unless the client consults with the health care provider (HCP). The client needs to take the medication at the same time every day and should be instructed not to stop the medication. A slight weight gain as a result of an improved appetite is expected, but after the dosage is stabilized, a weight gain of 5 lb or more weekly should be reported to the HCP. Caffeine-containing foods and fluids need to be avoided because they may contribute to steroid-ulcer development.

warfarin (coumadin) {admin once daily, avoid NSAIDs & aspirin}, enoxaparin (lovenox)

inhibit clotting factors (warfarin = factors VII, IX, X) TX: evolving stroke, pulmonary embolism, massive deep vein thrombosis, cardiac cath, MI, DIC S/S: hemorrhage, heparin induced thrombocytopenia, toxicity/overdose

688. The nurse is caring for a client with a diagnosis of influenza who first began to experience symptoms yesterday. Antiviral therapy is prescribed and the nurse provides instructions to the client about the therapy. Which statement by the client indicates an understanding of the instructions? 1. "I must take the medication exactly as prescribed." 2. "Once I start the medication, I will no longer be contagious." 3. "I will not get any colds or infections while taking this medication." 4. "This medication has minimal side effects and I can return to normal activities."

1. "I must take the medication exactly as prescribed."

179.) A nurse provides medication instructions to a client who had a kidney transplant about therapy with cyclosporine (Sandimmune). Which statement by the client indicates a need for further instruction? 1. "I need to obtain a yearly influenza vaccine." 2. "I need to have dental checkups every 3 months." 3. "I need to self-monitor my blood pressure at home." 4. "I need to call the health care provider (HCP) if my urine volume decreases or my urine becomes cloudy."

1. "I need to obtain a yearly influenza vaccine." Rationale: Cyclosporine is an immunosuppressant medication. Because of the medication's effects, the client should not receive any vaccinations without first consulting the HCP. The client should report decreased urine output or cloudy urine, which could indicate kidney rejection or infection, respectively. The client must be able to self-monitor blood pressure to check for the side effect of hypertension. The client needs meticulous oral care and dental cleaning every 3 months to help prevent gingival hyperplasia.

77.) Phenytoin (Dilantin), 100 mg orally three times daily, has been prescribed for a client for seizure control. The nurse reinforces instructions regarding the medication to the client. Which statement by the client indicates an understanding of the instructions? 1. "I will use a soft toothbrush to brush my teeth." 2. "It's all right to break the capsules to make it easier for me to swallow them." 3. "If I forget to take my medication, I can wait until the next dose and eliminate that dose." 4. "If my throat becomes sore, it's a normal effect of the medication and it's nothing to be concerned about."

1. "I will use a soft toothbrush to brush my teeth." Rationale: Phenytoin (Dilantin) is an anticonvulsant. Gingival hyperplasia, bleeding, swelling, and tenderness of the gums can occur with the use of this medication. The client needs to be taught good oral hygiene, gum massage, and the need for regular dentist visits. The client should not skip medication doses, because this could precipitate a seizure. Capsules should not be chewed or broken and they must be swallowed. The client needs to be instructed to report a sore throat, fever, glandular swelling, or any skin reaction, because this indicates hematological toxicity.

169.) Insulin glargine (Lantus) is prescribed for a client with diabetes mellitus. The nurse tells the client that it is best to take the insulin: 1. 1 hour after each meal 2. Once daily, at the same time each day 3. 15 minutes before breakfast, lunch, and dinner 4. Before each meal, on the basis of the blood glucose level

2. Once daily, at the same time each day Rationale: Insulin glargine is a long-acting recombinant DNA human insulin used to treat type 1 and type 2 diabetes mellitus. It has a 24-hour duration of action and is administered once a day, at the same time each day.

220.) A adult client with muscle spasms is taking an oral maintenance dose of baclofen (Lioresal). The nurse reviews the medication record, expecting that which dose should be prescribed? 1. 15 mg four times a day 2. 25 mg four times a day 3. 30 mg four times a day 4. 40 mg four times a day

1. 15 mg four times a day Rationale: Baclofen is dispensed in 10- and 20-mg tablets for oral use. Dosages are low initially and then gradually increased. Maintenance doses range from 15 to 20 mg administered three or four times a day.

136.) A nurse performs an admission assessment on a client who visits a health care clinic for the first time. The client tells the nurse that propylthiouracil (PTU) is taken daily. The nurse continues to collect data from the client, suspecting that the client has a history of: 1. Myxedema 2. Graves' disease 3. Addison's disease 4. Cushing's syndrome

2. Graves' disease Rationale: PTU inhibits thyroid hormone synthesis and is used to treat hyperthyroidism, or Graves' disease. Myxedema indicates hypothyroidism. Cushing's syndrome and Addison's disease are disorders related to adrenal function.

682. A client has been taking isoniazid (INH) for 1½ months. The client complains to a nurse about numbness, paresthesias, and tingling in the extremities. The nurse interprets that the client is experiencing: 1. Hypercalcemia 2. Peripheral neuritis 3. Small blood vessel spasm 4. Impaired peripheral circulation

2. Peripheral neuritis

145.) A nurse has a prescription to give a client albuterol (Proventil HFA) (two puffs) and beclomethasone dipropionate (Qvar) (nasal inhalation, two puffs), by metered-dose inhaler. The nurse administers the medication by giving the: 1. Albuterol first and then the beclomethasone dipropionate 2. Beclomethasone dipropionate first and then the albuterol 3. Alternating a single puff of each, beginning with the albuterol 4. Alternating a single puff of each, beginning with the beclomethasone dipropionate

1. Albuterol first and then the beclomethasone dipropionate Rationale: Albuterol is a bronchodilator. Beclomethasone dipropionate is a glucocorticoid. Bronchodilators are always administered before glucocorticoids when both are to be given on the same time schedule. This allows for widening of the air passages by the bronchodilator, which then makes the glucocorticoid more effective.

142.) A health care provider has written a prescription for ranitidine (Zantac), once daily. The nurse should schedule the medication for which of the following times? 1. At bedtime 2. After lunch 3. With supper 4. Before breakfast

1. At bedtime Rationale: A single daily dose of ranitidine is usually scheduled to be given at bedtime. This allows for a prolonged effect, and the greatest protection of the gastric mucosa. **recall that ranitidine suppresses secretions of gastric acids**

100.) Saquinavir (Invirase) is prescribed for the client who is human immunodeficiency virus seropositive. The nurse reinforces medication instructions and tells the client to: 1. Avoid sun exposure. 2. Eat low-calorie foods. 3. Eat foods that are low in fat. 4. Take the medication on an empty stomach.

1. Avoid sun exposure. Rationale: Saquinavir (Invirase) is an antiretroviral (protease inhibitor) used with other antiretroviral medications to manage human immunodeficiency virus infection. Saquinavir is administered with meals and is best absorbed if the client consumes high-calorie, high-fat meals. Saquinavir can cause photosensitivity, and the nurse should instruct the client to avoid sun exposure.

202.) A nurse is collecting data from a client about medications being taken, and the client tells the nurse that he is taking herbal supplements for the treatment of varicose veins. The nurse understands that the client is most likely taking which of the following? 1. Bilberry 2. Ginseng 3. Feverfew 4. Evening primrose

1. Bilberry Rationale: Bilberry is an herbal supplement that has been used to treat varicose veins. This supplement has also been used to treat cataracts, retinopathy, diabetes mellitus, and peripheral vascular disease. Ginseng has been used to improve memory performance and decrease blood glucose levels in type 2 diabetes mellitus. Feverfew is used to prevent migraine headaches and to treat rheumatoid arthritis. Evening primrose is used to treat eczema and skin irritation.

156.) A nurse is reviewing the laboratory results for a client receiving tacrolimus (Prograf). Which laboratory result would indicate to the nurse that the client is experiencing an adverse effect of the medication? 1. Blood glucose of 200 mg/dL 2. Potassium level of 3.8 mEq/L 3. Platelet count of 300,000 cells/mm3 4. White blood cell count of 6000 cells/mm3

1. Blood glucose of 200 mg/dL Rationale: A blood glucose level of 200 mg/dL is elevated above the normal range of 70 to 110 mg/dL and suggests an adverse effect. Other adverse effects include neurotoxicity evidenced by headache, tremor, insomnia; gastrointestinal (GI) effects such as diarrhea, nausea, and vomiting; hypertension; and hyperkalemia.

32.) Desmopressin acetate (DDAVP) is prescribed for the treatment of diabetes insipidus. The nurse monitors the client after medication administration for which therapeutic response? 1. Decreased urinary output 2. Decreased blood pressure 3. Decreased peripheral edema 4. Decreased blood glucose level

1. Decreased urinary output Rationale: Desmopressin promotes renal conservation of water. The hormone carries out this action by acting on the collecting ducts of the kidney to increase their permeability to water, which results in increased water reabsorption. The therapeutic effect of this medication would be manifested by a decreased urine output. Options 2, 3, and 4 are unrelated to the effects of this medication.

105.) A nurse is collecting data from a client and the client's spouse reports that the client is taking donepezil hydrochloride (Aricept). Which disorder would the nurse suspect that this client may have based on the use of this medication? 1. Dementia 2. Schizophrenia 3. Seizure disorder 4. Obsessive-compulsive disorder

1. Dementia Rationale: Donepezil hydrochloride is a cholinergic agent used in the treatment of mild to moderate dementia of the Alzheimer type. It enhances cholinergic functions by increasing the concentration of acetylcholine. It slows the progression of Alzheimer's disease. Options 2, 3, and 4 are incorrect.

33.) The home health care nurse is visiting a client who was recently diagnosed with type 2 diabetes mellitus. The client is prescribed repaglinide (Prandin) and metformin (Glucophage) and asks the nurse to explain these medications. The nurse should reinforce which instructions to the client? Select all that apply. 1. Diarrhea can occur secondary to the metformin. 2. The repaglinide is not taken if a meal is skipped. 3. The repaglinide is taken 30 minutes before eating. 4. Candy or another simple sugar is carried and used to treat mild hypoglycemia episodes. 5. Metformin increases hepatic glucose production to prevent hypoglycemia associated with repaglinide. 6. Muscle pain is an expected side effect of metformin and may be treated with acetaminophen (Tylenol).

1. Diarrhea can occur secondary to the metformin. 2. The repaglinide is not taken if a meal is skipped. 3. The repaglinide is taken 30 minutes before eating. 4. Candy or another simple sugar is carried and used to treat mild hypoglycemia episodes. Rationale: Repaglinide is a rapid-acting oral hypoglycemic agent that stimulates pancreatic insulin secretion that should be taken before meals, and that should be withheld if the client does not eat. Hypoglycemia is a side effect of repaglinide and the client should always be prepared by carrying a simple sugar with her or him at all times. Metformin is an oral hypoglycemic given in combination with repaglinide and works by decreasing hepatic glucose production. A common side effect of metformin is diarrhea. Muscle pain may occur as an adverse effect from metformin but it might signify a more serious condition that warrants health care provider notification, not the use of acetaminophen.

193.) Sodium hypochlorite (Dakin's solution) is prescribed for a client with a leg wound containing purulent drainage. The nurse is assisting in developing a plan of care for the client and includes which of the following in the plan? 1. Ensure that the solution is freshly prepared before use. 2. Soak a sterile dressing with solution and pack into the wound. 3. Allow the solution to remain in the wound following irrigation. 4. Apply the solution to the wound and on normal skin tissue surrounding the wound.

1. Ensure that the solution is freshly prepared before use. Rationale: Dakin solution is a chloride solution that is used for irrigating and cleaning necrotic or purulent wounds. It can be used for packing necrotic wounds. It cannot be used to pack purulent wounds because the solution is inactivated by copious pus. It should not come into contact with healing or normal tissue, and it should be rinsed off immediately if used for irrigation. Solutions are unstable and the nurse must ensure that the solution has been prepared fresh before use. **Eliminate options 2 and 3 first because they are comparable or alike. It makes sense to ensure that the solution is freshly prepared; therefore, select option 1**

957. A hospitalized client is started on phenelzine (Nardil) for the treatment of depression. A nurse instructs the client to avoid consuming which foods while taking this medication? Select all that apply. 1. Figs 2. Yogurt 3. Crackers 4. Aged cheese 5. Tossed salad 6. Oatmeal cookies

1. Figs 2. Yogurt 4. Aged cheese

112.) A hospitalized client is started on phenelzine sulfate (Nardil) for the treatment of depression. The nurse instructs the client to avoid consuming which foods while taking this medication? Select all that apply. 1. Figs 2. Yogurt 3. Crackers 4. Aged cheese 5 Tossed salad 6. Oatmeal cookies

1. Figs 2. Yogurt 4. Aged cheese Rationale: Phenelzine sulfate (Nardil) is a monoamine oxidase inhibitor(MAOI). The client should avoid taking in foods that are high in tyramine. Use of these foods could trigger a potentially fatal hypertensive crisis. Some foods to avoid include yogurt, aged cheeses, smoked or processed meats, red wines, and fruits such as avocados, raisins, and figs.

93.) The client who is human immunodeficiency virus seropositive has been taking stavudine (d4t, Zerit). The nurse monitors which of the following most closely while the client is taking this medication? 1. Gait 2. Appetite 3. Level of consciousness 4. Hemoglobin and hematocrit blood levels

1. Gait Rationale: Stavudine (d4t, Zerit) is an antiretroviral used to manage human immunodeficiency virus infection in clients who do not respond to or who cannot tolerate conventional therapy. The medication can cause peripheral neuropathy, and the nurse should monitor the client's gait closely and ask the client about paresthesia. Options 2, 3, and 4 are unrelated to the use of the medication.

91.) Cyclobenzaprine (Flexeril) is prescribed for a client to treat muscle spasms, and the nurse is reviewing the client's record. Which of the following disorders, if noted in the client's record, would indicate a need to contact the health care provider regarding the administration of this medication? 1. Glaucoma 2. Emphysema 3. Hyperthyroidism 4. Diabetes mellitus

1. Glaucoma Rationale: Because this medication has anticholinergic effects, it should be used with caution in clients with a history of urinary retention, angle-closure glaucoma, and increased intraocular pressure. Cyclobenzaprine hydrochloride should be used only for short-term 2- to 3-week therapy.

158.) A client with chronic renal failure is receiving epoetin alfa (Epogen, Procrit). Which laboratory result would indicate a therapeutic effect of the medication? 1. Hematocrit of 32% 2. Platelet count of 400,000 cells/mm3 3. White blood cell count of 6000 cells/mm3 4. Blood urea nitrogen (BUN) level of 15 mg/dL

1. Hematocrit of 32% Rationale: Epoetin alfa is used to reverse anemia associated with chronic renal failure. A therapeutic effect is seen when the hematocrit is between 30% and 33%. The laboratory tests noted in the other options are unrelated to the use of this medication.

43.) A histamine (H2)-receptor antagonist will be prescribed for a client. The nurse understands that which medications are H2-receptor antagonists? Select all that apply. 1. Nizatidine (Axid) 2. Ranitidine (Zantac) 3. Famotidine (Pepcid) 4. Cimetidine (Tagamet) 5. Esomeprazole (Nexium) 6. Lansoprazole (Prevacid)

1. Nizatidine (Axid) 2. Ranitidine (Zantac) 3. Famotidine (Pepcid) 4. Cimetidine (Tagamet) Rationale: H2-receptor antagonists suppress secretion of gastric acid, alleviate symptoms of heartburn, and assist in preventing complications of peptic ulcer disease. These medications also suppress gastric acid secretions and are used in active ulcer disease, erosive esophagitis, and pathological hypersecretory conditions. The other medications listed are proton pump inhibitors. H2-receptor antagonists medication names end with -dine. Proton pump inhibitors medication names end with -zole.

109.) A client taking buspirone (BuSpar) for 1 month returns to the clinic for a follow-up visit. Which of the following would indicate medication effectiveness? 1. No rapid heartbeats or anxiety 2. No paranoid thought processes 3. No thought broadcasting or delusions 4. No reports of alcohol withdrawal symptoms

1. No rapid heartbeats or anxiety Rationale: Buspirone hydrochloride is not recommended for the treatment of drug or alcohol withdrawal, paranoid thought disorders, or schizophrenia (thought broadcasting or delusions). Buspirone hydrochloride is most often indicated for the treatment of anxiety and aggression.

12.) A nurse is caring for a client who is receiving an intravenous (IV) infusion of an antineoplastic medication. During the infusion, the client complains of pain at the insertion site. During an inspection of the site, the nurse notes redness and swelling and that the rate of infusion of the medication has slowed. The nurse should take which appropriate action? 1. Notify the registered nurse. 2. Administer pain medication to reduce the discomfort. 3. Apply ice and maintain the infusion rate, as prescribed. 4. Elevate the extremity of the IV site, and slow the infusion.

1. Notify the registered nurse. Rationale: When antineoplastic medications (Chemotheraputic Agents) are administered via IV, great care must be taken to prevent the medication from escaping into the tissues surrounding the injection site, because pain, tissue damage, and necrosis can result. The nurse monitors for signs of extravasation, such as redness or swelling at the insertion site and a decreased infusion rate. If extravasation occurs, the registered nurse needs to be notified; he or she will then contact the health care provider.

235.) A tricyclic antidepressant is administered to a client daily. The nurse plans to monitor for the common side effects of the medication and includes which of the following in the plan of care? 1. Offer hard candy or gum periodically. 2. Offer a nutritious snack between meals. 3. Monitor the blood pressure every 2 hours. 4. Review the white blood cell (WBC) count results daily.

1. Offer hard candy or gum periodically. Rationale: Dry mouth is a common side effect of tricyclic antidepressants. Frequent mouth rinsing with water, sucking on hard candy, and chewing gum will alleviate this common side effect. It is not necessary to monitor the blood pressure every 2 hours. In addition, it is not necessary to check the WBC daily. Weight gain is a common side effect and frequent snacks will aggravate this problem.

18.) The nurse is reviewing the history and physical examination of a client who will be receiving asparaginase (Elspar), an antineoplastic agent. The nurse consults with the registered nurse regarding the administration of the medication if which of the following is documented in the client's history? 1. Pancreatitis 2. Diabetes mellitus 3. Myocardial infarction 4. Chronic obstructive pulmonary disease

1. Pancreatitis Rationale: Asparaginase (Elspar) is contraindicated if hypersensitivity exists, in pancreatitis, or if the client has a history of pancreatitis. The medication impairs pancreatic function and pancreatic function tests should be performed before therapy begins and when a week or more has elapsed between administration of the doses. The client needs to be monitored for signs of pancreatitis, which include nausea, vomiting, and abdominal pain. The conditions noted in options 2, 3, and 4 are not contraindicated with this medication.

645. A client has a PRN prescription for ondansetron. For which condition should the nurse administer this medication to the postoperative client? 1. Paralytic ileus 2. Incisional pain 3. Urinary retention 4. Nausea and vomiting

4 Ondansetron (Zofran) is an antiemetic used to treat postoperative nausea and vomiting, as well as nausea and vomiting associated with chemotherapy. The other options are incorrect.

171.) A nurse is preparing to administer furosemide (Lasix) to a client with a diagnosis of heart failure. The most important laboratory test result for the nurse to check before administering this medication is: 1. Potassium level 2. Creatinine level 3. Cholesterol level 4. Blood urea nitrogen

1. Potassium level Rationale: Furosemide is a loop diuretic. The medication causes a decrease in the client's electrolytes, especially potassium, sodium, and chloride. Administering furosemide to a client with low electrolyte levels could precipitate ventricular dysrhythmias. Options 2 and 4 reflect renal function. The cholesterol level is unrelated to the administration of this medication.

30.) A client with diabetes mellitus visits a health care clinic. The client's diabetes mellitus previously had been well controlled with glyburide (DiaBeta) daily, but recently the fasting blood glucose level has been 180 to 200 mg/dL. Which medication, if added to the client's regimen, may have contributed to the hyperglycemia? 1. Prednisone 2. Phenelzine (Nardil) 3. Atenolol (Tenormin) 4. Allopurinol (Zyloprim)

1. Prednisone Rationale: Prednisone may decrease the effect of oral hypoglycemics, insulin, diuretics, and potassium supplements. Option 2, a monoamine oxidase inhibitor, and option 3, a β-blocker, have their own intrinsic hypoglycemic activity. Option 4 decreases urinary excretion of sulfonylurea agents, causing increased levels of the oral agents, which can lead to hypoglycemia.

152.) Intravenous heparin therapy is prescribed for a client. While implementing this prescription, a nurse ensures that which of the following medications is available on the nursing unit? 1. Protamine sulfate 2. Potassium chloride 3. Phytonadione (vitamin K ) 4. Aminocaproic acid (Amicar)

1. Protamine sulfate Rationale: The antidote to heparin is protamine sulfate; it should be readily available for use if excessive bleeding or hemorrhage occurs. Potassium chloride is administered for a potassium deficit. Vitamin K is an antidote for warfarin sodium. Aminocaproic acid is the antidote for thrombolytic therapy.

691. Rifabutin (Mycobutin) is prescribed for a client with active Mycobacterium avium complex (MAC) disease and tuberculosis. For which of the following side effects of the medication should the nurse monitor? Select all that apply. 1. Signs of hepatitis 2. Flu-like syndrome 3. Low neutrophil count 4. Vitamin B6 deficiency 5. Ocular pain or blurred vision 6. Tingling and numbness of the fingers

1. Signs of hepatitis 2. Flu-like syndrome 3. Low neutrophil count 5. Ocular pain or blurred vision

102.) A client with human immunodeficiency virus is taking nevirapine (Viramune). The nurse should monitor for which adverse effects of the medication? Select all that apply. 1. Rash 2. Hepatotoxicity 3. Hyperglycemia 4. Peripheral neuropathy 5. Reduced bone mineral density

1. Rash 2. Hepatotoxicity Rationale: Nevirapine (Viramune) is a non-nucleoside reverse transcriptase inhibitors (NRTI) that is used to treat HIV infection. It is used in combination with other antiretroviral medications to treat HIV. Adverse effects include rash, Stevens-Johnson syndrome, hepatitis, and increased transaminase levels. Hyperglycemia, peripheral neuropathy, and reduced bone density are not adverse effects of this medication.

72.) Cinoxacin (Cinobac), a urinary antiseptic, is prescribed for the client. The nurse reviews the client's medical record and should contact the health care provider (HCP) regarding which documented finding to verify the prescription? Refer to chart. 1. Renal insufficiency 2. Chest x-ray: normal 3. Blood glucose, 102 mg/dL 4. Folic acid (vitamin B6) 0.5 mg, orally daily

1. Renal insufficiency Rationale: Cinoxacin should be administered with caution in clients with renal impairment. The dosage should be reduced, and failure to do so could result in accumulation of cinoxacin to toxic levels. Therefore the nurse would verify the prescription if the client had a documented history of renal insufficiency. The laboratory and diagnostic test results are normal findings. Folic acid (vitamin B6) may be prescribed for a client with renal insufficiency to prevent anemia.

90.) A nurse is reviewing the record of a client who has been prescribed baclofen (Lioresal). Which of the following disorders, if noted in the client's history, would alert the nurse to contact the health care provider? 1. Seizure disorders 2. Hyperthyroidism 3. Diabetes mellitus 4. Coronary artery disease

1. Seizure disorders Rationale: Clients with seizure disorders may have a lowered seizure threshold when baclofen is administered. Concurrent therapy may require an increase in the anticonvulsive medication. The disorders in options 2, 3, and 4 are not a concern when the client is taking baclofen.

53.) Rifabutin (Mycobutin) is prescribed for a client with active Mycobacterium avium complex (MAC) disease and tuberculosis. The nurse monitors for which side effects of the medication? Select all that apply. 1. Signs of hepatitis 2. Flu-like syndrome 3. Low neutrophil count 4. Vitamin B6 deficiency 5. Ocular pain or blurred vision 6. Tingling and numbness of the fingers

1. Signs of hepatitis 2. Flu-like syndrome 3. Low neutrophil count 5. Ocular pain or blurred vision Rationale: Rifabutin (Mycobutin) may be prescribed for a client with active MAC disease and tuberculosis. It inhibits mycobacterial DNA-dependent RNA polymerase and suppresses protein synthesis. Side effects include rash, gastrointestinal disturbances, neutropenia (low neutrophil count), red-orange body secretions, uveitis (blurred vision and eye pain), myositis, arthralgia, hepatitis, chest pain with dyspnea, and flu-like syndrome. Vitamin B6 deficiency and numbness and tingling in the extremities are associated with the use of isoniazid (INH). Ethambutol (Myambutol) also causes peripheral neuritis.

110.) A client taking lithium carbonate (Lithobid) reports vomiting, abdominal pain, diarrhea, blurred vision, tinnitus, and tremors. The lithium level is checked as a part of the routine follow-up and the level is 3.0 mEq/L. The nurse knows that this level is: 1. Toxic 2. Normal 3. Slightly above normal 4. Excessively below normal

1. Toxic Rationale: The therapeutic serum level of lithium is 0.6 to 1.2 mEq/L. A level of 3 mEq/L indicates toxicity.

92.) In monitoring a client's response to disease-modifying antirheumatic drugs (DMARDs), which findings would the nurse interpret as acceptable responses? Select all that apply. 1. Symptom control during periods of emotional stress 2. Normal white blood cell counts, platelet, and neutrophil counts 3. Radiological findings that show nonprogression of joint degeneration 4. An increased range of motion in the affected joints 3 months into therapy 5. Inflammation and irritation at the injection site 3 days after injection is given 6. A low-grade temperature upon rising in the morning that remains throughout the day

1. Symptom control during periods of emotional stress 2. Normal white blood cell counts, platelet, and neutrophil counts 3. Radiological findings that show nonprogression of joint degeneration 4. An increased range of motion in the affected joints 3 months into therapy Rationale: Because emotional stress frequently exacerbates the symptoms of rheumatoid arthritis, the absence of symptoms is a positive finding. DMARDs are given to slow progression of joint degeneration. In addition, the improvement in the range of motion after 3 months of therapy with normal blood work is a positive finding. Temperature elevation and inflammation and irritation at the medication injection site could indicate signs of infection.

149.) A client taking fexofenadine (Allegra) is scheduled for allergy skin testing and tells the nurse in the health care provider's office that a dose was taken this morning. The nurse determines that: 1. The client should reschedule the appointment. 2. A lower dose of allergen will need to be injected. 3. A higher dose of allergen will need to be injected. 4. The client should have the skin test read a day later than usual.

1. The client should reschedule the appointment. Rationale: Fexofenadine is an antihistamine, which provides relief of symptoms caused by allergy. Antihistamines should be discontinued for at least 3 days (72 hours) before allergy skin testing to avoid false-negative readings. This client should have the appointment rescheduled for 3 days after discontinuing the medication.

21.) A nurse is assisting with caring for a client with cancer who is receiving cisplatin. Select the adverse effects that the nurse monitors for that are associated with this medication. Select all that apply. 1. Tinnitus 2. Ototoxicity 3. Hyperkalemia 4. Hypercalcemia 5. Nephrotoxicity 6. Hypomagnesemia

1. Tinnitus 2. Ototoxicity 5. Nephrotoxicity 6. Hypomagnesemia Rationale: Cisplatin is an alkylating medication. Alkylating medications are cell cycle phase-nonspecific medications that affect the synthesis of DNA by causing the cross-linking of DNA to inhibit cell reproduction. Cisplatin may cause ototoxicity, tinnitus, hypokalemia, hypocalcemia, hypomagnesemia, and nephrotoxicity. Amifostine (Ethyol) may be administered before cisplatin to reduce the potential for renal toxicity.

213.) A client is admitted to the hospital with complaints of back spasms. The client states, "I have been taking two or three aspirin every 4 hours for the past week and it hasn't helped my back." Aspirin intoxication is suspected. Which of the following complaints would indicate aspirin intoxication? 1. Tinnitus 2. Constipation 3. Photosensitivity 4. Abdominal cramps

1. Tinnitus Rationale: Mild intoxication with acetylsalicylic acid (aspirin) is called salicylism and is commonly experienced when the daily dosage is higher than 4 g. Tinnitus (ringing in the ears) is the most frequently occurring effect noted with intoxication. Hyperventilation may occur because salicylate stimulates the respiratory center. Fever may result because salicylate interferes with the metabolic pathways involved with oxygen consumption and heat production. Options 2, 3, and 4 are incorrect.

3.) Salicylic acid is prescribed for a client with a diagnosis of psoriasis. The nurse monitors the client, knowing that which of the following would indicate the presence of systemic toxicity from this medication? 1. Tinnitus 2. Diarrhea 3. Constipation 4. Decreased respirations

1. Tinnitus Rationale: Salicylic acid is absorbed readily through the skin, and systemic toxicity (salicylism) can result. Symptoms include tinnitus, dizziness, hyperpnea, and psychological disturbances. Constipation and diarrhea are not associated with salicylism.

954. A client taking lithium carbonate (Lithobid) reports vomiting, abdominal pain, diarrhea, blurred vision, tinnitus, and tremors. The lithium level is 2.5 mEq/L. The nurse interprets this level as: 1. Toxic 2. Normal 3. Slightly above normal 4. Excessively below normal

1. Toxic

8.) A client with severe acne is seen in the clinic and the health care provider (HCP) prescribes isotretinoin. The nurse reviews the client's medication record and would contact the (HCP) if the client is taking which medication? 1. Vitamin A 2. Digoxin (Lanoxin) 3. Furosemide (Lasix) 4. Phenytoin (Dilantin)

1. Vitamin A Rationale: Isotretinoin is a metabolite of vitamin A and can produce generalized intensification of isotretinoin toxicity. Because of the potential for increased toxicity, vitamin A supplements should be discontinued before isotretinoin therapy. Options 2, 3, and 4 are not contraindicated with the use of isotretinoin.

239.) Which of the following precautions will the nurse specifically take during the administration of ribavirin (Virazole) to a child with respiratory syncytial virus (RSV)? 1. Wearing goggles 2. Wearing a gown 3. Wearing a gown and a mask 4. Handwashing before administration

1. Wearing goggles Rationale: Some caregivers experience headaches, burning nasal passages and eyes, and crystallization of soft contact lenses as a result of administration of ribavirin. Specific to this medication is the use of goggles. A gown is not necessary. A mask may be worn. Handwashing is to be performed before and after any child contact.

79.) Ibuprofen (Advil) is prescribed for a client. The nurse tells the client to take the medication: 1. With 8 oz of milk 2. In the morning after arising 3. 60 minutes before breakfast 4. At bedtime on an empty stomach

1. With 8 oz of milk Rationale: Ibuprofen is a nonsteroidal anti-inflammatory drug (NSAID). NSAIDs should be given with milk or food to prevent gastrointestinal irritation. Options 2, 3, and 4 are incorrect.

24.) A nurse is reinforcing teaching for a client regarding how to mix regular insulin and NPH insulin in the same syringe. Which of the following actions, if performed by the client, indicates the need for further teaching? 1. Withdraws the NPH insulin first 2. Withdraws the regular insulin first 3. Injects air into NPH insulin vial first 4. Injects an amount of air equal to the desired dose of insulin into the vial

1. Withdraws the NPH insulin first Rationale: When preparing a mixture of regular insulin with another insulin preparation, the regular insulin is drawn into the syringe first. This sequence will avoid contaminating the vial of regular insulin with insulin of another type. Options 2, 3, and 4 identify the correct actions for preparing NPH and regular insulin.

957. A client who has been taking buspirone for 1 month returns to the clinic for a followup assessment. The nurse determines that the medication is effective if the absence of which manifestation has occurred? 1. Paranoid thought process 2. Rapid heartbeat or anxiety 3. Alcohol withdrawal symptoms 4. Thought broadcasting or delusions

2 Buspirone (Buspar) is not recommended for the treatment of paranoid thought disorders, drug or alcohol withdrawal, or schizophrenia. Buspirone (Buspar) most often is indicated for the treatment of anxiety.

195.) A nurse is caring for a client who is taking metoprolol (Lopressor). The nurse measures the client's blood pressure (BP) and apical pulse (AP) immediately before administration. The client's BP is 122/78 mm/Hg and the AP is 58 beats/min. Based on this data, which of the following is the appropriate action? 1. Withhold the medication. 2. Notify the registered nurse immediately. 3. Administer the medication as prescribed. 4. Administer half of the prescribed medication.

1. Withhold the medication. Rationale: Metoprolol (Lopressor) is classified as a beta-adrenergic blocker and is used in the treatment of hypertension, angina, and myocardial infarction. Baseline nursing assessments include measurement of BP and AP immediately before administration. If the systolic BP is below 90 mm/Hg and the AP is below 60 beats/min, the nurse should withhold the medication and document this action. Although the registered nurse should be informed of the client's vital signs, it is not necessary to do so immediately. The medication should not be administered because the data is outside of the prescribed parameters for this medication. The nurse should not administer half of the medication, or alter any dosages at any point in time.

234.) A hospitalized client is started on phenelzine sulfate (Nardil) for the treatment of depression. At lunchtime, a tray is delivered to the client. Which food item on the tray will the nurse remove? 1. Yogurt 2. Crackers 3. Tossed salad 4. Oatmeal cookies

1. Yogurt Rationale: Phenelzine sulfate is a monoamine oxidase inhibitor (MAOI). The client should avoid taking in foods that are high in tyramine. These foods could trigger a potentially fatal hypertensive crisis. Foods to avoid include yogurt, aged cheeses, smoked or processed meats, red wines, and fruits such as avocados, raisins, or figs.

853. The nurse is preparing discharge instructions for a client receiving baclofen. Which instruction should be included in the teaching plan? 1. Restrict fluid intake. 2. Avoid the use of alcohol. 3. Stop the medication if diarrhea occurs. 4. Notify the health care provider (HCP) if fatigue occurs.

2 Baclofen is a skeletal muscle relaxant. The client should be cautioned against the use of alcohol and other central nervous system depressants because baclofen potentiates the depressant activity of these agents. Constipation rather than diarrhea is a side/adverse effect. Restriction of fluids is not necessary, but the client should be warned that urinary retention can occur. Fatigue is related to a central nervous system effect that is most intense during the early phase of therapy and diminishes with continued medication use. The client does not need to notify the HCP about fatigue.

360. A client in preterm labor (31 weeks) who is dilated to 4 cm has been started on magnesium sulfate and contractions have stopped. If the client's labor can be inhibited for the next 48 hours, the nurse anticipates a prescription for which medication? 1. Nalbuphine 2. Betamethasone 3. Rho(D) immune globulin 4. Dinoprostone

2 Betamethasone, a glucocorticoid, is given to increase the production of surfactant to stimulate fetal lung maturation. It is administered to clients in preterm labor at 28 to 32 weeks of gestation if the labor can be inhibited for 48 hours. Nalbuphine (Nubain) is an opioid analgesic. Rho(D) immune globulin (RhoGAM) is given to Rh-negative clients to prevent sensitization. Dinoprostone (Cervidil vaginal insert) is a prostaglandin given to ripen and soften the cervix and to stimulate uterine contractions.

761. Bethanechol chloride is prescribed for a client with urinary retention. Which disorder would be a contraindication to the administration of this medication? 1. Gastric atony 2. Urinary strictures 3. Neurogenic atony 4. Gastroesophageal reflux

2 Bethanechol chloride (Urecholine) can be hazardous to clients with urinary tract obstruction or weakness of the bladder wall. The medication has the ability to contract the bladder and thereby increase pressure within the urinary tract. Elevation of pressure within the urinary tract could rupture the bladder in clients with these conditions.

558. A client with acute myelocytic leukemia us being treated with busulfan. Which laboratory value would the nurse specifically monitor during treatment with this medication? 1. Clotting time 2. uric acid level 3. potassium level 4. blood glucose level

2 Busulfan (Myleran, Busulfex) can cause an increase in the uric acid level. Hyperuricemia can produce uric acid nephropathy, renal stones, and acute kidney injury. Option 1, 3, and 4 are not specifically related to this medication

720. A client with atrial fibrillation is receiving a continuous heparin infusion at 1000 units/hour. The nurse determines that the client is receiving the therapeutic effect based on which results? 1. Prothrombin time of 12.5 seconds 2. Activated partial thromboplastin time of 60 seconds 3. Activated partial thromboplastin time of 28 seconds 4. Activated partial thromboplastin time longer than 120 seconds

2 Common laboratory ranges for activated partial thromboplastin time are 20 to 36 seconds. Because the activated partial thromboplastin time should be 1.5 to 2.5 times the normal value, the client's activated partial thromboplastin time would be considered therapeutic if it was 60 seconds.

798. In preparation for cataract surgery, the nurse is to administer cyclopentolate eyedrops. The nurse understands that which characterizes the medication action? 1. Produces miosis of the operative eye 2. Dilates the pupil of the operative eye 3. Constricts the pupil of the operative eye 4. Provides lubrication to the operative eye

2 Cyclopentolate is a rapidly acting mydriatic and cycloplegic medication. Cyclopentolate is effective in 25 to 75 minutes, and accommodation returns in 6 to 24 hours. Cyclopentolate is used for preoperative mydriasis.

611. The nurse provides medication instructions to a client who is taking levothyroxine and should tell the client to notify the HCP if which problem occurs? 1. Fatigue 2. Tremors 3. Cold intolerance 4. Excessively dry skin

2 Excessive doses of levothyroxine can produce signs and symptoms of hyperthyroidism. These include tachycardia, chest pain, tremors, nervousness, insomnia, hyperthermia, extreme heat intolerance, and sweating. The client should be instructed to notify the HCP if these occur. Options 1, 3, and 4 are signs of hypothyroidism.

568. The nurse is providing medication instructions to a client with breast cancer who is receiving cyclophosphamide. The nurse should tell the client to take which actions? 1. Take the medication with food 2. Increase fluid intake to 2000-3000 mL/day 3. Decrease sodium intake while taking the medication 4. Increase potassium intake while taking the medication

2 Hemorrhagic cystitis is an adverse effect that can occur with the use of cyclophosphamide. The client needs to be instructed to drink copious amounts of fluid during the administration of this medication. Clients also should monitor urine output for hematuria. The medication should be taken on an empty stomach, unless GI upset occurs. Hyperkalemia can result from the use of medications; therefore, the client would not be told to increase potassium intake. The client would not be instructed to alter sodium intake.

790. Betaxolol hydrochloride eyedrops have been prescribed for a client with glaucoma. Which nursing action is most appropriate related to monitoring for side/adverse effects of this medication? 1. Monitoring temperature 2. Monitoring blood pressure 3. Assessing peripheral pulses 4. Assessing blood glucose level

2 Hypotension, dizziness, nausea, diaphoresis, headache, fatigue, constipation, and diarrhea are side/adverse effects of the medication. Nursing interventions include monitoring the blood pressure for hypotension and assessing the pulse for strength, weakness, irregular rate, and bradycardia. Options 1, 3, and 4 are not specifically associated with this medication.

849. A client has been on treatment for rheumatoid arthritis for 3 weeks. During the administration of etanercept, which is most important for the nurse to assess? 1. The injection site for itching and edema 2. The white blood cell counts and platelet counts 3. Whether the client is experiencing fatigue and joint pain 4. Whether the client is experiencing a metallic taste in the mouth, and a loss of appetite

2 Infection and pancytopenia are side/adverse effects of etanercept. Laboratory studies are performed prior to and during medication treatment. The appearance of abnormal white blood cell counts and abnormal platelet counts can alert the nurse to a potentially life-threatening infection. Injection site itching is a common occurrence following administration. A metallic taste and loss of appetite are not common signs of side/adverse effects of this medication.

685. A client has been taking isoniazid for 11⁄2 months. The client complains to the nurse about numbness, paresthesias, and tingling in the extremities. The nurse interprets that the client is experiencing which problem? 1. Hypercalcemia 2. Peripheral neuritis 3. Small blood vessel spasm 4. Impaired peripheral circulation

2 Isoniazid is an antitubercular medication. A common side effect of isoniazid is peripheral neuritis, manifested by numbness, tingling, and paresthesias in the extremities. This can be minimized with pyridoxine (vitamin B6) intake. Options 1, 3, and 4 are incorrect.

686. A client is to begin a 6-month course of therapy with isoniazid. The nurse should plan to teach the client to take which action? 1. Use alcohol in small amounts only. 2. Report yellow eyes or skin immediately. 3. Increase intake of Swiss or aged cheeses. 4. Avoid vitamin supplements during therapy.

2 Isoniazid is hepatotoxic, and therefore the client is taught to report signs and symptoms of hepatitis immediately, which include yellow skin and sclera. For the same reason, alcohol should be avoided during therapy. The client should avoid intake of Swiss cheese, fish such as tuna, and foods containing tyramine because they may cause a reaction characterized by redness and itching of the skin, flushing, sweating, tachycardia, headache, or lightheadedness. The client can avoid developing peripheral neuritis by increasing the intake of pyridoxine (vitamin B6) during the course of isoniazid therapy.

357. A pregnant client is receiving magnesium sulfate for the management of preeclampsia. The nurse determines that the client is experiencing toxicity from the medication if which finding is noted on assessment? 1. Proteinuria of 3 + 2. Respirations of 10 breaths/minute 3. Presence of deep tendon reflexes 4. Serum magnesium level of 6 mEq/L

2 Magnesium toxicity can occur from magnesium sulfate therapy. Signs of magnesium sulfate toxicity relate to the central nervous system depressant effects of the medication and include respiratory depression, loss of deep tendon reflexes, and a sudden decline in fetal heart rate and maternal heart rate and blood pressure. Therapeutic serum levels of magnesium are 4 to 7.5 mEq/L. Proteinuria of 3 + is an expected finding in a client with preeclampsia.

chapter 77- psych 944. A client's medication sheet contains a prescription for sertraline (Zoloft). To ensure safe administration of the medication, a nurse would administer the dose: 1. On an empty stomach 2. At the same time each evening 3. Evenly spaced around the clock 4. As needed when the client complains of depression

2. At the same time each evening

osteoporosis

most common cause of fractured hip= reduced calcium in bones result of hormonal changes during perimenaopausal years

361. Methylergonovine is prescribed for a woman to treat postpartum hemorrhage. Before administration of methylergonovine, what is the priority nursing assessment? 1. Uterine tone 2. Blood pressure 3. Amount of lochia 4. Deep tendon reflexes

2 Methylergonovine, an ergot alkaloid, is used to prevent or control postpartum hemorrhage by contracting the uterus. Methylergonovine causes continuous uterine contractions and may elevate the blood pressure. A priority assessment before the administration of the medication is to check the blood pressure. The health care provider should be notified if hypertension is present. Although options 1, 3, and 4 may be components of the postpartum assessment, the correct option, blood pressure, is related specifically to the administration of this medication.

650. A client has been taking omeprazole for 4 weeks. The ambulatory care nurse evaluates that the client is receiving the optimal intended effect of the medication if the client reports the absence of which symptom? 1. Diarrhea 2. Heartburn 3. Flatulence 4. Constipation

2 Omeprazole (Prilosec) is a proton pump inhibitor classified as an antiulcer agent. The intended effect of the medication is relief of pain from gastric irritation, often called heartburn by clients. Omeprazole is not used to treat the conditions identified in options 1, 3, and 4.

795. Which medication, if prescribed for the client with glaucoma, should the nurse question? 1. Betaxolol 2. Atropine sulfate 3. Pilocarpine hydrochloride 4. Pilocarpine

2 Options 1, 3, and 4 are miotic agents used to treat glaucoma. The correct option is a mydriatic and cycloplegic (also anticholinergic) medication, and its use is contraindicated in clients with glaucoma. Mydriatic medications dilate the pupil and can cause an increase in intraocular pressure in the eye.

951. The nurse is describing the medication side and adverse effects to a client who is taking oxazepam. What information should the nurse incorporate in the discussion? 1. Consume a low-fiber diet. 2. Increase fluids and bulk in the diet. 3. Rest if the heart begins to beat rapidly. 4. Take antidiarrheal agents if diarrhea occurs.

2 Oxazepam (Serax) causes constipation, and the client is instructed to increase fluid intake and bulk (high fiber) in the diet. If the heart begins to beat fast, the health care provider (HCP) is notified because this could indicate overdose. In addition, diarrhea could indicate an incomplete intestinal obstruction and, if this occurs, the HCP is notified.

612. The nurse performs an admission assessment on a client who visits a health care clinic for the first time. The client tells the nurse that propylthiouracil is taken daily. The nurse continues to collect data from the client, suspecting that the client has a history of which condition? 1. Myxedema 2. Graves' disease 3. Addison's disease 4. Cushing's syndrome

2 Propylthiouracil (PTU) inhibits thyroid hormone synthesis and is used to treat hyperthyroidism, or Graves' disease. Myxedema indicates hypothyroidism. Cushing's syndrome and Addison's disease are disorders related to adrenal function.

362. The nurse is preparing to administer beractant to a premature infant who has respiratory distress syndrome. The nurse plans to administer the medication by which route? 1. Intradermal 2. Intratracheal 3. Subcutaneous 4. Intramuscular

2 Respiratory distress syndrome is a serious lung disorder caused by immaturity and the inability to produce surfactant, resulting in hypoxia and acidosis. It is common in premature infants and may be due to lung immaturity as a result of surfactant deficiency. The mainstay of treatment is the administration of exogenous surfactant, which is administered by the intratracheal route. Options 1, 3, and 4 are not routes of administration for this medication.

690. The nurse has a prescription to give a client salmeterol, two puffs, and beclomethasone dipropionate, two puffs, by metered-dose inhaler. The nurse should administer the medication using which procedure? 1. Beclomethasone first and then the salmeterol 2. Salmeterol first and then the beclomethasone 3. Alternating a single puff of each, beginning with the salmeterol 4. Alternating a single puff of each, beginning with the beclomethasone

2 Salmeterol (Serevent Diskus) is an adrenergic type of bronchodilator and beclomethasone dipropionate (Qvar) is a glucocorticoid. Bronchodilators are always administered before glucocorticoids when both are to be given on the same time schedule. This allows for widening of the air passages by the bronchodilator, which then makes the glucocorticoid more effective.

960. A hospitalized client has begun taking bupropion as an antidepressant agent. The nurse understands that which is an adverse effect, indicating that the client is taking an excessive amount of medication? 1. Constipation 2. Seizure activity 3. Increased weight 4. Dizziness when getting upright

2 Seizure activity can occur in clients taking bupropion (Wellbutrin) dosages greater than 450 mg daily. Weight gain is an occasional side effect, whereas constipation is a common side effect of this medication. This medication does not cause significant orthostatic blood pressure changes.

948. A client's medication sheet contains a prescription for sertraline. To ensure safe administration of the medication, how should the nurse administer the dose? 1. On an empty stomach 2. At the same time each evening 3. Evenly spaced around the clock 4. As needed when the client complains of depression

2 Sertraline (Zoloft) is classified as an antidepressant. Sertraline (Zoloft) generally is administered once every 24 hours. It may be administered in the morning or evening, but evening administration may be preferable because drowsiness is a side effect. The medication may be administered without food or with food if gastrointestinal distress occurs. Sertraline (Zoloft) is not prescribed for use as needed.

564. A client with metastatic breast cancer is receiving tamoxifen. The nurse specifically monitors which laboratory value while the client is taking this medication? 1. glucose level 2. calcium level 3. potassium level 4. prothrombin time

2 Tamoxifen may increase calcium, cholesterol, and triglyceride levels. Before the initiation of therapy, a complete blood count, platelet count, and serum calcium level should be assessed. These blood levels, along with cholesterol and triglyceride levels should be monitored periodically during therapy. The nurse should assess for hypercalcemia while the client is taking this medication. signs of hypercalcemia include increased urine volume, excessive thirst, nausea, vomiting, constipation, hypotonicity of muscles, and deep bone and flank pain.

gangrene

necrosis/tissue death priority prevent infection

955. The nurse notes that a client with schizophrenia and receiving an antipsychotic medication is moving her mouth, protruding her tongue, and grimacing as she watches television. The nurse determines that the client is experiencing which medication complication? 1. Parkinsonism 2. Tardive dyskinesia 3. Hypertensive crisis 4. Neuroleptic malignant syndrome

2 Tardive dyskinesia is a reaction that can occur from antipsychotic medication. It is characterized by uncontrollable involuntary movements of the body and extremities, particularly the tongue. Parkinsonism is characterized by tremors, masklike facies, rigidity, and a shuffling gait. Hypertensive crisis can occur from the use of monoamine oxidase inhibitors and is characterized by hypertension, occipital headache radiating frontally, neck stiffness and soreness, nausea, and vomiting. Neuroleptic malignant syndrome is a potentially fatal syndrome that may occur at any time during therapy with neuroleptic (antipsychotic) medications. It is characterized by dyspnea or tachypnea, tachycardia or irregular pulse rate, fever, blood pressure changes, increased sweating, loss of bladder control, and skeletal muscle rigidity.

649. A client who chronically uses NSAIDs has been taking misoprostol. The nurse determines that the medication is having the intended therapeutic effect if which finding is noted? 1. Resolved diarrhea 2. Relief of epigastric pain 3. Decreased platelet count 4. Decreased white blood cell count

2 The client who chronically uses NSAIDs is prone to gastric mucosal injury. Misoprostol (Cytotec) is a gastric protectant and is given specifically to prevent this occurrence. Diarrhea can be a side effect of this medication but is not an intended effect. Options 3 and 4 are incorrect.

643. A client with Crohn's disease is scheduled to receive an infusion of infliximab. What intervention by the nurse will determine the effectiveness of treatment? 1. Monitoring the leukocyte count for 2 days after the infusion 2. Checking the frequency and consistency of bowel movements 3. Checking serum liver enzyme levels before and after the infusion 4. Carrying out a Hematest on gastric fluids after the infusion is completed

2 The principle manifestations of Crohn's disease are diarrhea and abdominal pain. Infliximab (Remicade) is an immunomodulator that reduces the degree of inflammation in the colon, thereby reducing the diarrhea. Options 1, 3, and 4 are unrelated to this medication.

827. The nurse has given medication instructions to a client receiving phenytoin. Which statement indicates that the client has an adequate understanding of the instructions? 1. "Alcohol is not contraindicated while taking this medication." 2. "Good oral hygiene is needed, including brushing and flossing." 3. "The medication dose may be self-adjusted, depending on side effects." 4. "The morning dose of the medication should be taken before a serum drug level is drawn."

2 Typical anticonvulsant medication instructions include taking the prescribed daily dosage to keep the blood level of the drug constant and having a sample drawn for serum drug level determination before taking the morning dose. The client is taught not to stop the medication abruptly, to avoid alcohol, to check with a health care provider before taking over-the-counter medications, to avoid activities in which alertness and coordination are required until medication effects are known, to provide good oral hygiene, and to obtain regular dental care. The client should also wear a Medic-Alert bracelet.

725. A client with atrial fibrillation secondary to mitral stenosis is receiving a heparin sodium infusion at 1000 units/hour and warfarin sodium 7.5 mg at 5:00 PM daily. The morning laboratory results are as follows: activated partial thromboplastin time (aPTT) = 32 seconds; internationalized normalized ratio (INR) = 1.3. The nurse should plan to take which action based on the client's laboratory results? 1. Collaborate with the health care provider (HCP) to discontinue the heparin infusion and administer the warfarin sodium as prescribed. 2. Collaborate with the HCP to obtain a prescription to increase the heparin infusion and administer the warfarin sodium as prescribed. 3. Collaborate with the HCP to withhold the warfarin sodium since the client is receiving a heparin infusion and the aPTT is within the therapeutic range. 4. Collaborate with the HCP to continue the heparin infusion at the same rate and to discuss use of dabigatran etexilate in place of warfarin sodium.

2 When a client is receiving warfarin (Coumadin) for clot prevention due to atrial fibrillation, an INR of 2 to 3 is appropriate for most clients. Until the INR has achieved a therapeutic range the client should be maintained on a continuous heparin infusion with the aPTT ranging between 60 and 80 seconds. Therefore, the nurse should collaborate with the health care provider to obtain a prescription to increase the heparin infusion and to administer the warfarin as prescribed.

822. Meperidine hydrochloride has been prescribed for a client to treat pain. Which are side/adverse effects of this medication? Select all that apply. 1. Diarrhea 2. Tremors 3. Drowsiness 4. Hypotension 5. Urinary frequency 6. Increased respiratory rate

2, 3, 4 Meperidine hydrochloride is an opioid analgesic. Side/adverse effects include respiratory depression, drowsiness, hypotension, constipation, urinary retention, nausea, vomiting, and tremors.

729. The nurse is monitoring a client who is taking digoxin for adverse effects. Which findings are characteristic of digoxin toxicity? Select all that apply. 1. Tremors 2. Diarrhea 3. Irritability 4. Blurred vision 5. Nausea and vomiting

2, 4, 5 Digoxin (Lanoxin) is a cardiac glycoside. The risk of toxicity can occur with the use of this medication. Toxicity can lead to life-threatening events and the nurse needs to monitor the client closely for signs of toxicity. Early signs of toxicity include gastrointestinal manifestations such as anorexia, nausea, vomiting, and diarrhea. Subsequent manifestations include headache; visual disturbances such as diplopia, blurred vision, yellow-green halos, and photophobia; drowsiness; fatigue; and weakness. Cardiac rhythm abnormalities can also occur. The nurse also monitors the digoxin level. Therapeutic levels for digoxin range from 0.5 to 2 ng/mL.

176.) A nurse notes that a client is taking lansoprazole (Prevacid). On data collection, the nurse asks which question to determine medication effectiveness? 1. "Has your appetite increased?" 2. "Are you experiencing any heartburn?" 3. "Do you have any problems with vision?" 4. "Do you experience any leg pain when walking?"

2. "Are you experiencing any heartburn?" Rationale: Lansoprazole is a gastric acid pump inhibitor used to treat gastric and duodenal ulcers, erosive esophagitis, and hypersecretory conditions. It also is used to treat gastroesophageal reflux disease (GERD). It is not used to treat visual problems, problems with appetite, or leg pain. **NOTE: "-zole" refers to gastric acid pump inhibitors**

183.) A client who received a kidney transplant is taking azathioprine (Imuran), and the nurse provides instructions about the medication. Which statement by the client indicates a need for further instructions? 1. "I need to watch for signs of infection." 2. "I need to discontinue the medication after 14 days of use." 3. "I can take the medication with meals to minimize nausea." 4. "I need to call the health care provider (HCP) if more than one dose is missed."

2. "I need to discontinue the medication after 14 days of use." Rationale: Azathioprine is an immunosuppressant medication that is taken for life. Because of the effects of the medication, the client must watch for signs of infection, which are reported immediately to the HCP. The client should also call the HCP if more than one dose is missed. The medication may be taken with meals to minimize nausea.

78.) A client is taking phenytoin (Dilantin) for seizure control and a sample for a serum drug level is drawn. Which of the following indicates a therapeutic serum drug range? 1. 5 to 10 mcg/mL 2. 10 to 20 mcg/mL 3. 20 to 30 mcg/mL 4. 30 to 40 mcg/mL

2. 10 to 20 mcg/mL Rationale: The therapeutic serum drug level range for phenytoin (Dilantin) is 10 to 20 mcg/mL. ** A helpful hint may be to remember that the theophylline therapeutic range and the acetaminophen (Tylenol) therapeutic range are the same as the phenytoin (Dilantin) therapeutic range.**

215.) A client with rheumatoid arthritis is taking acetylsalicylic acid (aspirin) on a daily basis. Which medication dose should the nurse expect the client to be taking? 1. 1 g daily 2. 4 g daily 3. 325 mg daily 4. 1000 mg daily

2. 4 g daily Rationale: Aspirin may be used to treat the client with rheumatoid arthritis. It may also be used to reduce the risk of recurrent transient ischemic attack (TIA) or brain attack (stroke) or reduce the risk of myocardial infarction (MI) in clients with unstable angina or a history of a previous MI. The normal dose for clients being treated with aspirin to decrease thrombosis and MI is 300 to 325 mg/day. Clients being treated to prevent TIAs are usually prescribed 1.3 g/day in two to four divided doses. Clients with rheumatoid arthritis are treated with 3.6 to 5.4 g/day in divided doses. **Eliminate options 1 and 4 because they are alike**

88.) Dantrolene sodium (Dantrium) is prescribed for a client experiencing flexor spasms, and the client asks the nurse about the action of the medication. The nurse responds, knowing that the therapeutic action of this medication is which of the following? 1. Depresses spinal reflexes 2. Acts directly on the skeletal muscle to relieve spasticity 3. Acts within the spinal cord to suppress hyperactive reflexes 4. Acts on the central nervous system (CNS) to suppress spasms

2. Acts directly on the skeletal muscle to relieve spasticity Rationale: Dantrium acts directly on skeletal muscle to relieve muscle spasticity. The primary action is the suppression of calcium release from the sarcoplasmic reticulum. This in turn decreases the ability of the skeletal muscle to contract. **Options 1, 3, and 4 are all comparable or alike in that they address CNS suppression and the depression of reflexes. Therefore, eliminate these options.**

normocapnia

normal arterial carbon dioxide pressure (PaCO2 35 to 45 mm Hg).

74.) A client with myasthenia gravis is receiving pyridostigmine (Mestinon). The nurse monitors for signs and symptoms of cholinergic crisis caused by overdose of the medication. The nurse checks the medication supply to ensure that which medication is available for administration if a cholinergic crisis occurs? 1. Vitamin K 2. Atropine sulfate 3. Protamine sulfate 4. Acetylcysteine (Mucomyst)

2. Atropine sulfate Rationale: The antidote for cholinergic crisis is atropine sulfate. Vitamin K is the antidote for warfarin (Coumadin). Protamine sulfate is the antidote for heparin, and acetylcysteine (Mucomyst) is the antidote for acetaminophen (Tylenol).

86.) A nurse is reinforcing discharge instructions to a client receiving baclofen (Lioresal). Which of the following would the nurse include in the instructions? 1. Restrict fluid intake. 2. Avoid the use of alcohol. 3. Stop the medication if diarrhea occurs. 4. Notify the health care provider if fatigue occurs.

2. Avoid the use of alcohol. Rationale: Baclofen is a central nervous system (CNS) depressant. The client should be cautioned against the use of alcohol and other CNS depressants, because baclofen potentiates the depressant activity of these agents. Constipation rather than diarrhea is an adverse effect of baclofen. It is not necessary to restrict fluids, but the client should be warned that urinary retention can occur. Fatigue is related to a CNS effect that is most intense during the early phase of therapy and diminishes with continued medication use. It is not necessary that the client notify the health care provider if fatigue occurs.

199.) A nurse is applying a topical glucocorticoid to a client with eczema. The nurse monitors for systemic absorption of the medication if the medication is being applied to which of the following body areas? 1. Back 2. Axilla 3. Soles of the feet 4. Palms of the hands

2. Axilla Rationale: Topical glucocorticoids can be absorbed into the systemic circulation. Absorption is higher from regions where the skin is especially permeable (scalp, axillae, face, eyelids, neck, perineum, genitalia), and lower from regions where penetrability is poor (back, palms, soles). **Eliminate options 3 and 4 because these body areas are similar in terms of skin characteristics**

123.) A nurse is planning to administer amlodipine (Norvasc) to a client. The nurse plans to check which of the following before giving the medication? 1. Respiratory rate 2. Blood pressure and heart rate 3. Heart rate and respiratory rate 4. Level of consciousness and blood pressure

2. Blood pressure and heart rate Rationale: Amlodipine is a calcium channel blocker. This medication decreases the rate and force of cardiac contraction. Before administering a calcium channel blocking agent, the nurse should check the blood pressure and heart rate, which could both decrease in response to the action of this medication. This action will help to prevent or identify early problems related to decreased cardiac contractility, heart rate, and conduction. **amlodipine is a calcium channel blocker, and this group of medications decreases the rate and force of cardiac contraction. This in turn lowers the pulse rate and blood pressure.**

224.) Neuroleptic malignant syndrome is suspected in a client who is taking chlorpromazine. Which medication would the nurse prepare in anticipation of being prescribed to treat this adverse effect related to the use of chlorpromazine? 1. Protamine sulfate 2. Bromocriptine (Parlodel) 3. Phytonadione (vitamin K) 4. Enalapril maleate (Vasotec)

2. Bromocriptine (Parlodel) Rationale: Bromocriptine is an antiparkinsonian prolactin inhibitor used in the treatment of neuroleptic malignant syndrome. Vitamin K is the antidote for warfarin (Coumadin) overdose. Protamine sulfate is the antidote for heparin overdose. Enalapril maleate is an antihypertensive used in the treatment of hypertension.

20.) The client with metastatic breast cancer is receiving tamoxifen. The nurse specifically monitors which laboratory value while the client is taking this medication? 1. Glucose level 2. Calcium level 3. Potassium level 4. Prothrombin time

2. Calcium level Rationale: Tamoxifen may increase calcium, cholesterol, and triglyceride levels. Before the initiation of therapy, a complete blood count, platelet count, and serum calcium levels should be assessed. These blood levels, along with cholesterol and triglyceride levels, should be monitored periodically during therapy. The nurse should assess for hypercalcemia while the client is taking this medication. Signs of hypercalcemia include increased urine volume, excessive thirst, nausea, vomiting, constipation, hypotonicity of muscles, and deep bone and flank pain.

170.) Atenolol hydrochloride (Tenormin) is prescribed for a hospitalized client. The nurse should perform which of the following as a priority action before administering the medication? 1. Listen to the client's lung sounds. 2. Check the client's blood pressure. 3. Check the recent electrolyte levels. 4. Assess the client for muscle weakness.

2. Check the client's blood pressure. Rationale: Atenolol hydrochloride is a beta-blocker used to treat hypertension. Therefore the priority nursing action before administration of the medication is to check the client's blood pressure. The nurse also checks the client's apical heart rate. If the systolic blood pressure is below 90 mm Hg or the apical pulse is 60 beats per minute or lower, the medication is withheld and the registered nurse and/or health care provider is notified. The nurse would check baseline renal and liver function tests. The medication may cause weakness, and the nurse would assist the client with activities if weakness occurs. **Beta-blockers have "-lol" at the end of the medication name**

34.) A client with Crohn's disease is scheduled to receive an infusion of infliximab (Remicade). The nurse assisting in caring for the client should take which action to monitor the effectiveness of treatment? 1. Monitoring the leukocyte count for 2 days after the infusion 2. Checking the frequency and consistency of bowel movements 3. Checking serum liver enzyme levels before and after the infusion 4. Carrying out a Hematest on gastric fluids after the infusion is completed

2. Checking the frequency and consistency of bowel movements Rationale: The principal manifestations of Crohn's disease are diarrhea and abdominal pain. Infliximab (Remicade) is an immunomodulator that reduces the degree of inflammation in the colon, thereby reducing the diarrhea. Options 1, 3, and 4 are unrelated to this medication.

146.) A client has begun therapy with theophylline (Theo-24). The nurse tells the client to limit the intake of which of the following while taking this medication? 1. Oranges and pineapple 2. Coffee, cola, and chocolate 3. Oysters, lobster, and shrimp 4. Cottage cheese, cream cheese, and dairy creamers

2. Coffee, cola, and chocolate Rationale: Theophylline is a xanthine bronchodilator. The nurse teaches the client to limit the intake of xanthine-containing foods while taking this medication. These include coffee, cola, and chocolate.

63.) A client with coronary artery disease complains of substernal chest pain. After checking the client's heart rate and blood pressure, a nurse administers nitroglycerin, 0.4 mg, sublingually. After 5 minutes, the client states, "My chest still hurts." Select the appropriate actions that the nurse should take. Select all that apply. 1. Call a code blue. 2. Contact the registered nurse. 3. Contact the client's family. 4. Assess the client's pain level. 5. Check the client's blood pressure. 6. Administer a second nitroglycerin, 0.4 mg, sublingually.

2. Contact the registered nurse. 4. Assess the client's pain level. 5. Check the client's blood pressure. 6. Administer a second nitroglycerin, 0.4 mg, sublingually. Rationale: The usual guideline for administering nitroglycerin tablets for a hospitalized client with chest pain is to administer one tablet every 5 minutes PRN for chest pain, for a total dose of three tablets. The registered nurse should be notified of the client's condition, who will then notify the health care provider as appropriate. Because the client is still complaining of chest pain, the nurse would administer a second nitroglycerin tablet. The nurse would assess the client's pain level and check the client's blood pressure before administering each nitroglycerin dose. There are no data in the question that indicate the need to call a code blue. In addition, it is not necessary to contact the client's family unless the client has requested this.

209.) A client with multiple sclerosis is receiving diazepam (Valium), a centrally acting skeletal muscle relaxant. Which of the following would indicate that the client is experiencing a side effect related to this medication? 1. Headache 2. Drowsiness 3. Urinary retention 4. Increased salivation

2. Drowsiness Rationale: Incoordination and drowsiness are common side effects resulting from this medication. Options 1, 3, and 4 are incorrect.

240.) A client with Parkinson's disease has been prescribed benztropine (Cogentin). The nurse monitors for which gastrointestinal (GI) side effect of this medication? 1. Diarrhea 2. Dry mouth 3. Increased appetite 4. Hyperactive bowel sounds

2. Dry mouth Rationale: Common GI side effects of benztropine therapy include constipation and dry mouth. Other GI side effects include nausea and ileus. These effects are the result of the anticholinergic properties of the medication. **Eliminate options 1 and 4 because they are comparable or alike. Recall that the medication is an anticholinergic, which causes dry mouth**

952. A nurse is performing a follow-up teaching session with a client discharged 1 month ago. The client is taking fluoxetine (Prozac). What information would be important for the nurse to obtain during this client visit regarding the side effects of the medication? 1. Cardiovascular symptoms 2. Gastrointestinal dysfunctions 3. Problems with mouth dryness 4. Problems with excessive sweating

2. Gastrointestinal dysfunctions

108.) A nurse is performing a follow-up teaching session with a client discharged 1 month ago who is taking fluoxetine (Prozac). What information would be important for the nurse to gather regarding the adverse effects related to the medication? 1. Cardiovascular symptoms 2. Gastrointestinal dysfunctions 3. Problems with mouth dryness 4. Problems with excessive sweating

2. Gastrointestinal dysfunctions Rationale: The most common adverse effects related to fluoxetine include central nervous system (CNS) and gastrointestinal (GI) system dysfunction. This medication affects the GI system by causing nausea and vomiting, cramping, and diarrhea. Options 1, 3, and 4 are not adverse effects of this medication.

131.) The nurse is reinforcing medication instructions to a client with breast cancer who is receiving cyclophosphamide (Neosar). The nurse tells the client to: 1. Take the medication with food. 2. Increase fluid intake to 2000 to 3000 mL daily. 3. Decrease sodium intake while taking the medication. 4. Increase potassium intake while taking the medication.

2. Increase fluid intake to 2000 to 3000 mL daily. Rationale: Hemorrhagic cystitis is a toxic effect that can occur with the use of cyclophosphamide. The client needs to be instructed to drink copious amounts of fluid during the administration of this medication. Clients also should monitor urine output for hematuria. The medication should be taken on an empty stomach, unless gastrointestinal (GI) upset occurs. Hyperkalemia can result from the use of the medication; therefore the client would not be told to increase potassium intake. The client would not be instructed to alter sodium intake.

947. A nurse is describing the medication side effects to a client who is taking oxazepam (Serax). The nurse incorporates in discussions with the client the need to: 1. Consume a low-fiber diet. 2. Increase fluids and bulk in the diet. 3. Rest if the heart begins to beat rapidly. 4. Take antidiarrheal agents if diarrhea occurs.

2. Increase fluids and bulk in the diet.

101.) Ketoconazole is prescribed for a client with a diagnosis of candidiasis. Select the interventions that the nurse includes when administering this medication. Select all that apply. 1. Restrict fluid intake. 2. Instruct the client to avoid alcohol. 3. Monitor hepatic and liver function studies. 4. Administer the medication with an antacid. 5. Instruct the client to avoid exposure to the sun. 6. Administer the medication on an empty stomach.

2. Instruct the client to avoid alcohol. 3. Monitor hepatic and liver function studies. 5. Instruct the client to avoid exposure to the sun. Rationale: Ketoconazole is an antifungal medication. It is administered with food (not on an empty stomach) and antacids are avoided for 2 hours after taking the medication to ensure absorption. The medication is hepatotoxic and the nurse monitors liver function studies. The client is instructed to avoid exposure to the sun because the medication increases photosensitivity. The client is also instructed to avoid alcohol. There is no reason for the client to restrict fluid intake. In fact, this could be harmful to the client.

89.) A nurse is reviewing the laboratory studies on a client receiving dantrolene sodium (Dantrium). Which laboratory test would identify an adverse effect associated with the administration of this medication? 1. Creatinine 2. Liver function tests 3. Blood urea nitrogen 4. Hematological function tests

2. Liver function tests Rationale: Dose-related liver damage is the most serious adverse effect of dantrolene. To reduce the risk of liver damage, liver function tests should be performed before treatment and periodically throughout the treatment course. It is administered in the lowest effective dosage for the shortest time necessary. **Eliminate options 1 and 3 because these tests both assess kidney function.**

65.) A nurse is reinforcing discharge instructions to a client receiving sulfisoxazole. Which of the following should be included in the list of instructions? 1. Restrict fluid intake. 2. Maintain a high fluid intake. 3. If the urine turns dark brown, call the health care provider (HCP) immediately. 4. Decrease the dosage when symptoms are improving to prevent an allergic response.

2. Maintain a high fluid intake. Rationale: Each dose of sulfisoxazole should be administered with a full glass of water, and the client should maintain a high fluid intake. The medication is more soluble in alkaline urine. The client should not be instructed to taper or discontinue the dose. Some forms of sulfisoxazole cause urine to turn dark brown or red. This does not indicate the need to notify the HCP.

161.) A nurse is caring for a client with severe back pain, and codeine sulfate has been prescribed for the client. Which of the following would the nurse include in the plan of care while the client is taking this medication? 1. Restrict fluid intake. 2. Monitor bowel activity. 3. Monitor for hypertension. 4. Monitor peripheral pulses.

2. Monitor bowel activity. Rationale: While the client is taking codeine sulfate, an opioid analgesic, the nurse would monitor vital signs and monitor for hypotension. The nurse should also increase fluid intake, palpate the bladder for urinary retention, auscultate bowel sounds, and monitor the pattern of daily bowel activity and stool consistency (codeine can cause constipation). The nurse should monitor respiratory status and initiate breathing and coughing exercises. In addition, the nurse monitors the effectiveness of the pain medication.

204.) A client receives a dose of edrophonium (Enlon). The client shows improvement in muscle strength for a period of time following the injection. The nurse interprets that this finding is compatible with: 1. Multiple sclerosis 2. Myasthenia gravis 3. Muscular dystrophy 4. Amyotrophic lateral sclerosis

2. Myasthenia gravis Rationale: Myasthenia gravis can often be diagnosed based on clinical signs and symptoms. The diagnosis can be confirmed by injecting the client with a dose of edrophonium . This medication inhibits the breakdown of an enzyme in the neuromuscular junction, so more acetylcholine binds to receptors. If the muscle is strengthened for 3 to 5 minutes after this injection, it confirms a diagnosis of myasthenia gravis. Another medication, neostigmine (Prostigmin), also may be used because its effect lasts for 1 to 2 hours, providing a better analysis. For either medication, atropine sulfate should be available as the antidote.

949. A nurse is teaching a client who is being started on imipramine (Tofranil) about the medication. The nurse informs the client that the maximum desired effects may: 1. Start during the first week of administration 2. Not occur for 2 to 3 weeks of administration 3. Start during the second week of administration 4. Not occur until after 2 months of administration

2. Not occur for 2 to 3 weeks of administration

227.) When teaching a client who is being started on imipramine hydrochloride (Tofranil), the nurse would inform the client that the desired effects of the medication may: 1. Start during the first week of administration 2. Not occur for 2 to 3 weeks of administration 3. Start during the second week of administration 4. Not occur until after a month of administration

2. Not occur for 2 to 3 weeks of administration Rationale: The therapeutic effects of administration of imipramine hydrochloride may not occur for 2 to 3 weeks after the antidepressant therapy has been initiated. Therefore options 1, 3, and 4 are incorrect.

chronic bronchitis

obstructive pulmonary disease characterized by excessive production of mucus and chronic inflammatory changes in the bronchi, resulting in a cough with expectoration for at least 3 months of the year for more than 2 consecutive years.

228.) A client receiving an anxiolytic medication complains that he feels very "faint" when he tries to get out of bed in the morning. The nurse recognizes this complaint as a symptom of: 1. Cardiac dysrhythmias 2. Postural hypotension 3. Psychosomatic symptoms 4. Respiratory insufficiency

2. Postural hypotension Rationale: Anxiolytic medications can cause postural hypotension. The client needs to be taught to rise to a sitting position and get out of bed slowly because of this adverse effect related to the medication. Options 1, 3, and 4 are unrelated to the use of this medication.

50.) A nurse has given a client taking ethambutol (Myambutol) information about the medication. The nurse determines that the client understands the instructions if the client states that he or she will immediately report: 1. Impaired sense of hearing 2. Problems with visual acuity 3. Gastrointestinal (GI) side effects 4. Orange-red discoloration of body secretions

2. Problems with visual acuity Rationale: Ethambutol causes optic neuritis, which decreases visual acuity and the ability to discriminate between the colors red and green. This poses a potential safety hazard when a client is driving a motor vehicle. The client is taught to report this symptom immediately. The client is also taught to take the medication with food if GI upset occurs. Impaired hearing results from antitubercular therapy with streptomycin. Orange-red discoloration of secretions occurs with rifampin (Rifadin).

25.) A home care nurse visits a client recently diagnosed with diabetes mellitus who is taking Humulin NPH insulin daily. The client asks the nurse how to store the unopened vials of insulin. The nurse tells the client to: 1. Freeze the insulin. 2. Refrigerate the insulin. 3. Store the insulin in a dark, dry place. 4. Keep the insulin at room temperature.

2. Refrigerate the insulin. Rationale: Insulin in unopened vials should be stored under refrigeration until needed. Vials should not be frozen. When stored unopened under refrigeration, insulin can be used up to the expiration date on the vial. Options 1, 3, and 4 are incorrect.

40.) The client who chronically uses nonsteroidal anti-inflammatory drugs has been taking misoprostol (Cytotec). The nurse determines that the medication is having the intended therapeutic effect if which of the following is noted? 1. Resolved diarrhea 2. Relief of epigastric pain 3. Decreased platelet count 4. Decreased white blood cell count

2. Relief of epigastric pain Rationale: The client who chronically uses nonsteroidal anti-inflammatory drugs (NSAIDs) is prone to gastric mucosal injury. Misoprostol is a gastric protectant and is given specifically to prevent this occurrence. Diarrhea can be a side effect of the medication, but is not an intended effect. Options 3 and 4 are incorrect.

168.) Colcrys (colchicine) is prescribed for a client with a diagnosis of gout. The nurse reviews the client's medical history in the health record, knowing that the medication would be contraindicated in which disorder? 1. Myxedema 2. Renal failure 3. Hypothyroidism 4. Diabetes mellitus

2. Renal failure Rationale: Colchicine is contraindicated in clients with severe gastrointestinal, renal, hepatic or cardiac disorders, or with blood dyscrasias. Clients with impaired renal function may exhibit myopathy and neuropathy manifested as generalized weakness. This medication should be used with caution in clients with impaired hepatic function, older clients, and debilitated clients. **Note that options 1, 3, and 4 are all endocrine-related disorders: Myxedema=Hypothyroidism**

48.) A client is to begin a 6-month course of therapy with isoniazid (INH). A nurse plans to teach the client to: 1. Drink alcohol in small amounts only. 2. Report yellow eyes or skin immediately. 3. Increase intake of Swiss or aged cheeses. 4. Avoid vitamin supplements during therapy.

2. Report yellow eyes or skin immediately. Rationale: INH is hepatotoxic, and therefore the client is taught to report signs and symptoms of hepatitis immediately (which include yellow skin and sclera). For the same reason, alcohol should be avoided during therapy. The client should avoid intake of Swiss cheese, fish such as tuna, and foods containing tyramine because they may cause a reaction characterized by redness and itching of the skin, flushing, sweating, tachycardia, headache, or lightheadedness. The client can avoid developing peripheral neuritis by increasing the intake of pyridoxine (vitamin B6) during the course of INH therapy for TB.

23.) A client who has been newly diagnosed with diabetes mellitus has been stabilized with daily insulin injections. Which information should the nurse teach when carrying out plans for discharge? 1. Keep insulin vials refrigerated at all times. 2. Rotate the insulin injection sites systematically. 3. Increase the amount of insulin before unusual exercise. 4. Monitor the urine acetone level to determine the insulin dosage.

2. Rotate the insulin injection sites systematically. Rationale: Insulin dosages should not be adjusted or increased before unusual exercise. If acetone is found in the urine, it may possibly indicate the need for additional insulin. To minimize the discomfort associated with insulin injections, the insulin should be administered at room temperature. Injection sites should be systematically rotated from one area to another. The client should be instructed to give injections in one area, about 1 inch apart, until the whole area has been used and then to change to another site. This prevents dramatic changes in daily insulin absorption.

687. A nurse has an order to give a client salmeterol (Serevent Diskus), two puffs, and beclomethasone dipropionate (Qvar), two puffs, by metered-dose inhaler. The nurse administers the medication by giving the: 1. Beclomethasone first and then the salmeterol 2. Salmeterol first and then the beclomethasone 3. Alternating a single puff of each, beginning with the salmeterol 4. Alternating a single puff of each, beginning with the beclomethasone

2. Salmeterol first and then the beclomethasone

114.) A postoperative client requests medication for flatulence (gas pains). Which medication from the following PRN list should the nurse administer to this client? 1. Ondansetron (Zofran) 2. Simethicone (Mylicon) 3. Acetaminophen (Tylenol) 4. Magnesium hydroxide (milk of magnesia, MOM)

2. Simethicone (Mylicon) Rationale: Simethicone is an antiflatulent used in the relief of pain caused by excessive gas in the gastrointestinal tract. Ondansetron is used to treat postoperative nausea and vomiting. Acetaminophen is a nonopioid analgesic. Magnesium hydroxide is an antacid and laxative.

45.) A client has a prescription to take guaifenesin (Humibid) every 4 hours, as needed. The nurse determines that the client understands the most effective use of this medication if the client states that he or she will: 1. Watch for irritability as a side effect. 2. Take the tablet with a full glass of water. 3. Take an extra dose if the cough is accompanied by fever. 4. Crush the sustained-release tablet if immediate relief is needed.

2. Take the tablet with a full glass of water. Rationale: Guaifenesin is an expectorant. It should be taken with a full glass of water to decrease viscosity of secretions. Sustained-release preparations should not be broken open, crushed, or chewed. The medication may occasionally cause dizziness, headache, or drowsiness as side effects. The client should contact the health care provider if the cough lasts longer than 1 week or is accompanied by fever, rash, sore throat, or persistent headache.

214.) A health care provider initiates carbidopa/levodopa (Sinemet) therapy for the client with Parkinson's disease. A few days after the client starts the medication, the client complains of nausea and vomiting. The nurse tells the client that: 1. Taking an antiemetic is the best measure to prevent the nausea. 2. Taking the medication with food will help to prevent the nausea. 3. This is an expected side effect of the medication and will decrease over time. 4. The nausea and vomiting will decrease when the dose of levodopa is stabilized.

2. Taking the medication with food will help to prevent the nausea. Rationale: If carbidopa/levodopa is causing nausea and vomiting, the nurse would tell the client that taking the medication with food will prevent the nausea. Additionally, the client should be instructed not to take the medication with a high-protein meal because the high-protein will affect absorption. Antiemetics from the phenothiazine class should not be used because they block the therapeutic action of dopamine. **eliminate options 3 and 4 because they are comparable or alike**

721. A nurse is monitoring a client who is taking propranolol (Inderal). Which assessment data would indicate a potential serious complication associated with propranolol? 1. The development of complaints of insomnia 2. The development of audible expiratory wheezes 3. A baseline blood pressure of 150/80 mm Hg followed by a blood pressure of 138/72 mm Hg after two doses of the medication 4. A baseline resting heart rate of 88 beats/min followed by a resting heart rate of 72 beats/min after two doses of the medication

2. The development of audible expiratory wheezes

57.) A nurse is monitoring a client who is taking propranolol (Inderal LA). Which data collection finding would indicate a potential serious complication associated with propranolol? 1. The development of complaints of insomnia 2. The development of audible expiratory wheezes 3. A baseline blood pressure of 150/80 mm Hg followed by a blood pressure of 138/72 mm Hg after two doses of the medication 4. A baseline resting heart rate of 88 beats/min followed by a resting heart rate of 72 beats/min after two doses of the medication

2. The development of audible expiratory wheezes Rationale: Audible expiratory wheezes may indicate a serious adverse reaction, bronchospasm. β-Blockers may induce this reaction, particularly in clients with chronic obstructive pulmonary disease or asthma. Normal decreases in blood pressure and heart rate are expected. Insomnia is a frequent mild side effect and should be monitored.

28.) The health care provider (HCP) prescribes exenatide (Byetta) for a client with type 1 diabetes mellitus who takes insulin. The nurse knows that which of the following is the appropriate intervention? 1. The medication is administered within 60 minutes before the morning and evening meal. 2. The medication is withheld and the HCP is called to question the prescription for the client. 3. The client is monitored for gastrointestinal side effects after administration of the medication. 4. The insulin is withdrawn from the Penlet into an insulin syringe to prepare for administration.

2. The medication is withheld and the HCP is called to question the prescription for the client. Rationale: Exenatide (Byetta) is an incretin mimetic used for type 2 diabetes mellitus only. It is not recommended for clients taking insulin. Hence, the nurse should hold the medication and question the HCP regarding this prescription. Although options 1 and 3 are correct statements about the medication, in this situation the medication should not be administered. The medication is packaged in prefilled pens ready for injection without the need for drawing it up into another syringe.

654. The nurse has given instructions to a client who has just been prescribed cholestyramine. Which statement by the client indicates a need for further instructions? 1. "I will continue taking vitamin supplements" 2. "This medication will help lower my cholesterol" 3. "This medication should only be taken with water" 4. "A high-fiber diet is important while taking this medication"

3 Cholestyramine (Questran) is a bile acid sequestrant used to lower the cholesterol level, and client compliance is a problem because of its taste and palatability. The use of flavored products or fruit juices can improve the taste. Some side effects of bile acid sequestrants include constipation and decreased vitamin absorption.

165.) The client has been on treatment for rheumatoid arthritis for 3 weeks. During the administration of etanercept (Enbrel), it is most important for the nurse to assess: 1. The injection site for itching and edema 2. The white blood cell counts and platelet counts 3. Whether the client is experiencing fatigue and joint pain 4. A metallic taste in the mouth and a loss of appetite

2. The white blood cell counts and platelet counts Rationale: Infection and pancytopenia are adverse effects of etanercept (Enbrel). Laboratory studies are performed before and during treatment. The appearance of abnormal white blood cell counts and abnormal platelet counts can alert the nurse to a potential life-threatening infection. Injection site itching is a common occurrence following administration of the medication. In early treatment, residual fatigue and joint pain may still be apparent. A metallic taste and loss of appetite are not common signs of side effects of this medication.

126.) A nurse is caring for a client who has been prescribed furosemide (Lasix) and is monitoring for adverse effects associated with this medication. Which of the following should the nurse recognize as a potential adverse effect Select all that apply. 1. Nausea 2. Tinnitus 3. Hypotension 4. Hypokalemia 5. Photosensitivity 6. Increased urinary frequency

2. Tinnitus 3. Hypotension 4. Hypokalemia Rationale: Furosemide is a loop diuretic; therefore, an expected effect is increased urinary frequency. Nausea is a frequent side effect, not an adverse effect. Photosensitivity is an occasional side effect. Adverse effects include tinnitus (ototoxicity), hypotension, and hypokalemia and occur as a result of sudden volume depletion.

51.) Cycloserine (Seromycin) is added to the medication regimen for a client with tuberculosis. Which of the following would the nurse include in the client-teaching plan regarding this medication? 1. To take the medication before meals 2. To return to the clinic weekly for serum drug-level testing 3. It is not necessary to call the health care provider (HCP) if a skin rash occurs. 4. It is not necessary to restrict alcohol intake with this medication.

2. To return to the clinic weekly for serum drug-level testing Rationale: Cycloserine (Seromycin) is an antitubercular medication that requires weekly serum drug level determinations to monitor for the potential of neurotoxicity. Serum drug levels lower than 30 mcg/mL reduce the incidence of neurotoxicity. The medication must be taken after meals to prevent gastrointestinal irritation. The client must be instructed to notify the HCP if a skin rash or signs of central nervous system toxicity are noted. Alcohol must be avoided because it increases the risk of seizure activity.

7.) Isotretinoin is prescribed for a client with severe acne. Before the administration of this medication, the nurse anticipates that which laboratory test will be prescribed? 1. Platelet count 2. Triglyceride level 3. Complete blood count 4. White blood cell count

2. Triglyceride level Rationale: Isotretinoin can elevate triglyceride levels. Blood triglyceride levels should be measured before treatment and periodically thereafter until the effect on the triglycerides has been evaluated. Options 1, 3, and 4 do not need to be monitored specifically during this treatment.

14.) The client with acute myelocytic leukemia is being treated with busulfan (Myleran). Which laboratory value would the nurse specifically monitor during treatment with this medication? 1. Clotting time 2. Uric acid level 3. Potassium level 4. Blood glucose level

2. Uric acid level Rationale: Busulfan (Myleran) can cause an increase in the uric acid level. Hyperuricemia can produce uric acid nephropathy, renal stones, and acute renal failure. Options 1, 3, and 4 are not specifically related to this medication.

68.) Bethanechol chloride (Urecholine) is prescribed for a client with urinary retention. Which disorder would be a contraindication to the administration of this medication? 1. Gastric atony 2. Urinary strictures 3. Neurogenic atony 4. Gastroesophageal reflux

2. Urinary strictures Rationale: Bethanechol chloride (Urecholine) can be harmful to clients with urinary tract obstruction or weakness of the bladder wall. The medication has the ability to contract the bladder and thereby increase pressure within the urinary tract. Elevation of pressure within the urinary tract could rupture the bladder in clients with these conditions.

55.) A client who is receiving digoxin (Lanoxin) daily has a serum potassium level of 3.0 mEq/L and is complaining of anorexia. A health care provider prescribes a digoxin level to rule out digoxin toxicity. A nurse checks the results, knowing that which of the following is the therapeutic serum level (range) for digoxin? 1. 3 to 5 ng/mL 2. 0.5 to 2 ng/mL 3. 1.2 to 2.8 ng/mL 4. 3.5 to 5.5 ng/mL

2.) 0.5 to 2 ng/mL Rationale: Therapeutic levels for digoxin range from 0.5 to 2 ng/mL. Therefore, options 1, 3, and 4 are incorrect.

TPN total protein nutrition

only regular insulin is adm. IV return containing NPH

Opioid toxicity-what to check first

oxygen saturation

762. The nurse, who is administering bethanechol chloride, is monitoring for cholinergic overdose associated with the medication. The nurse should check the client for which sign of overdose? 1. Dry skin 2. Dry mouth 3. Bradycardia 4. Signs of dehydration

3 Cholinergic overdose of bethanechol chloride produces manifestations of excessive muscarinic stimulation such as salivation, sweating, involuntary urination and defecation, bradycardia, and severe hypotension. Treatment includes supportive measures and the administration of atropine sulfate subcutaneously or intravenously.

614. A daily dose of prednisone is prescribed for a client. The nurse provides instructions to the client regarding administration of the medication and should instruct the client that which time is best to take this medication? 1. At noon 2. At bedtime 3. Early mornings 4. Any time at the same time, each day

3 Corticosteroids (glucocorticoids) should be administered before 9AM. Administration at this time helps minimize adrenal insufficiency and mimics the burst of glucocorticoids released naturally be the adrenal glands each morning. Options 1, 2, and 4 are incorrect.

961. A client receiving tricyclic antidepressants arrives at the mental health clinic. Which observation would indicate that the client is following the medication plan correctly? 1. Client reports not going to work for this past week. 2. Client complains of not being able to "do anything" anymore. 3. Client arrives at the clinic neat and appropriate in appearance. 4. Client reports sleeping 12 hours per night and 3 to 4 hours during the day.

3 Depressed individuals sleep for long periods, are unable to go to work, and feel as if they cannot "do anything." When these clients have had some therapeutic effect from their medication, they report resolution of many of these complaints and exhibit an improvement in their appearance. Options 1, 2, and 4 identify continued depression.

877. The nurse is reviewing the results of serum laboratory studies drawn on a client with acquired immunodeficiency syndrome who is receiving didanosine. The nurse interprets that the client may have the medication discontinued by the health care provider if which elevated result is noted? 1. Serum protein level 2. Blood glucose level 3. Serum amylase level 4. Serum creatinine level

3 Didanosine can cause pancreatitis. A serum amylase level that is increased to 1.5 to 2 times normal may signify pancreatitis in the client with acquired immunodeficiency syndrome and is potentially fatal. The medication may have to be discontinued. The medication is also hepatotoxic and can result in liver failure.

679. A client has a prescription to take guaifenesin. The nurse determines that the client understands the proper administration of this medication if the client states that he or she will perform which action? 1. Take an extra dose if fever develops 2. Take the medication with meals only 3. Take the tablet with a full glass of water 4. Decrease the amount of daily fluid intake

3 Guaifenesin (Mucinex) is an expectorant and should be taken with a full glass of water to decrease the viscosity of secretions. Extra doses should not be taken. The client should contact the health care provider if the cough lasts longer than 1 week or is accompanied by fever, rash, sore throat, or persistent headache. Fluids are needed to decrease the viscosity of secretions. The medication does not have to be taken with meals.

878. The nurse is caring for a postrenal transplantation client taking cyclosporine. The nurse notes an increase in one of the client's vital signs and the client is complaining of a headache. What vital sign is most likely increased? 1. Pulse 2. Respirations 3. Blood pressure 4. Pulse oximetry

3 Hypertension can occur in a client taking cyclosporine (Sandimmune) and, because this client is also complaining of a headache, the blood pressure is the vital sign to be monitored most closely. Other adverse effects include infection, nephrotoxicity, and hirsutism. Options 1, 2, and 4 are unrelated to the use of this medication.

567. The nurse is analyzing the laboratory results of a client with leukemia who has received a regimen of chemotherapy. Which laboratory value would the nurse specifically note as a result of the massive cell destruction that occurred from the chemotherapy? 1. Anemia 2. decreased platelets 3. Increased uric acid level 4. Decreased leukocyte count

3 Hyperuremia is especially common following treatment of leukemias and lyphomas because chemotherapy results in massive cell kill. Although options 1, 2, and 4 also may be noted, an increased uric acid level is related specifically to cell destruction

829. A client with trigeminal neuralgia tells the nurse that acetaminophen is taken daily for the relief of generalized discomfort. Which laboratory value would indicate toxicity associated with the medication? 1. Sodium level of 140 mEq/L 2. Prothrombin time of 12 seconds 3. Direct bilirubin level of 2 mg/dL 4. Platelet count of 400,000 cells/mm3

3 In adults, overdose of acetaminophen causes liver damage. The correct option is an indicator of liver function and is the only option that indicates an abnormal laboratory value. The normal direct bilirubin level is 0 to 0.3 mg/dL. The normal sodium level is 135 to 145 mEq/L. The normal prothrombin time is 10 to 13 seconds. The normal platelet count is 150,000 to 400,000 cells/mm3.

857. The nurse is administering an intravenous dose of methocarbamol to a client with multiple sclerosis. For which side/adverse effects should the nurse monitor? 1. Tachycardia 2. Rapid pulse 3. Bradycardia 4. Hypertension

3 Intravenous administration of methocarbamol can cause hypotension and bradycardia. The nurse needs to monitor for these side/adverse effects. Options 1, 2, and 4 are not effects with administration of this medication.

689. A client with tuberculosis is being started on antituberculosis therapy with isoniazid. Before giving the client the first dose, the nurse should ensure that which baseline study has been completed? 1. Electrolyte levels 2. Coagulation times 3. Liver enzyme levels 4. Serum creatinine level

3 Isoniazid therapy can cause an elevation of hepatic enzyme levels and hepatitis. Therefore, liver enzyme levels are monitored when therapy is initiated and during the first 3 months of therapy. They may be monitored longer in the client who is older than 50 or abuses alcohol. The laboratory tests in options 1, 2, and 4 are not necessary.

565. Megestrol acetate, an antineoplastic medication, is prescribed for a client with metastatic endometrial carcinoma. The nurse reviews the clients history and should contact the health care provider if which diagnosis is documented in the client's history? 1. Gout 2. Asthma 3. Thrombophlebitis 4. Myocardial infarction

3 Megestrol acetate (Megace) suppresses the release of luteinizing hormone from the anterior pituitary by inhibiting pituitary function and regressing tumor size. Megestrol is used with caution if the client has a history of thrombophlebitis. Options 1, 2, and 4 are not contraindications for the medication

796. A miotic medication has been prescribed for the client with glaucoma and the client asks the nurse about the purpose of the medication. Which response should the nurse provide to the client? 1. "The medication will help dilate the eye to prevent pressure from occurring." 2. "The medication will relax the muscles of the eyes and prevent blurred vision." 3. "The medication causes the pupil to constrict and will lower the pressure in the eye." 4. "The medication will help block the responses that are sent to the muscles in the eye."

3 Miotics cause pupillary constriction and are used to treat glaucoma. They lower the intraocular pressure, thereby increasing blood flow to the retina and decreasing retinal damage and loss of vision. Miotics cause a contraction of the ciliary muscle and a widening of the trabecular meshwork. Options 1, 2, and 4 are incorrect.

768. The nurse receives a call from a client concerned about eliminating brown-colored urine after taking nitrofurantoin for a urinary tract infection. The nurse should make which appropriate response? 1. "Discontinue taking the medication and make an appointment for a urine culture." 2. "Decrease your medication to half the dose because your urine is too concentrated." 3. "Continue taking the medication because the urine is discolored from the medication." 4. "Take magnesium hydroxide (Maalox) with your medication to lighten the urine color."

3 Nitrofurantoin (Furadantin) imparts a harmless brown color to the urine and the medication should not be discontinued until the prescribed dose is completed. Magnesium hydroxide (Maalox) will not affect urine color. In addition, antacids should be avoided because they interfere with medication effectiveness.

820. The home health nurse visits a client who is taking phenytoin for control of seizures. During the assessment, the nurse notes that the client is taking birth control pills. Which information should the nurse include in the teaching plan? 1. Pregnancy should be avoided while taking phenytoin. 2. The client may stop the medication if it is causing severe gastrointestinal effects. 3. There is the potential of decreased effectiveness of birth control pills while taking phenytoin. 4. There is the increased risk of thrombophlebitis while taking phenytoin and birth control pills together.

3 Phenytoin enhances the rate of estrogen metabolism, which can decrease the effectiveness of some birth control pills. Options 1, 2, and 4 are inappropriate instructions.

687. A client has been started on long-term therapy with rifampin. The nurse should provide which information to the client about the medication? 1. Should always be taken with food or antacids 2. Should be double-dosed if one dose is forgotten 3. Causes orange discoloration of sweat, tears, urine, and feces 4. May be discontinued independently if symptoms are gone in 3 months

3 Rifampin (Rifadin) should be taken exactly as directed. Doses should not be doubled or skipped. The client should not stop therapy until directed to do so by a health care provider. The medication should be administered on an empty stomach unless it causes gastrointestinal upset, and then it may be taken with food. Antacids, if prescribed, should be taken at least 1 hour before the medication. Rifampin causes orange-red discoloration of body secretions and will stain soft contact lenses permanently.

603. Sildenafil is prescribed to treat a client with erectile dysfunction. The nurse reviews the client's medical record and should question the prescription if which data is noted in the client's history? 1. Insomnia 2. Neuralgia 3. Use of nitroglycerin 4. Use of multivitamins

3 Sildenafil (Viagra) enhances the vasodilating effects of nitric oxide in the corpus cavernosum of the penis, thus sustaining an erection. Because of the effect of the medication, it is contraindicated with concurrent use of organic nitrates and nitroglycerin. Sildenafil is not contraindicated with the use of vitamins. Insomnia and neuralgia are side effects of the medication.

683. Terbutaline is prescribed for a client with bronchitis. The nurse understands that this medication should be used with caution if which medical condition is present in the client? 1. Osteoarthritis 2. Hypothyroidism 3. Diabetes mellitus 4. Polycystic disease

3 Terbutaline is a bronchodilator and is contraindicated in clients with hypersensitivity to sympathomimetics. It should be used with caution in clients with impaired cardiac function, diabetes mellitus, hypertension, hyperthyroidism, or a history of seizures. The medication may increase blood glucose levels.

733. A client receiving thrombolytic therapy with a continuous infusion of alteplase suddenly becomes extremely anxious and complains of itching. The nurse hears stridor and notes generalized urticaria and hypotension. Which nursing action is the priority? 1. Administer oxygen and protamine sulfate. 2. Cut the infusion rate in half and sit the client up in bed. 3. Stop the infusion and call the health care provider (HCP). 4. Administer diphenhydramine (Benadryl) and continue the infusion.

3 The client is experiencing an anaphylactic reaction. Therefore, the priority action is to stop the infusion and notify the HCP. The client may be treated with epinephrine, antihistamines, and corticosteroids as prescribed.

823. A client is taking the prescribed dose of phenytoin to control seizures. Results of a phenytoin blood level study reveal a level of 35 mcg/mL. Which finding would be expected as a result of this laboratory result? 1. Hypotension 2. Tachycardia 3. Slurred speech 4. No abnormal finding

3 The therapeutic phenytoin level is 10 to 20 mcg/mL. At a level higher than 20 mcg/mL, involuntary movements of the eyeballs (nystagmus) occur. At a level higher than 30 mcg/mL, ataxia and slurred speech occur.

727. The nurse is planning to administer hydrochlorothiazide to a client. The nurse understands that which is a concern related to the administration of this medication? 1. Hypouricemia, hyperkalemia 2. Increased risk of osteoporosis 3. Hypokalemia, hyperglycemia, sulfa allergy 4. Hyperkalemia, hypoglycemia, penicillin allergy

3 Thiazide diuretics such as hydrochlorothiazide are sulfa-based medications, and a client with a sulfa allergy is at risk for an allergic reaction. Also, clients are at risk for hypokalemia, hyperglycemia, hypercalcemia, hyperlipidemia, and hyperuricemia.

128.) A nurse is providing instructions to an adolescent who has a history of seizures and is taking an anticonvulsant medication. Which of the following statements indicates that the client understands the instructions? 1. "I will never be able to drive a car." 2. "My anticonvulsant medication will clear up my skin." 3. "I can't drink alcohol while I am taking my medication." 4. "If I forget my morning medication, I can take two pills at bedtime."

3. "I can't drink alcohol while I am taking my medication." Rationale: Alcohol will lower the seizure threshold and should be avoided. Adolescents can obtain a driver's license in most states when they have been seizure free for 1 year. Anticonvulsants cause acne and oily skin; therefore a dermatologist may need to be consulted. If an anticonvulsant medication is missed, the health care provider should be notified.

106.) Fluoxetine (Prozac) is prescribed for the client. The nurse reinforces instructions to the client regarding the administration of the medication. Which statement by the client indicates an understanding about administration of the medication? 1. "I should take the medication with my evening meal." 2. "I should take the medication at noon with an antacid." 3. "I should take the medication in the morning when I first arise." 4. "I should take the medication right before bedtime with a snack."

3. "I should take the medication in the morning when I first arise." Rationale: Fluoxetine hydrochloride is administered in the early morning without consideration to meals. **Eliminate options 1, 2, and 4 because they are comparable or alike and indicate taking the medication with an antacid or food.**

197.) Collagenase (Santyl) is prescribed for a client with a severe burn to the hand. The nurse provides instructions to the client regarding the use of the medication. Which statement by the client indicates an accurate understanding of the use of this medication? 1. "I will apply the ointment once a day and leave it open to the air." 2. "I will apply the ointment twice a day and leave it open to the air." 3. "I will apply the ointment once a day and cover it with a sterile dressing." 4. "I will apply the ointment at bedtime and in the morning and cover it with a sterile dressing."

3. "I will apply the ointment once a day and cover it with a sterile dressing." Rationale: Collagenase is used to promote debridement of dermal lesions and severe burns. It is usually applied once daily and covered with a sterile dressing.

11.) The health care provider has prescribed silver sulfadiazine (Silvadene) for the client with a partial-thickness burn, which has cultured positive for gram-negative bacteria. The nurse is reinforcing information to the client about the medication. Which statement made by the client indicates a lack of understanding about the treatments? 1. "The medication is an antibacterial." 2. "The medication will help heal the burn." 3. "The medication will permanently stain my skin." 4. "The medication should be applied directly to the wound."

3. "The medication will permanently stain my skin." Rationale: Silver sulfadiazine (Silvadene) is an antibacterial that has a broad spectrum of activity against gram-negative bacteria, gram-positive bacteria, and yeast. It is applied directly to the wound to assist in healing. It does not stain the skin.

42.) A client with a peptic ulcer is diagnosed with a Helicobacter pylori infection. The nurse is reinforcing teaching for the client about the medications prescribed, including clarithromycin (Biaxin), esomeprazole (Nexium), and amoxicillin (Amoxil). Which statement by the client indicates the best understanding of the medication regimen? 1. "My ulcer will heal because these medications will kill the bacteria." 2. "These medications are only taken when I have pain from my ulcer." 3. "The medications will kill the bacteria and stop the acid production." 4. "These medications will coat the ulcer and decrease the acid production in my stomach."

3. "The medications will kill the bacteria and stop the acid production." Rationale: Triple therapy for Helicobacter pylori infection usually includes two antibacterial drugs and a proton pump inhibitor. Clarithromycin and amoxicillin are antibacterials. Esomeprazole is a proton pump inhibitor. These medications will kill the bacteria and decrease acid production.

141.) The nurse has reinforced instructions to a client who has been prescribed cholestyramine (Questran). Which statement by the client indicates a need for further instructions? 1. "I will continue taking vitamin supplements." 2. "This medication will help lower my cholesterol." 3. "This medication should only be taken with water." 4. "A high-fiber diet is important while taking this medication."

3. "This medication should only be taken with water." Rationale: Cholestyramine (Questran) is a bile acid sequestrant used to lower the cholesterol level, and client compliance is a problem because of its taste and palatability. The use of flavored products or fruit juices can improve the taste. Some side effects of bile acid sequestrants include constipation and decreased vitamin absorption. **Note the closed-ended word "only" in option 3**

191.) A child is brought to the emergency department for treatment of an acute asthma attack. The nurse prepares to administer which of the following medications first? 1. Oral corticosteroids 2. A leukotriene modifier 3. A β2 agonist 4. A nonsteroidal anti-inflammatory

3. A β2 agonist Rationale: In treating an acute asthma attack, a short acting β2 agonist such as albuterol (Proventil HFA) will be given to produce bronchodilation. Options 1, 2, and 4 are long-term control (preventive) medications.

120.) A client is taking lansoprazole (Prevacid) for the chronic management of Zollinger-Ellison syndrome. The nurse advises the client to take which of the following products if needed for a headache? 1. Naprosyn (Aleve) 2. Ibuprofen (Advil) 3. Acetaminophen (Tylenol) 4. Acetylsalicylic acid (aspirin)

3. Acetaminophen (Tylenol) Rationale: Zollinger-Ellison syndrome is a hypersecretory condition of the stomach. The client should avoid taking medications that are irritating to the stomach lining. Irritants would include aspirin and nonsteroidal antiinflammatory drugs (ibuprofen). The client should be advised to take acetaminophen for headache. **Remember that options that are comparable or alike are not likely to be correct. With this in mind, eliminate options 1 and 2 first.**

233.) Diphenhydramine hydrochloride (Benadryl) is used in the treatment of allergic rhinitis for a hospitalized client with a chronic psychotic disorder. The client asks the nurse why the medication is being discontinued before hospital discharge. The nurse responds, knowing that: 1. Allergic symptoms are short in duration. 2. This medication promotes long-term extrapyramidal symptoms. 3. Addictive properties are enhanced in the presence of psychotropic medications. 4. Poor compliance causes this medication to fail to reach its therapeutic blood level.

3. Addictive properties are enhanced in the presence of psychotropic medications. Rationale: The addictive properties of diphenhydramine hydrochloride are enhanced when used with psychotropic medications. Allergic symptoms may not be short term and will occur if allergens are present in the environment. Poor compliance may be a problem with psychotic clients but is not the subject of the question. Diphenhydramine hydrochloride may be used for extrapyramidal symptoms and mild medication-induced movement disorders.

35.) The client has a PRN prescription for loperamide hydrochloride (Imodium). The nurse understands that this medication is used for which condition? 1. Constipation 2. Abdominal pain 3. An episode of diarrhea 4. Hematest-positive nasogastric tube drainage

3. An episode of diarrhea Rationale: Loperamide is an antidiarrheal agent. It is used to manage acute and also chronic diarrhea in conditions such as inflammatory bowel disease. Loperamide also can be used to reduce the volume of drainage from an ileostomy. It is not used for the conditions in options 1, 2, and 4.

207.) A client is suspected of having myasthenia gravis, and the health care provider administers edrophonium (Enlon) to determine the diagnosis. After administration of this medication, which of the following would indicate the presence of myasthenia gravis? 1. Joint pain 2. A decrease in muscle strength 3. An increase in muscle strength 4. Feelings of faintness, dizziness, hypotension, and signs of flushing in the client

3. An increase in muscle strength Rationale: Edrophonium is a short-acting acetylcholinesterase inhibitor used as a diagnostic agent. When a client with suspected myasthenia gravis is given the medication intravenously, an increase in muscle strength would be seen in 1 to 3 minutes. If no response occurs, another dose is given over the next 2 minutes, and muscle strength is again tested. If no increase in muscle strength occurs with this higher dose, the muscle weakness is not caused by myasthenia gravis. Clients receiving injections of this medication commonly demonstrate a drop of blood pressure, feel faint and dizzy, and are flushed.

139.) Prednisone is prescribed for a client with diabetes mellitus who is taking Humulin neutral protamine Hagedorn (NPH) insulin daily. Which of the following prescription changes does the nurse anticipate during therapy with the prednisone? 1. An additional dose of prednisone daily 2. A decreased amount of daily Humulin NPH insulin 3. An increased amount of daily Humulin NPH insulin 4. The addition of an oral hypoglycemic medication daily

3. An increased amount of daily Humulin NPH insulin Rationale: Glucocorticoids can elevate blood glucose levels. Clients with diabetes mellitus may need their dosages of insulin or oral hypoglycemic medications increased during glucocorticoid therapy. Therefore the other options are incorrect.

107.) A client receiving a tricyclic antidepressant arrives at the mental health clinic. Which observation indicates that the client is correctly following the medication plan? 1. Reports not going to work for this past week 2. Complains of not being able to "do anything" anymore 3. Arrives at the clinic neat and appropriate in appearance 4. Reports sleeping 12 hours per night and 3 to 4 hours during the day

3. Arrives at the clinic neat and appropriate in appearance Rationale: Depressed individuals will sleep for long periods, are not able to go to work, and feel as if they cannot "do anything." Once they have had some therapeutic effect from their medication, they will report resolution of many of these complaints as well as demonstrate an improvement in their appearance.

174.) A client with portosystemic encephalopathy is receiving oral lactulose (Chronulac) daily. The nurse assesses which of the following to determine medication effectiveness? 1. Lung sounds 2. Blood pressure 3. Blood ammonia level 4. Serum potassium level

3. Blood ammonia level Rationale: Lactulose is a hyperosmotic laxative and ammonia detoxicant. It is used to prevent or treat portosystemic encephalopathy, including hepatic precoma and coma. It also is used to treat constipation. The medication retains ammonia in the colon (decreases the blood ammonia concentration), producing an osmotic effect. It promotes increased peristalsis and bowel evacuation, expelling ammonia from the colon.

96.) The nurse is caring for a postrenal transplant client taking cyclosporine (Sandimmune, Gengraf, Neoral). The nurse notes an increase in one of the client's vital signs, and the client is complaining of a headache. What is the vital sign that is most likely increased? 1. Pulse 2. Respirations 3. Blood pressure 4. Pulse oximetry

3. Blood pressure Rationale: Hypertension can occur in a client taking cyclosporine (Sandimmune, Gengraf, Neoral), and because this client is also complaining of a headache, the blood pressure is the vital sign to be monitoring most closely. Other adverse effects include infection, nephrotoxicity, and hirsutism. Options 1, 2, and 4 are unrelated to the use of this medication.

153.) A client is diagnosed with pulmonary embolism and is to be treated with streptokinase (Streptase). A nurse would report which priority data collection finding to the registered nurse before initiating this therapy? 1. Adventitious breath sounds 2. Temperature of 99.4° F orally 3. Blood pressure of 198/110 mm Hg 4. Respiratory rate of 28 breaths/min

3. Blood pressure of 198/110 mm Hg Rationale: Thrombolytic therapy is contraindicated in a number of preexisting conditions in which there is a risk of uncontrolled bleeding, similar to the case in anticoagulant therapy. Thrombolytic therapy also is contraindicated in severe uncontrolled hypertension because of the risk of cerebral hemorrhage. Therefore the nurse would report the results of the blood pressure to the registered nurse before initiating therapy. The findings in options 1, 2, and 4 may be present in the client with pulmonary embolism.

69.) A nurse who is administering bethanechol chloride (Urecholine) is monitoring for acute toxicity associated with the medication. The nurse checks the client for which sign of toxicity? 1. Dry skin 2. Dry mouth 3. Bradycardia 4. Signs of dehydration

3. Bradycardia Rationale: Toxicity (overdose) produces manifestations of excessive muscarinic stimulation such as salivation, sweating, involuntary urination and defecation, bradycardia, and severe hypotension. Treatment includes supportive measures and the administration of atropine sulfate subcutaneously or intravenously.

1) A nurse is caring for a client with hyperparathyroidism and notes that the client's serum calcium level is 13 mg/dL. Which medication should the nurse prepare to administer as prescribed to the client? 1. Calcium chloride 2. Calcium gluconate 3. Calcitonin (Miacalcin) 4. Large doses of vitamin D

3. Calcitonin (Miacalcin) Rationale: The normal serum calcium level is 8.6 to 10.0 mg/dL. This client is experiencing hypercalcemia. Calcium gluconate and calcium chloride are medications used for the treatment of tetany, which occurs as a result of acute hypocalcemia. In hypercalcemia, large doses of vitamin D need to be avoided. Calcitonin, a thyroid hormone, decreases the plasma calcium level by inhibiting bone resorption and lowering the serum calcium concentration.

49.) A client has been started on long-term therapy with rifampin (Rifadin). A nurse teaches the client that the medication: 1. Should always be taken with food or antacids 2. Should be double-dosed if one dose is forgotten 3. Causes orange discoloration of sweat, tears, urine, and feces 4. May be discontinued independently if symptoms are gone in 3 months

3. Causes orange discoloration of sweat, tears, urine, and feces Rationale: Rifampin should be taken exactly as directed as part of TB therapy. Doses should not be doubled or skipped. The client should not stop therapy until directed to do so by a health care provider. The medication should be administered on an empty stomach unless it causes gastrointestinal upset, and then it may be taken with food. Antacids, if prescribed, should be taken at least 1 hour before the medication. Rifampin causes orange-red discoloration of body secretions and will permanently stain soft contact lenses.

38.) An older client recently has been taking cimetidine (Tagamet). The nurse monitors the client for which most frequent central nervous system side effect of this medication? 1. Tremors 2. Dizziness 3. Confusion 4. Hallucinations

3. Confusion Rationale: Cimetidine is a histamine 2 (H2)-receptor antagonist. Older clients are especially susceptible to central nervous system side effects of cimetidine. The most frequent of these is confusion. Less common central nervous system side effects include headache, dizziness, drowsiness, and hallucinations.

16.) The clinic nurse is reviewing a teaching plan for the client receiving an antineoplastic medication. When implementing the plan, the nurse tells the client: 1. To take aspirin (acetylsalicylic acid) as needed for headache 2. Drink beverages containing alcohol in moderate amounts each evening 3. Consult with health care providers (HCPs) before receiving immunizations 4. That it is not necessary to consult HCPs before receiving a flu vaccine at the local health fair

3. Consult with health care providers (HCPs) before receiving immunizations Rationale: Because antineoplastic medications lower the resistance of the body, clients must be informed not to receive immunizations without a HCP's approval. Clients also need to avoid contact with individuals who have recently received a live virus vaccine. Clients need to avoid aspirin and aspirin-containing products to minimize the risk of bleeding, and they need to avoid alcohol to minimize the risk of toxicity and side effects.

680. Terbutaline (Brethine) is prescribed for a client with bronchitis. A nurse understands that this medication should be used with caution if which of the following medical conditions is present in the client? 1. Osteoarthritis 2. Hypothyroidism 3. Diabetes mellitus 4. Polycystic disease

3. Diabetes mellitus

727. A 66-year-old client complaining of not feeling well is seen in a clinic. The client is taking several medications for the control of heart disease and hypertension. These medications include atenolol (Tenormin), digoxin (Lanoxin), and chlorothiazide (Diuril). A tentative diagnosis of digoxin toxicity is made. Which of the following assessment data would support this diagnosis? 1. Dyspnea, edema, and palpitations 2. Chest pain, hypotension, and paresthesia 3. Double vision, loss of appetite, and nausea 4. Constipation, dry mouth, and sleep disorder

3. Double vision, loss of appetite, and nausea

150.) A client complaining of not feeling well is seen in a clinic. The client is taking several medications for the control of heart disease and hypertension. These medications include a β-blocker, digoxin (Lanoxin), and a diuretic. A tentative diagnosis of digoxin toxicity is made. Which of the following assessment data would support this diagnosis? 1. Dyspnea, edema, and palpitations 2. Chest pain, hypotension, and paresthesia 3. Double vision, loss of appetite, and nausea 4. Constipation, dry mouth, and sleep disorder

3. Double vision, loss of appetite, and nausea Rationale: Double vision, loss of appetite, and nausea are signs of digoxin toxicity. Additional signs of digoxin toxicity include bradycardia, difficulty reading, visual alterations such as green and yellow vision or seeing spots or halos, confusion, vomiting, diarrhea, decreased libido, and impotence. **gastrointestinal (GI) and visual disturbances occur with digoxin toxicity**

148.) A client is taking cetirizine hydrochloride (Zyrtec). The nurse checks for which of the following side effects of this medication? 1. Diarrhea 2. Excitability 3. Drowsiness 4. Excess salivation

3. Drowsiness Rationale: A frequent side effect of cetirizine hydrochloride (Zyrtec), an antihistamine, is drowsiness or sedation. Others include blurred vision, hypertension (and sometimes hypotension), dry mouth, constipation, urinary retention, and sweating.

85.) A nurse is monitoring a client receiving baclofen (Lioresal) for side effects related to the medication. Which of the following would indicate that the client is experiencing a side effect? 1. Polyuria 2. Diarrhea 3. Drowsiness 4. Muscular excitability

3. Drowsiness Rationale: Baclofen is a central nervous system (CNS) depressant and frequently causes drowsiness, dizziness, weakness, and fatigue. It can also cause nausea, constipation, and urinary retention. Clients should be warned about the possible reactions. Options 1, 2, and 4 are not side effects.

138.) A daily dose of prednisone is prescribed for a client. A nurse reinforces instructions to the client regarding administration of the medication and instructs the client that the best time to take this medication is: 1. At noon 2. At bedtime 3. Early morning 4. Anytime, at the same time, each day

3. Early morning Rationale: Corticosteroids (glucocorticoids) should be administered before 9:00 AM. Administration at this time helps minimize adrenal insufficiency and mimics the burst of glucocorticoids released naturally by the adrenal glands each morning. **Note the suffix "-sone," and recall that medication names that end with these letters are corticosteroids.**

244.) A client has a prescription for valproic acid (Depakene) orally once daily. The nurse plans to: 1. Administer the medication with an antacid. 2. Administer the medication with a carbonated beverage. 3. Ensure that the medication is administered at the same time each day. 4. Ensure that the medication is administered 2 hours before breakfast only, when the client's stomach is empty.

3. Ensure that the medication is administered at the same time each day. Rationale: Valproic acid is an anticonvulsant, antimanic, and antimigraine medication. It may be administered with or without food. It should not be taken with an antacid or carbonated beverage because these products will affect medication absorption. The medication is administered at the same time each day to maintain therapeutic serum levels. **Use general pharmacology guidelines to assist in eliminating options 1 and 2. Eliminate option 4 because of the closed-ended word "only."**

97.) Amikacin (Amikin) is prescribed for a client with a bacterial infection. The client is instructed to contact the health care provider (HCP) immediately if which of the following occurs? 1. Nausea 2. Lethargy 3. Hearing loss 4. Muscle aches

3. Hearing loss Rationale: Amikacin (Amikin) is an aminoglycoside. Adverse effects of aminoglycosides include ototoxicity (hearing problems), confusion, disorientation, gastrointestinal irritation, palpitations, blood pressure changes, nephrotoxicity, and hypersensitivity. The nurse instructs the client to report hearing loss to the HCP immediately. Lethargy and muscle aches are not associated with the use of this medication. It is not necessary to contact the HCP immediately if nausea occurs. If nausea persists or results in vomiting, the HCP should be notified. **(most aminoglycoside medication names end in the letters -cin)**

130.) The nurse is analyzing the laboratory results of a client with leukemia who has received a regimen of chemotherapy. Which laboratory value would the nurse specifically note as a result of the massive cell destruction that occurred from the chemotherapy? 1. Anemia 2. Decreased platelets 3. Increased uric acid level 4. Decreased leukocyte count

3. Increased uric acid level Rationale: Hyperuricemia is especially common following treatment for leukemias and lymphomas because chemotherapy results in a massive cell kill. Although options 1, 2, and 4 also may be noted, an increased uric acid level is related specifically to cell destruction.

122.) A client who has begun taking fosinopril (Monopril) is very distressed, telling the nurse that he cannot taste food normally since beginning the medication 2 weeks ago. The nurse provides the best support to the client by: 1. Telling the client not to take the medication with food 2. Suggesting that the client taper the dose until taste returns to normal 3. Informing the client that impaired taste is expected and generally disappears in 2 to 3 months 4. Requesting that the health care provider (HCP) change the prescription to another brand of angiotensin-converting enzyme (ACE) inhibitor

3. Informing the client that impaired taste is expected and generally disappears in 2 to 3 months Rationale: ACE inhibitors, such as fosinopril, cause temporary impairment of taste (dysgeusia). The nurse can tell the client that this effect usually disappears in 2 to 3 months, even with continued therapy, and provide nutritional counseling if appropriate to avoid weight loss. Options 1, 2, and 4 are inappropriate actions. Taking this medication with or without food does not affect absorption and action. The dosage should never be tapered without HCP approval and the medication should never be stopped abruptly.

230.) A client is placed on chloral hydrate (Somnote) for short-term treatment. Which nursing action indicates an understanding of the major side effect of this medication? 1. Monitoring neurological signs every 2 hours 2. Monitoring the blood pressure every 4 hours 3. Instructing the client to call for ambulation assistance 4. Lowering the bed and clearing a path to the bathroom at bedtime

3. Instructing the client to call for ambulation assistance Rationale: Chloral hydrate (a sedative-hypnotic) causes sedation and impairment of motor coordination; therefore, safety measures need to be implemented. The client is instructed to call for assistance with ambulation. Options 1 and 2 are not specifically associated with the use of this medication. Although option 4 is an appropriate nursing intervention, it is most important to instruct the client to call for assistance with ambulation.

210.) Dantrolene (Dantrium) is prescribed for a client with a spinal cord injury for discomfort resulting from spasticity. The nurse tells the client about the importance of follow-up and the need for which blood study? 1. Creatinine level 2. Sedimentation rate 3. Liver function studies 4. White blood cell count

3. Liver function studies Rationale: Dantrolene can cause liver damage, and the nurse should monitor liver function studies. Baseline liver function studies are done before therapy starts, and regular liver function studies are performed throughout therapy. Dantrolene is discontinued if no relief of spasticity is achieved in 6 weeks.

59.) A client is diagnosed with an acute myocardial infarction and is receiving tissue plasminogen activator, alteplase (Activase, tPA). Which action is a priority nursing intervention? 1. Monitor for renal failure. 2. Monitor psychosocial status. 3. Monitor for signs of bleeding. 4. Have heparin sodium available.

3. Monitor for signs of bleeding. Rationale: Tissue plasminogen activator is a thrombolytic. Hemorrhage is a complication of any type of thrombolytic medication. The client is monitored for bleeding. Monitoring for renal failure and monitoring the client's psychosocial status are important but are not the most critical interventions. Heparin is given after thrombolytic therapy, but the question is not asking about follow-up medications.

728. A client is being treated for acute congestive heart failure with intravenously administered bumetanide (Bumex). The vital signs are as follows: blood pressure, 100/60 mm Hg; pulse, 96 beats/min; and respirations, 24 breaths/min. After the initial dose, which of the following is the priority assessment? 1. Monitoring weight loss 2. Monitoring urine output 3. Monitoring blood pressure 4. Monitoring potassium level

3. Monitoring blood pressure

151.) A client is being treated for acute congestive heart failure with intravenously administered bumetanide. The vital signs are as follows: blood pressure, 100/60 mm Hg; pulse, 96 beats/min; and respirations, 24 breaths/min. After the initial dose, which of the following is the priority assessment? 1. Monitoring weight loss 2. Monitoring temperature 3. Monitoring blood pressure 4. Monitoring potassium level

3. Monitoring blood pressure Rationale: Bumetanide is a loop diuretic. Hypotension is a common side effect associated with the use of this medication. The other options also require assessment but are not the priority. **priority ABCs—airway, breathing, and circulation**

Sumatriptan (treats migraine headaches) adverse effect

pain, tightness, pressure, or heaviness in the chest, throat, neck, and/or jaw slow or difficult speech

201.) A nurse is preparing to administer eardrops to an infant. The nurse plans to: 1. Pull up and back on the ear and direct the solution onto the eardrum. 2. Pull down and back on the ear and direct the solution onto the eardrum. 3. Pull down and back on the ear and direct the solution toward the wall of the canal. 4. Pull up and back on the ear lobe and direct the solution toward the wall of the canal.

3. Pull down and back on the ear and direct the solution toward the wall of the canal. Rationale: When administering eardrops to an infant, the nurse pulls the ear down and straight back. In the adult or a child older than 3 years, the ear is pulled up and back to straighten the auditory canal. The medication is administered by aiming it at the wall of the canal rather than directly onto the eardrum.

37.) The client has begun medication therapy with pancrelipase (Pancrease MT). The nurse evaluates that the medication is having the optimal intended benefit if which effect is observed? 1. Weight loss 2. Relief of heartburn 3. Reduction of steatorrhea 4. Absence of abdominal pain

3. Reduction of steatorrhea Rationale: Pancrelipase (Pancrease MT) is a pancreatic enzyme used in clients with pancreatitis as a digestive aid. The medication should reduce the amount of fatty stools (steatorrhea). Another intended effect could be improved nutritional status. It is not used to treat abdominal pain or heartburn. Its use could result in weight gain but should not result in weight loss if it is aiding in digestion.

677. A nurse is preparing to administer a dose of naloxone hydrochloride (Narcan) intravenously to a client with an intravenous opioid overdose. Which supportive medical equipment would the nurse plan to have at the client's bedside if needed? 1. Nasogastric tube 2. Paracentesis tray 3. Resuscitation equipment 4. Central line insertion tray

3. Resuscitation equipment

137.) A nurse is reinforcing instructions for a client regarding intranasal desmopressin acetate (DDAVP). The nurse tells the client that which of the following is a side effect of the medication? 1. Headache 2. Vulval pain 3. Runny nose 4. Flushed skin

3. Runny nose Rationale: Desmopressin administered by the intranasal route can cause a runny or stuffy nose. Headache, vulval pain, and flushed skin are side effects if the medication is administered by the intravenous (IV) route.

95.) The nurse is reviewing the results of serum laboratory studies drawn on a client with acquired immunodeficiency syndrome who is receiving didanosine (Videx). The nurse interprets that the client may have the medication discontinued by the health care provider if which of the following significantly elevated results is noted? 1. Serum protein 2. Blood glucose 3. Serum amylase 4. Serum creatinine

3. Serum amylase Rationale: Didanosine (Videx) can cause pancreatitis. A serum amylase level that is increased 1.5 to 2 times normal may signify pancreatitis in the client with acquired immunodeficiency syndrome and is potentially fatal. The medication may have to be discontinued. The medication is also hepatotoxic and can result in liver failure.

98.) The nurse is assigned to care for a client with cytomegalovirus retinitis and acquired immunodeficiency syndrome who is receiving foscarnet. The nurse should check the latest results of which of the following laboratory studies while the client is taking this medication? 1. CD4 cell count 2. Serum albumin 3. Serum creatinine 4. Lymphocyte count

3. Serum creatinine Rationale: Foscarnet is toxic to the kidneys. Serum creatinine is monitored before therapy, two to three times per week during induction therapy, and at least weekly during maintenance therapy. Foscarnet may also cause decreased levels of calcium, magnesium, phosphorus, and potassium. Thus these levels are also measured with the same frequency.

58.) Isosorbide mononitrate (Imdur) is prescribed for a client with angina pectoris. The client tells the nurse that the medication is causing a chronic headache. The nurse appropriately suggests that the client: 1. Cut the dose in half. 2. Discontinue the medication. 3. Take the medication with food. 4. Contact the health care provider (HCP).

3. Take the medication with food. Rationale: Isosorbide mononitrate is an antianginal medication. Headache is a frequent side effect of isosorbide mononitrate and usually disappears during continued therapy. If a headache occurs during therapy, the client should be instructed to take the medication with food or meals. It is not necessary to contact the HCP unless the headaches persist with therapy. It is not appropriate to instruct the client to discontinue therapy or adjust the dosages.

NCLEX book chapter 59 676. A client has a prescription to take guaifenesin (Mucinex). The nurse determines that the client understands the proper administration of this medication if the client states that he or she will: 1. Take an extra dose if fever develops. 2. Take the medication with meals only. 3. Take the tablet with a full glass of water. 4. Decrease the amount of daily fluid intake.

3. Take the tablet with a full glass of water.

hypokalemia

patients on diuretics will change patients normal ECG = U wave is positive deflection following the T wave often present in hypokalemia

small bowel obstruction

peritonis w/ Temperature of 102 notify HCP immediately abdominal cramping

80.) A nurse is caring for a client who is taking phenytoin (Dilantin) for control of seizures. During data collection, the nurse notes that the client is taking birth control pills. Which of the following information should the nurse provide to the client? 1. Pregnancy should be avoided while taking phenytoin (Dilantin). 2. The client may stop taking the phenytoin (Dilantin) if it is causing severe gastrointestinal effects. 3. The potential for decreased effectiveness of the birth control pills exists while taking phenytoin (Dilantin). 4. The increased risk of thrombophlebitis exists while taking phenytoin (Dilantin) and birth control pills together.

3. The potential for decreased effectiveness of the birth control pills exists while taking phenytoin (Dilantin). Rationale: Phenytoin (Dilantin) enhances the rate of estrogen metabolism, which can decrease the effectiveness of some birth control pills. Options 1, 2, are 4 are not accurate.

817. The home health nurse visits a client who is taking phenytoin (Dilantin) for control of seizures. During the assessment, the nurse notes that the client is taking birth control pills. Which of the following information should the nurse include in the teaching plan? 1. Pregnancy should be avoided while taking phenytoin. 2. The client may stop the medication if it is causing severe gastrointestinal effects. 3. There is the potential of decreased effectiveness of birth control pills while taking phenytoin. 4. There is the increased risk of thrombophlebitis while taking phenytoin and birth control pills together.

3. There is the potential of decreased effectiveness of birth control pills while taking phenytoin.

129.) Megestrol acetate (Megace), an antineoplastic medication, is prescribed for the client with metastatic endometrial carcinoma. The nurse reviews the client's history and contacts the registered nurse if which diagnosis is documented in the client's history? 1. Gout 2. Asthma 3. Thrombophlebitis 4. Myocardial infarction

3. Thrombophlebitis Rationale: Megestrol acetate (Megace) suppresses the release of luteinizing hormone from the anterior pituitary by inhibiting pituitary function and regressing tumor size. Megestrol is used with caution if the client has a history of thrombophlebitis. **megestrol acetate is a hormonal antagonist enzyme and that a side effect is thrombotic disorders**

103.) A nurse is caring for a hospitalized client who has been taking clozapine (Clozaril) for the treatment of a schizophrenic disorder. Which laboratory study prescribed for the client will the nurse specifically review to monitor for an adverse effect associated with the use of this medication? 1. Platelet count 2. Cholesterol level 3. White blood cell count 4. Blood urea nitrogen level

3. White blood cell count Rationale: Hematological reactions can occur in the client taking clozapine and include agranulocytosis and mild leukopenia. The white blood cell count should be checked before initiating treatment and should be monitored closely during the use of this medication. The client should also be monitored for signs indicating agranulocytosis, which may include sore throat, malaise, and fever. Options 1, 2, and 4 are unrelated to this medication.

81.) A client with trigeminal neuralgia is being treated with carbamazepine (Tegretol). Which laboratory result would indicate that the client is experiencing an adverse reaction to the medication? 1. Sodium level, 140 mEq/L 2. Uric acid level, 5.0 mg/dL 3. White blood cell count, 3000 cells/mm3 4. Blood urea nitrogen (BUN) level, 15 mg/dL

3. White blood cell count, 3000 cells/mm3 Rationale: Adverse effects of carbamazepine (Tegretol) appear as blood dyscrasias, including aplastic anemia, agranulocytosis, thrombocytopenia, leukopenia, cardiovascular disturbances, thrombophlebitis, dysrhythmias, and dermatological effects. Options 1, 2, and 4 identify normal laboratory values.

3. A client is taking benzphetamine. The nurse teaches the client which information about this drug? a. That it may cause drowsiness b. That it may lead to hypotension c. That it is a respiratory stimulant d. That it is safe during pregnancy 4. The nurse monitoring a client for methylphenidate withdrawal should observe the client for which condition? a. Tremors b. Insomnia d. Tachycardia

3= b. That it may lead to hypotension 4= c. Weakness

949. A client with schizophrenia has been started on medication therapy with clozapine. The nurse should assess the results of which laboratory study to monitor for adverse effects from this medication? 1. Platelet count 2. Blood glucose level 3. Liver function studies 4. White blood cell count

4 A client taking clozapine (Clozaril) may experience agranulocytosis, which is monitored by reviewing the results of the white blood cell count. Treatment is interrupted if the white blood cell count decreases to less than 3000 cells/mm3. Agranulocytosis could be fatal if undetected and untreated. The other laboratory studies are not related specifically to the use of this medication.

561. A client with ovarian cancer is being treated with vincristine (Vincasar). The nurse monitors the client, knowing that which manifestation indicates an adverse effect specific to this medication? 1. Diarrhea 2. Hair loss 3. Chest pain 4. Peripheral neutropathy

4 An adverse effect specific to vincristine is peripheral neuropathy, which occurs in almost every client. Peripheral neuropathy can be manifested as numbness and tingling in the fingers and toes. Depression of the Achilles tendon reflex may be the first clinical sign indicating peripheral neuropathy. Constipation rather than diarrhea is most likely to occyr with this medication, although diarrhea may occur occasionally. Hair loss occurs with nearly all the antineoplastic medications. Chest pain is unrelated to this medication

828. A client with myasthenia gravis has become increasingly weaker. The health care provider prepares to identify whether the client is reacting to an overdose of the medication (cholinergic crisis) or an increasing severity of the disease (myasthenic crisis). An injection of edrophonium is administered. Which finding would indicate that the client is in cholinergic crisis? 1. No change in the condition 2. Complaints of muscle spasms 3. An improvement of the weakness 4. A temporary worsening of the condition

4 An edrophonium injection makes the client in cholinergic crisis temporarily worse. An improvement in the weakness indicates myasthenia crisis. Muscle spasms are not associated with this test.

730. Prior to administering a client's daily dose of digoxin, the nurse reviews the client's laboratory data and notes the following results: serum calcium, 9.8 mg/dL; serum magnesium, 1.2 mg/dL; serum potassium, 4.1 mEq/L; serum creatinine, 0.9 mg/dL. Which result should alert the nurse that the client is at risk for digoxin toxicity? 1. Serum calcium level 2. Serum potassium level 3. Serum creatinine level 4. Serum magnesium level

4 An increased risk of toxicity exists in clients with hypercalcemia, hypokalemia, hypomagnesemia, hypothyroidism, and impaired renal function. The calcium, creatinine, and potassium levels are all within normal limits. The normal range for magnesium is 1.6 to 2.6 mg/dL and the results in the correct option are reflective of hypomagnesemia.

557. A client with squamous cell carcinoma of the larynx is receiving bleomycin intravenously. The nurse caring for the client anticipates that which diagnostic study will be prescribed? 1. Echocardiography 2. Elecctrocardiography 3. Cervical radiography 4. Pulmonary function studies

4 Bleomycin is an antineoplastic medication that can cause interstitial pneumonia, which can progress to pulmonary fibrosis. Pulmonary function studies along with hematological, hepatic, and renal fucntion tests need to be monitored. The nurse needs to monitor lung sounds for dyspnea and crackles which indicate pulmonary toxicity. The medication need to be discontinued immediately if pulmonary toxicity occurs. Options 1, 2, and 3 are unrelated to the specific use of this medication.

569. A client with non-Hodgkin's lymphoma is receiving daunrubicin. Which finding would indicate to the nurse that the client is experiencing an adverse effect related to the medication? 1. Fever 2. Sores in the mouth and throat 3. Complaints of nausea and vomiting 4. Crackles o auscultation of the lungs

4 Cardiotoxicity noted by abnormal electrocardipgraphic findings or cardiomyopathy manifested as heart failure (lung crackles) is an adverse effect of daunorubicin. Bone marrow depression is also an adverse effect. Fever is a freuqnt side effect and sores in the mouth and throat can occur occasionally. Nausea and comiting is a frequent side effect associated with the medication that beings a few hours after administration and last 24-48 hrs. Option 1, 2, and 3 are not adverse effects.

759. Trimethoprim-sulfamethoxazole is prescribed for a client. The nurse should instruct the client to report which symptom if it develops during the course of this medication therapy? 1. Nausea 2. Diarrhea 3. Headache 4. Sore throat

4 Clients taking trimethoprim (TMP)-sulfamethoxazole (SMZ) should be informed about early signs/symptoms of blood disorders that can occur from this medication. These include sore throat, fever, and pallor, and the client should be instructed to notify the health care provider (HCP) if these occur. The other options do not require HCP notification.

959. A client gives the home health nurse a bottle of clomipramine. The nurse notes that the medication has not been taken by the client in 2 months. Which behavior observed in the client would validate noncompliance with this medication? 1. Complaints of insomnia 2. Complaints of hunger and fatigue 3. A pulse rate less than 60 beats/minute 4. Frequent hand-washing with hot soapy water

4 Clomipramine (Anafranil) is a tricyclic antidepressant used to treat obsessive-compulsive disorder. Sedation sometimes occurs. Insomnia seldom is a side effect. Weight gain and tachycardia are side/adverse effects of this medication.

682. A cromolyn sodium inhaler is prescribed for a client with allergic asthma. The nurse provides instructions regarding the side and adverse effects of this medication and should tell the client that which undesirable effect is associated with this medication? 1. Insomnia 2. Constipation 3. Hypotension 4. Bronchospasm

4 Cromolyn sodium is an inhaled nonsteroidal antiallergy agent and a mast cell stabilizer. Undesirable effects associated with inhalation therapy of cromolyn sodium are bronchospasm, cough, nasal congestion, throat irritation, and wheezing. Clients receiving this medication orally may experience pruritus, nausea, diarrhea, and myalgia.

721. The nurse provides discharge instructions to a client who is taking warfarin sodium. Which statement, by the client, reflects the need for further teaching? 1. "I will avoid alcohol consumption." 2. "I will take my pills every day at the same time." 3. "I have already called my family to pick up a Medic-Alert bracelet." 4. "I will take enteric-coated aspirin for my headaches because it is coated."

4 Ecotrin is an aspirin-containing product and should be avoided. Alcohol consumption should be avoided by a client taking warfarin sodium. Taking prescribed medication at the same time each day increases client compliance. The Medic-Alert bracelet provides health care personnel emergency information.

693. The nurse has just administered the first dose of omalizumab to a client. Which statement by the client would alert the nurse that the client may be experiencing a lifethreatening effect? 1. "I have a severe headache." 2. "My feet are quite swollen." 3. "I am nauseated and may vomit." 4. "My lips and tongue are swollen."

4 Omalizumab (Xolair) is an antiinflammatory used for long-term control of asthma. Anaphylactic reactions can occur with the administration of omalizumab. The nurse administering the medication should monitor for adverse reactions of the medication. Swelling of the lips and tongue are an indication of an adverse reaction. The client statements in options 1, 2, and 3 are not indicative of an adverse reaction.

688. The nurse has given a client taking ethambutol information about the medication. The nurse determines that the client understands the instructions if the client states he or she will immediately report which finding? 1. Impaired sense of hearing 2. Gastrointestinal side effects 3. Orange-red discoloration of body secretions 4. Difficulty in discriminating the color red from green

4 Ethambutol (Myambutol) causes optic neuritis, which decreases visual acuity and the ability to discriminate between the colors red and green. This poses a potential safety hazard when a client is driving a motor vehicle. The client is taught to report this symptom immediately. The client also is taught to take the medication with food if gastrointestinal upset occurs. Impaired hearing results from antitubercular therapy with streptomycin. Orange-red discoloration of secretions occurs with rifampin (Rifadin).

735. A client is prescribed nicotinic acid (niacin) for hyperlipidemia and the nurse provides instructions to the client about the medication. Which statement by the client indicates an understanding of the instructions? 1. "It is not necessary to avoid the use of alcohol." 2. "The medication should be taken with meals to decrease flushing." 3. "Clay-colored stools are a common side effect and should not be of concern." 4. "Ibuprofen (Motrin) taken 30 minutes before the nicotinic acid should decrease the flushing."

4 Flushing is a side effect of this medication. Aspirin or a nonsteroidal antiinflammatory drug can be taken 30 minutes prior to taking the medication to decrease flushing. Alcohol consumption needs to be avoided because it will enhance this side effect. The medication should be taken with meals to decrease gastrointestinal upset; however, taking the medication with meals has no effect on the flushing. Clay-colored stools are a sign of hepatic dysfunction and should be immediately reported to the health care provider (HCP).

873. The client with acquired immunodeficiency syndrome and Pneumocystis jiroveci infection has been receiving pentamidine. The client develops a temperature of 101° F. The nurse continues to assess the client, knowing that this sign most likely indicates which condition? 1. That the dose of the medication is too low 2. That the client is experiencing toxic effects of the medication 3. That the client has developed inadequacy of thermoregulation 4. That the client has developed another infection caused by leukopenic effects of the medication

4 Frequent side/adverse effects of this medication include leukopenia, thrombocytopenia, and anemia. The client should be monitored routinely for signs and symptoms of infection. Options 1, 2, and 3 are inaccurate interpretations.

756. A client who has a cold is seen in the emergency department with an inability to void. Because the client has a history of benign prostatic hyperplasia, the nurse determines that the client should be questioned about the use of which medication? 1. Diuretics 2. Antibiotics 3. Antitussives 4. Decongestants

4 In the client with benign prostatic hyperplasia, episodes of urinary retention can be triggered by certain medications, such as decongestants, anticholinergics, and antidepressants. The client should be questioned about the use of these medications if the client has urinary retention. Retention also can be precipitated by other factors, such as alcoholic beverages, infection, bed rest, and becoming chilled.

365. Methylergonovine is prescribed for a client with postpartum hemorrhage. Before administering the medication, the nurse contacts the health care provider who prescribed the medication if which condition is documented in the client's medical history? 1. Hypotension 2. Hypothyroidism 3. Diabetes mellitus 4. Peripheral vascular disease

4 Methylergonovine is an ergot alkaloid used to treat postpartum hemorrhage. Ergot alkaloids are contraindicated in clients with significant cardiovascular disease, peripheral vascular disease, hypertension, preeclampsia, or eclampsia. These conditions are worsened by the vasoconstrictive effects of the ergot alkaloids. Options 1, 2, and 3 are not contraindications related to the use of ergot alkaloids.

652. A client has a new prescription for metoclopramide. On review of the chart, the nurse identifies that this medication can be safely administered with which condition? 1. Intestinal obstruction 2. Peptic ulcer with melena 3. Diverticulitis with perforation 4. Vomiting following cancer chemotherapy

4 Metoclopramide (Reglan) is a gastrointestinal stimulant and antiemetic. Because it is a GI stimulant, it is contraindicated with GI obstruction, hemorrhage, or perforation. It is used in the treatment of emesis after surgery, chemotherapy, and radiation.

764. Following kidney transplantation, cyclosporine is prescribed for a client. Which laboratory result would indicate an adverse effect from the use of this medication? 1. Normal hemoglobin level 2. Decreased creatinine level 3. Decreased white blood cell count 4. Elevated blood urea nitrogen level

4 Nephrotoxicity can occur from the use of cyclosporine (Sandimmune). Nephrotoxicity is evaluated by monitoring for elevated blood urea nitrogen and serum creatinine levels. Cyclosporine does not depress the bone marrow.

728. The home health care nurse is visiting a client with elevated triglyceride levels and a serum cholesterol level of 398 mg/dL. The client is taking cholestyramine (Questran). Which statement, by the client, indicates the need for further education? 1. "Constipation and bloating might be a problem." 2. "I'll continue to watch my diet and reduce my fats." 3. "Walking a mile each day will help the whole process." 4. "I'll continue my nicotinic acid from the health food store."

4 Nicotinic acid, even an over-the-counter form, should be avoided because it may lead to liver abnormalities. All lipid lowering medications also can cause liver abnormalities, so a combination of nicotinic acid and cholestyramine resin needs to be avoided. Constipation and bloating are the two most common side effects. Walking and the reduction of fats in the diet are therapeutic measures to reduce cholesterol and triglyceride levels.

757. Nitrofurantoin is prescribed for a client with a urinary tract infection. The client contacts the nurse and reports a cough, chills, fever, and difficulty breathing. The nurse should make which interpretation about the client's complaints? 1. The client may have contracted the flu. 2. The client is experiencing anaphylaxis. 3. The client is experiencing expected effects of the medication. 4. The client is experiencing a pulmonary reaction requiring cessation of the medication.

4 Nitrofurantoin can induce two kinds of pulmonary reactions: acute and subacute. Acute reactions, which are most common, manifest with dyspnea, chest pain, chills, fever, cough, and alveolar infiltrates. These symptoms resolve 2 to 4 days after discontinuing the medication. Acute pulmonary responses are thought to be hypersensitivity reactions. Subacute reactions are rare and occur during prolonged treatment. Symptoms (e.g., dyspnea, cough, malaise) usually regress over weeks to months following nitrofurantoin withdrawal. However, in some clients, permanent lung damage may occur. The remaining options are incorrect interpretations.

950. A client is scheduled for discharge and will be taking phenobarbital sodium for an extended period. The nurse would place highest priority on teaching the client which point that directly relates to client safety? 1. Take the medication only with meals. 2. Take the medication at the same time each day. 3. Use a dose container to help prevent missed doses. 4. Avoid drinking alcohol while taking this medication.

4 Phenobarbital is an anticonvulsant and hypnotic agent. The client should avoid taking any other central nervous system depressants such as alcohol while taking this medication. The medication may be given without regard to meals. Taking the medication at the same time each day enhances compliance and maintains more stable blood levels of the medication. Using a dose container or "pillbox" may be helpful for some clients.

852. Alendronate is prescribed for a client with osteoporosis and the nurse is providing instructions on administration of the medication. Which instruction should the nurse provide? 1. Take the medication at bedtime. 2. Take the medication in the morning with breakfast. 3. Lie down for 30 minutes after taking the medication. 4. Take the medication with a full glass of water after rising in the morning.

4 Precautions need to be taken with the administration of alendronate to prevent gastrointestinal side/adverse effects (especially esophageal irritation) and to increase absorption of the medication. The medication needs to be taken with a full glass of water after rising in the morning. The client should not eat or drink anything for 30 minutes following administration and should not lie down after taking the medication.

952. The nurse is administering risperidone to a client who is scheduled to be discharged. Before discharge, which instruction should the nurse provide to the client? 1. Get adequate sunlight. 2. Continue driving as usual. 3. Avoid foods rich in potassium. 4. Get up slowly when changing positions.

4 Risperidone (Risperdal) can cause orthostatic hypotension. Sunlight should be avoided by the client taking this medication. With any psychotropic medication, caution needs to be taken (such as with driving or other activities requiring alertness) until the individual can determine whether his or her level of alertness is affected. Food interaction is not a concern.

723. A client is being treated with procainamide for a cardiac dysrhythmia. Following intravenous administration of the medication, the client complains of dizziness. What intervention should the nurse take first? 1. Measure the heart rate on the rhythm strip. 2. Administer prescribed nitroglycerin tablets. 3. Obtain a 12-lead electrocardiogram immediately. 4. Auscultate the client's apical pulse and obtain a blood pressure.

4 Signs of toxicity from procainamide include confusion, dizziness, drowsiness, decreased urination, nausea, vomiting, and tachydysrhythmias. If the client complains of dizziness, the nurse should assess the vital signs first. Although measuring the heart rate on the rhythm strip and obtaining a 12-lead EKG may be interventions, these would be done after the vital signs are taken. Nitroglycerin is a vasodilator and will lower the blood pressure.

563. Tamoxifen citrate is prescribed for a client with metastatic breast carcinoma. The nurse administering the medication understands that which is the primary action of this medication? 1. It increased DNA and RNA synthesis 2. It promotes the biosynthesis of nucleic acids 3. It increases estrogen concentration and estrogen response 4. It competitively binds to estrogen receptors on tumors and other tissue targets

4 Tamoxifen is an antineoplastic medication tha competes with estradiol for binding to estrogen in tissues containing high concentrations of receptors. Uamoxifen is used to treat metastatic breast carcinoma in women and men. Tamoxifen is also effective in delaying the recurrence of cancer following mastectomy. Tamoxifen reduces DNA synthesis and estrogen response.

821. The nurse is caring for a client in the emergency department who has been diagnosed with Bell's palsy. The client has been taking acetaminophen, and acetaminophen overdose is suspected. Which antidote should the nurse anticipate to be prescribed? 1. Pentostatin 2. Auranofin 3. Fludarabine 4. Acetylcysteine

4 The antidote for acetaminophen is acetylcysteine (Mucomyst). The normal therapeutic serum level of acetaminophen is 10 to 20 mcg/mL. A toxic level is higher than 50 mcg/mL, and levels higher than 200 mcg/mL could indicate hepatotoxicity. Auranofin (Ridaura) is a gold preparation used to treat rheumatoid arthritis. Pentostatin (Nipent) and fludarabine (Fludara) are antineoplastic agents.

760. Phenazopyridine is prescribed for a client for symptomatic relief of pain resulting from a lower urinary tract infection. The nurse should provide the client with which information regarding this medication? 1. Take the medication at bedtime. 2. Take the medication before meals. 3. Discontinue the medication if a headache occurs. 4. A reddish orange discoloration of the urine may occur.

4 The nurse should instruct the client that a reddish-orange discoloration of urine may occur. The nurse also should instruct the client that this discoloration can stain fabric. The medication should be taken after meals to reduce the possibility of gastrointestinal upset. A headache is an occasional side effect of the medication and does not warrant discontinuation of the medication.

763. Oxybutynin chloride is prescribed for a client with neurogenic bladder. Which sign would indicate a possible toxic effect related to this medication? 1. Pallor 2. Drowsiness 3. Bradycardia 4. Restlessness

4 Toxicity (overdosage) of oxybutynin produces central nervous system excitation, such as nervousness, restlessness, hallucinations, and irritability. Other signs of toxicity include hypotension or hypertension, confusion, tachycardia, flushed or red face, and signs of respiratory depression. Drowsiness is a frequent side effect of the medication but does not indicate overdosage.

718. A nurse provides discharge instructions to a postoperative client who is taking warfarin sodium (Coumadin). Which statement, if made by the client, reflects the need for further teaching? 1. "I will take my pills every day at the same time." 2. "I will avoid alcohol consumption." 3. "I have already called my family to pick up a Medic-Alert bracelet." 4. "I will take Ecotrin (enteric-coated aspirin) for my headaches because it is coated."

4. "I will take Ecotrin (enteric-coated aspirin) for my headaches because it is coated."

54.) A nurse reinforces discharge instructions to a postoperative client who is taking warfarin sodium (Coumadin). Which statement, if made by the client, reflects the need for further teaching? 1. "I will take my pills every day at the same time." 2. "I will be certain to avoid alcohol consumption." 3. "I have already called my family to pick up a Medic-Alert bracelet." 4. "I will take Ecotrin (enteric-coated aspirin) for my headaches because it is coated."

4. "I will take Ecotrin (enteric-coated aspirin) for my headaches because it is coated." Rationale: Ecotrin is an aspirin-containing product and should be avoided. Alcohol consumption should be avoided by a client taking warfarin sodium. Taking prescribed medication at the same time each day increases client compliance. The Medic-Alert bracelet provides health care personnel emergency information.

61.) A home health care nurse is visiting a client with elevated triglyceride levels and a serum cholesterol level of 398 mg/dL. The client is taking cholestyramine (Questran). Which of the following statements, if made by the client, indicates the need for further education? 1. "Constipation and bloating might be a problem." 2. "I'll continue to watch my diet and reduce my fats." 3. "Walking a mile each day will help the whole process." 4. "I'll continue my nicotinic acid from the health food store."

4. "I'll continue my nicotinic acid from the health food store." Rationale: Nicotinic acid, even an over-the-counter form, should be avoided because it may lead to liver abnormalities. All lipid-lowering medications also can cause liver abnormalities, so a combination of nicotinic acid and cholestyramine resin is to be avoided. Constipation and bloating are the two most common side effects. Walking and the reduction of fats in the diet are therapeutic measures to reduce cholesterol and triglyceride levels.

218.) A film-coated form of diflunisal has been prescribed for a client for the treatment of chronic rheumatoid arthritis. The client calls the clinic nurse because of difficulty swallowing the tablets. Which initial instruction should the nurse provide to the client? 1. "Crush the tablets and mix them with food." 2. "Notify the health care provider for a medication change." 3. "Open the tablet and mix the contents with food." 4. "Swallow the tablets with large amounts of water or milk."

4. "Swallow the tablets with large amounts of water or milk." Rationale: Diflunisal may be given with water, milk, or meals. The tablets should not be crushed or broken open. Taking the medication with a large amount of water or milk should be tried before contacting the health care provider.

198.) Coal tar has been prescribed for a client with a diagnosis of psoriasis, and the nurse provides instructions to the client about the medication. Which statement by the client indicates a need for further instructions? 1. "The medication can cause phototoxicity." 2. "The medication has an unpleasant odor." 3. "The medication can stain the skin and hair." 4. "The medication can cause systemic effects."

4. "The medication can cause systemic effects." Rationale: Coal tar is used to treat psoriasis and other chronic disorders of the skin. It suppresses DNA synthesis, mitotic activity, and cell proliferation. It has an unpleasant odor, can frequently stain the skin and hair, and can cause phototoxicity. Systemic toxicity does not occur. **The name of the medication will assist in eliminating options 2 and 3**

87.) A client with acute muscle spasms has been taking baclofen (Lioresal). The client calls the clinic nurse because of continuous feelings of weakness and fatigue and asks the nurse about discontinuing the medication. The nurse should make which appropriate response to the client? 1. "You should never stop the medication." 2. "It is best that you taper the dose if you intend to stop the medication." 3. "It is okay to stop the medication if you think that you can tolerate the muscle spasms." 4. "Weakness and fatigue commonly occur and will diminish with continued medication use."

4. "Weakness and fatigue commonly occur and will diminish with continued medication use." Rationale: The client should be instructed that symptoms such as drowsiness, weakness, and fatigue are more intense in the early phase of therapy and diminish with continued medication use. The client should be instructed never to withdraw or stop the medication abruptly, because abrupt withdrawal can cause visual hallucinations, paranoid ideation, and seizures. It is best for the nurse to inform the client that these symptoms will subside and encourage the client to continue the use of the medication.

75.) A client with myasthenia gravis becomes increasingly weak. The health care provider prepares to identify whether the client is reacting to an overdose of the medication (cholinergic crisis) or increasing severity of the disease (myasthenic crisis). An injection of edrophonium (Enlon) is administered. Which of the following indicates that the client is in cholinergic crisis? 1. No change in the condition 2. Complaints of muscle spasms 3. An improvement of the weakness 4. A temporary worsening of the condition

4. A temporary worsening of the condition Rationale: An edrophonium (Enlon) injection, a cholinergic drug, makes the client in cholinergic crisis temporarily worse. This is known as a negative test. An improvement of weakness would occur if the client were experiencing myasthenia gravis. Options 1 and 2 would not occur in either crisis.

818. The nurse is caring for a client in the emergency department diagnosed with Bell's palsy. The client has been taking acetaminophen (Tylenol), and acetaminophen overdose is suspected. The nurse anticipates that the antidote to be prescribed is: 1. Pentostatin (Nipent) 2. Auranofin (Ridaura) 3. Fludarabine (Fludara) 4. Acetylcysteine (Mucomyst)

4. Acetylcysteine (Mucomyst)

722. A nurse is caring for a client receiving a heparin intravenous (IV) infusion. The nurse anticipates that which laboratory study will be prescribed to monitor the therapeutic effect of heparin? 1. Hematocrit 2. Hemoglobin 3. Prothrombin time 4. Activated partial thromboplastin time

4. Activated partial thromboplastin time

56.) Heparin sodium is prescribed for the client. The nurse expects that the health care provider will prescribe which of the following to monitor for a therapeutic effect of the medication? 1. Hematocrit level 2. Hemoglobin level 3. Prothrombin time (PT) 4. Activated partial thromboplastin time (aPTT)

4. Activated partial thromboplastin time (aPTT) Rationale: The PT will assess for the therapeutic effect of warfarin sodium (Coumadin) and the aPTT will assess the therapeutic effect of heparin sodium. Heparin sodium doses are determined based on these laboratory results. The hemoglobin and hematocrit values assess red blood cell concentrations.

242.) A client who was started on anticonvulsant therapy with clonazepam (Klonopin) tells the nurse of increasing clumsiness and unsteadiness since starting the medication. The client is visibly upset by these manifestations and asks the nurse what to do. The nurse's response is based on the understanding that these symptoms: 1. Usually occur if the client takes the medication with food 2. Are probably the result of an interaction with another medication 3. Indicate that the client is experiencing a severe untoward reaction to the medication 4. Are worse during initial therapy and decrease or disappear with long-term use

4. Are worse during initial therapy and decrease or disappear with long-term use Rationale: Drowsiness, unsteadiness, and clumsiness are expected effects of the medication during early therapy. They are dose related and usually diminish or disappear altogether with continued use of the medication. It does not indicate that a severe side effect is occurring. It is also unrelated to interaction with another medication. The client is encouraged to take this medication with food to minimize gastrointestinal upset. **Eliminate options 2 and 3 first because they are comparable or alike and because of the word "severe" in option 3**

720. A client is being treated with procainamide (Procanbid) for a cardiac dysrhythmia. Following intravenous administration of the medication, the client complains of dizziness. What intervention should the nurse take first? 1. Administer prescribed nitroglycerin tablets. 2. Measure the heart rate on the rhythm strip. 3. Obtain a 12-lead electrocardiogram immediately. 4. Auscultate the client's apical pulse and obtain a blood pressure.

4. Auscultate the client's apical pulse and obtain a blood pressure.

946. A client is scheduled for discharge and will be taking phenobarbital (Luminal) for an extended period. A nurse would place highest priority on teaching the client which of the following points that directly relates to client safety? 1. Take the medication only with meals. 2. Take the medication at the same time each day. 3. Use a dose container to help prevent missed doses. 4. Avoid drinking alcohol while taking this medication.

4. Avoid drinking alcohol while taking this medication.

178.) Methylergonovine (Methergine) is prescribed for a client with postpartum hemorrhage caused by uterine atony. Before administering the medication, the nurse checks which of the following as the important client parameter? 1. Temperature 2. Lochial flow 3. Urine output 4. Blood pressure

4. Blood pressure Rationale: Methylergonovine is an ergot alkaloid used for postpartum hemorrhage. It stimulates contraction of the uterus and causes arterial vasoconstriction. Ergot alkaloids are avoided in clients with significant cardiovascular disease, peripheral disease, hypertension, eclampsia, or preeclampsia. These conditions are worsened by the vasoconstrictive effects of the ergot alkaloids. The nurse would check the client's blood pressure before administering the medication and would follow agency protocols regarding withholding of the medication. Options 1, 2, and 3 are items that are checked in the postpartum period, but they are unrelated to the use of this medication.

679. A cromolyn sodium (Intal) inhaler is prescribed for a client with allergic asthma. A nurse provides instructions regarding the side effects of this medication. The nurse tells that client that which undesirable effect is associated with this medication? 1. Insomnia 2. Constipation 3. Hypotension 4. Bronchospasm

4. Bronchospasm

237.) A client who is on lithium carbonate (Lithobid) will be discharged at the end of the week. In formulating a discharge teaching plan, the nurse will instruct the client that it is most important to: 1. Avoid soy sauce, wine, and aged cheese. 2. Have the lithium level checked every week. 3. Take medication only as prescribed because it can become addicting. 4. Check with the psychiatrist before using any over-the-counter (OTC) medications or prescription medications.

4. Check with the psychiatrist before using any over-the-counter (OTC) medications or prescription medications. Rationale: Lithium is the medication of choice to treat manic-depressive illness. Many OTC medications interact with lithium, and the client is instructed to avoid OTC medications while taking lithium. Lithium is not addicting, and, although serum lithium levels need to be monitored, it is not necessary to check these levels every week. A tyramine-free diet is associated with monoamine oxidase inhibitors.

lactulose (Cephulac)

reduce blood ammonia by excreation of ammonia by stools 2 -3 soft stools per day

217.) A health care provider prescribes auranofin (Ridaura) for a client with rheumatoid arthritis. Which of the following would indicate to the nurse that the client is experiencing toxicity related to the medication? 1. Joint pain 2. Constipation 3. Ringing in the ears 4. Complaints of a metallic taste in the mouth

4. Complaints of a metallic taste in the mouth Rationale: Ridaura is the one gold preparation that is given orally rather than by injection. Gastrointestinal reactions including diarrhea, abdominal pain, nausea, and loss of appetite are common early in therapy, but these usually subside in the first 3 months of therapy. Early symptoms of toxicity include a rash, purple blotches, pruritus, mouth lesions, and a metallic taste in the mouth.

94.) The client with acquired immunodeficiency syndrome has begun therapy with zidovudine (Retrovir, Azidothymidine, AZT, ZDV). The nurse carefully monitors which of the following laboratory results during treatment with this medication? 1. Blood culture 2. Blood glucose level 3. Blood urea nitrogen 4. Complete blood count

4. Complete blood count Rationale: A common side effect of therapy with zidovudine is leukopenia and anemia. The nurse monitors the complete blood count results for these changes. Options 1, 2, and 3 are unrelated to the use of this medication.

124.) A client with chronic renal failure is receiving ferrous sulfate (Feosol). The nurse monitors the client for which common side effect associated with this medication? 1. Diarrhea 2. Weakness 3. Headache 4. Constipation

4. Constipation Rationale: Feosol is an iron supplement used to treat anemia. Constipation is a frequent and uncomfortable side effect associated with the administration of oral iron supplements. Stool softeners are often prescribed to prevent constipation. **Focus on the name of the medication. Recalling that oral iron can cause constipation will easily direct you to the correct option.**

155.) Mycophenolate mofetil (CellCept) is prescribed for a client as prophylaxis for organ rejection following an allogeneic renal transplant. Which of the following instructions does the nurse reinforce regarding administration of this medication? 1. Administer following meals. 2. Take the medication with a magnesium-type antacid. 3. Open the capsule and mix with food for administration. 4. Contact the health care provider (HCP) if a sore throat occurs.

4. Contact the health care provider (HCP) if a sore throat occurs. Rationale: Mycophenolate mofetil should be administered on an empty stomach. The capsules should not be opened or crushed. The client should contact the HCP if unusual bleeding or bruising, sore throat, mouth sores, abdominal pain, or fever occurs because these are adverse effects of the medication. Antacids containing magnesium and aluminum may decrease the absorption of the medication and therefore should not be taken with the medication. The medication may be given in combination with corticosteroids and cyclosporine. **neutropenia can occur with this medication**

132.) The client with non-Hodgkin's lymphoma is receiving daunorubicin (DaunoXome). Which of the following would indicate to the nurse that the client is experiencing a toxic effect related to the medication? 1. Fever 2. Diarrhea 3. Complaints of nausea and vomiting 4. Crackles on auscultation of the lungs

4. Crackles on auscultation of the lungs Rationale: Cardiotoxicity noted by abnormal electrocardiographic findings or cardiomyopathy manifested as congestive heart failure is a toxic effect of daunorubicin. Bone marrow depression is also a toxic effect. Nausea and vomiting are frequent side effects associated with the medication that begins a few hours after administration and lasts 24 to 48 hours. Fever is a frequent side effect, and diarrhea can occur occasionally. The other options, however, are not toxic effects. **keep in mind that the question is asking about a toxic effect and think: ABCs—airway, breathing, and circulation**

117.) A nurse has given the client taking ethambutol (Myambutol) information about the medication. The nurse determines that the client understands the instructions if the client immediately reports: 1. Impaired sense of hearing 2. Distressing gastrointestinal side effects 3. Orange-red discoloration of body secretions 4. Difficulty discriminating the color red from green

4. Difficulty discriminating the color red from green Rationale: Ethambutol causes optic neuritis, which decreases visual acuity and the ability to discriminate between the colors red and green. This poses a potential safety hazard when driving a motor vehicle. The client is taught to report this symptom immediately. The client is also taught to take the medication with food if gastrointestinal upset occurs. Impaired hearing results from antitubercular therapy with streptomycin. Orange-red discoloration of secretions occurs with rifampin (Rifadin).

685. A nurse has given a client taking ethambutol (Myambutol) information about the medication. The nurse determines that the client understands the instructions if the client states to report immediately: 1. Impaired sense of hearing 2. Gastrointestinal side effects 3. Orange-red discoloration of body secretions 4. Difficulty in discriminating the color red from green

4. Difficulty in discriminating the color red from green

190.) A child is hospitalized with a diagnosis of lead poisoning. The nurse assisting in caring for the child would prepare to assist in administering which of the following medications? 1. Activated charcoal 2. Sodium bicarbonate 3. Syrup of ipecac syrup 4. Dimercaprol (BAL in Oil)

4. Dimercaprol (BAL in Oil) Rationale: Dimercaprol is a chelating agent that is administered to remove lead from the circulating blood and from some tissues and organs for excretion in the urine. Sodium bicarbonate may be used in salicylate poisoning. Syrup of ipecac is used in the hospital setting in poisonings to induce vomiting. Activated charcoal is used to decrease absorption in certain poisoning situations. Note that dimercaprol is prepared with peanut oil, and hence should be avoided by clients with known or suspected peanut allergy.

203.) A nurse is preparing to give the postcraniotomy client medication for incisional pain. The family asks the nurse why the client is receiving codeine sulfate and not "something stronger." In formulating a response, the nurse incorporates the understanding that codeine: 1. Is one of the strongest opioid analgesics available 2. Cannot lead to physical or psychological dependence 3. Does not cause gastrointestinal upset or constipation as do other opioids 4. Does not alter respirations or mask neurological signs as do other opioids

4. Does not alter respirations or mask neurological signs as do other opioids Rationale: Codeine sulfate is the opioid analgesic often used for clients after craniotomy. It is frequently combined with a nonopioid analgesic such as acetaminophen for added effect. It does not alter the respiratory rate or mask neurological signs as do other opioids. Side effects of codeine include gastrointestinal upset and constipation. The medication can lead to physical and psychological dependence with chronic use. It is not the strongest opioid analgesic available.

223.) A client with a psychotic disorder is being treated with haloperidol (Haldol). Which of the following would indicate the presence of a toxic effect of this medication? 1. Nausea 2. Hypotension 3. Blurred vision 4. Excessive salivation

4. Excessive salivation Rationale: Toxic effects include extrapyramidal symptoms (EPS) noted as marked drowsiness and lethargy, excessive salivation, and a fixed stare. Akathisia, acute dystonias, and tardive dyskinesia are also signs of toxicity. Hypotension, nausea, and blurred vision are occasional side effects.

955. A home health nurse visits a client. The client gives the nurse a bottle of clomipramine (Anafranil). The nurse notes that the medication has not been taken by the client in 2 months. What behaviors observed in the client would validate noncompliance with this medication? 1. Complaints of insomnia 2. Complaints of hunger and fatigue 3. A pulse rate less than 60 beats/min 4. Frequent handwashing with hot soapy water

4. Frequent handwashing with hot soapy water

231.) A client admitted to the hospital gives the nurse a bottle of clomipramine (Anafranil). The nurse notes that the medication has not been taken by the client in 2 months. What behaviors observed in the client would validate noncompliance with this medication? 1. Complaints of hunger 2. Complaints of insomnia 3. A pulse rate less than 60 beats per minute 4. Frequent handwashing with hot, soapy water

4. Frequent handwashing with hot, soapy water Rationale: Clomipramine is commonly used in the treatment of obsessive-compulsive disorder. Handwashing is a common obsessive-compulsive behavior. Weight gain is a common side effect of this medication. Tachycardia and sedation are side effects. Insomnia may occur but is seldom a side effect.

236.) A client is being treated for depression with amitriptyline hydrochloride. During the initial phases of treatment, the most important nursing intervention is: 1. Prescribing the client a tyramine-free diet 2. Checking the client for anticholinergic effects 3. Monitoring blood levels frequently because there is a narrow range between therapeutic and toxic blood levels of this medication 4. Getting baseline postural blood pressures before administering the medication and each time the medication is administered

4. Getting baseline postural blood pressures before administering the medication and each time the medication is administered Rationale: Amitriptyline hydrochloride is a tricyclic antidepressant often used to treat depression. It causes orthostatic changes and can produce hypotension and tachycardia. This can be frightening to the client and dangerous because it can result in dizziness and client falls. The client must be instructed to move slowly from a lying to a sitting to a standing position to avoid injury if these effects are experienced. The client may also experience sedation, dry mouth, constipation, blurred vision, and other anticholinergic effects, but these are transient and will diminish with time.

196.) A client has been prescribed amikacin (Amikin). Which of the following priority baseline functions should be monitored? 1. Apical pulse 2. Liver function 3. Blood pressure 4. Hearing acuity

4. Hearing acuity Rationale: Amikacin (Amikin) is an antibiotic. This medication can cause ototoxicity and nephrotoxicity; therefore, hearing acuity tests and kidney function studies should be performed before the initiation of therapy. Apical pulse, liver function studies, and blood pressure are not specifically related to the use of this medication.

175.) A nurse notes that a client is receiving lamivudine (Epivir). The nurse determines that this medication has been prescribed to treat which of the following? 1. Pancreatitis 2. Pharyngitis 3. Tonic-clonic seizures 4. Human immunodeficiency virus (HIV) infection

4. Human immunodeficiency virus (HIV) infection Rationale: Lamivudine is a nucleoside reverse transcriptase inhibitor and antiviral medication. It slows HIV replication and reduces the progression of HIV infection. It also is used to treat chronic hepatitis B and is used for prophylaxis in health care workers at risk of acquiring HIV after occupational exposure to the virus. **Note the letters "-vir" in the trade name for this medication**

192.) A nurse is collecting medication information from a client, and the client states that she is taking garlic as an herbal supplement. The nurse understands that the client is most likely treating which of the following conditions? 1. Eczema 2. Insomnia 3. Migraines 4. Hyperlipidemia

4. Hyperlipidemia Rationale: Garlic is an herbal supplement that is used to treat hyperlipidemia and hypertension. An herbal supplement that may be used to treat eczema is evening primrose. Insomnia has been treated with both valerian root and chamomile. Migraines have been treated with feverfew.

73.) A client with myasthenia gravis is suspected of having cholinergic crisis. Which of the following indicate that this crisis exists? 1. Ataxia 2. Mouth sores 3. Hypotension 4. Hypertension

4. Hypertension Rationale: Cholinergic crisis occurs as a result of an overdose of medication. Indications of cholinergic crisis include gastrointestinal disturbances, nausea, vomiting, diarrhea, abdominal cramps, increased salivation and tearing, miosis, hypertension, sweating, and increased bronchial secretions.

chapter 67- neuro 816. Carbidopa-levodopa (Sinemet) is prescribed for the client with Parkinson's disease. The nurse monitors the client for side effects to the medication. Which of the following would indicate that the client is experiencing a side effect? 1. Pruritus 2. Tachycardia 3. Hypertension 4. Impaired voluntary movements

4. Impaired voluntary movements

212.) Mannitol (Osmitrol) is being administered to a client with increased intracranial pressure following a head injury. The nurse assisting in caring for the client knows that which of the following indicates the therapeutic action of this medication? 1. Prevents the filtration of sodium and water through the kidneys 2. Prevents the filtration of sodium and potassium through the kidneys 3. Decreases water loss by promoting the reabsorption of sodium and water in the loop of Henle 4. Induces diuresis by raising the osmotic pressure of glomerular filtrate, thereby inhibiting tubular reabsorption of water and solutes

4. Induces diuresis by raising the osmotic pressure of glomerular filtrate, thereby inhibiting tubular reabsorption of water and solutes Rationale: Mannitol is an osmotic diuretic that induces diuresis by raising the osmotic pressure of glomerular filtrate, thereby inhibiting tubular reabsorption of water and solutes. It is used to reduce intracranial pressure in the client with head trauma.

200.) A client is seen in the clinic for complaints of skin itchiness that has been persistent over the past several weeks. Following data collection, it has been determined that the client has scabies. Lindane is prescribed, and the nurse is asked to provide instructions to the client regarding the use of the medication. The nurse tells the client to: 1. Apply a thick layer of cream to the entire body. 2. Apply the cream as prescribed for 2 days in a row. 3. Apply to the entire body and scalp, excluding the face. 4. Leave the cream on for 8 to 12 hours and then remove by washing.

4. Leave the cream on for 8 to 12 hours and then remove by washing. Rationale: Lindane is applied in a thin layer to the entire body below the head. No more than 30 g (1 oz) should be used. The medication is removed by washing 8 to 12 hours later. Usually, only one application is required.

119.) A client with diabetes mellitus who has been controlled with daily insulin has been placed on atenolol (Tenormin) for the control of angina pectoris. Because of the effects of atenolol, the nurse determines that which of the following is the most reliable indicator of hypoglycemia? 1. Sweating 2. Tachycardia 3. Nervousness 4. Low blood glucose level

4. Low blood glucose level Rationale: β-Adrenergic blocking agents, such as atenolol, inhibit the appearance of signs and symptoms of acute hypoglycemia, which would include nervousness, increased heart rate, and sweating. Therefore, the client receiving this medication should adhere to the therapeutic regimen and monitor blood glucose levels carefully. Option 4 is the most reliable indicator of hypoglycemia.

205.) A nurse is assisting in preparing to administer acetylcysteine (Mucomyst) to a client with an overdose of acetaminophen (Tylenol). The nurse prepares to administer the medication by: 1. Administering the medication subcutaneously in the deltoid muscle 2. Administering the medication by the intramuscular route in the gluteal muscle 3. Administering the medication by the intramuscular route, mixed in 10 mL of normal saline 4. Mixing the medication in a flavored ice drink and allowing the client to drink the medication through a straw

4. Mixing the medication in a flavored ice drink and allowing the client to drink the medication through a straw Rationale: Because acetylcysteine has a pervasive odor of rotten eggs, it must be disguised in a flavored ice drink. It is consumed preferably through a straw to minimize contact with the mouth. It is not administered by the intramuscular or subcutaneous route. **Knowing that the medication is a solution that is also used for nebulization treatments will assist you to select the option that indicates an oral route**

143.) A client has just taken a dose of trimethobenzamide (Tigan). The nurse plans to monitor this client for relief of: 1. Heartburn 2. Constipation 3. Abdominal pain 4. Nausea and vomiting

4. Nausea and vomiting Rationale: Trimethobenzamide is an antiemetic agent used in the treatment of nausea and vomiting. The other options are incorrect.

17.) The client with ovarian cancer is being treated with vincristine (Oncovin). The nurse monitors the client, knowing that which of the following indicates a side effect specific to this medication? 1. Diarrhea 2. Hair loss 3. Chest pain 4. Numbness and tingling in the fingers and toes

4. Numbness and tingling in the fingers and toes Rationale: A side effect specific to vincristine is peripheral neuropathy, which occurs in almost every client. Peripheral neuropathy can be manifested as numbness and tingling in the fingers and toes. Depression of the Achilles tendon reflex may be the first clinical sign indicating peripheral neuropathy. Constipation rather than diarrhea is most likely to occur with this medication, although diarrhea may occur occasionally. Hair loss occurs with nearly all the antineoplastic medications. Chest pain is unrelated to this medication.

39.) The client with a gastric ulcer has a prescription for sucralfate (Carafate), 1 g by mouth four times daily. The nurse schedules the medication for which times? 1. With meals and at bedtime 2. Every 6 hours around the clock 3. One hour after meals and at bedtime 4. One hour before meals and at bedtime

4. One hour before meals and at bedtime Rationale: Sucralfate is a gastric protectant. The medication should be scheduled for administration 1 hour before meals and at bedtime. The medication is timed to allow it to form a protective coating over the ulcer before food intake stimulates gastric acid production and mechanical irritation. The other options are incorrect.

2.) Oral iron supplements are prescribed for a 6-year-old child with iron deficiency anemia. The nurse instructs the mother to administer the iron with which best food item? 1. Milk 2. Water 3. Apple juice 4. Orange juice

4. Orange juice Rationale: Vitamin C increases the absorption of iron by the body. The mother should be instructed to administer the medication with a citrus fruit or a juice that is high in vitamin C. Milk may affect absorption of the iron. Water will not assist in absorption. Orange juice contains a greater amount of vitamin C than apple juice.

15.) The client with small cell lung cancer is being treated with etoposide (VePesid). The nurse who is assisting in caring for the client during its administration understands that which side effect is specifically associated with this medication? 1. Alopecia 2. Chest pain 3. Pulmonary fibrosis 4. Orthostatic hypotension

4. Orthostatic hypotension Rationale: A side effect specific to etoposide is orthostatic hypotension. The client's blood pressure is monitored during the infusion. Hair loss occurs with nearly all the antineoplastic medications. Chest pain and pulmonary fibrosis are unrelated to this medication.

222.) A nurse has administered a dose of diazepam (Valium) to a client. The nurse would take which important action before leaving the client's room? 1. Giving the client a bedpan 2. Drawing the shades or blinds closed 3. Turning down the volume on the television 4. Per agency policy, putting up the side rails on the bed

4. Per agency policy, putting up the side rails on the bed Rationale: Diazepam is a sedative-hypnotic with anticonvulsant and skeletal muscle relaxant properties. The nurse should institute safety measures before leaving the client's room to ensure that the client does not injure herself or himself. The most frequent side effects of this medication are dizziness, drowsiness, and lethargy. For this reason, the nurse puts the side rails up on the bed before leaving the room to prevent falls. Options 1, 2, and 3 may be helpful measures that provide a comfortable, restful environment, but option 4 is the one that provides for the client's safety needs.

189.) Prostaglandin E1 is prescribed for a child with transposition of the great arteries. The mother of the child asks the nurse why the child needs the medication. The nurse tells the mother that the medication: 1. Prevents hypercyanotic (blue or tet) spells 2. Maintains an adequate hormone level 3. Maintains the position of the great arteries 4. Provides adequate oxygen saturation and maintains cardiac output

4. Provides adequate oxygen saturation and maintains cardiac output Rationale: A child with transposition of the great arteries may receive prostaglandin E1 temporarily to increase blood mixing if systemic and pulmonary mixing are inadequate to maintain adequate cardiac output. Options 1, 2, and 3 are incorrect. In addition, hypercyanotic spells occur in tetralogy of Fallot. **Use the ABCs—airway, breathing, and circulation—to answer the question. The correct option addresses circulation**

13.) The client with squamous cell carcinoma of the larynx is receiving bleomycin intravenously. The nurse caring for the client anticipates that which diagnostic study will be prescribed? 1. Echocardiography 2. Electrocardiography 3. Cervical radiography 4. Pulmonary function studies

4. Pulmonary function studies Rationale: Bleomycin is an antineoplastic medication (Chemotheraputic Agents) that can cause interstitial pneumonitis, which can progress to pulmonary fibrosis. Pulmonary function studies along with hematological, hepatic, and renal function tests need to be monitored. The nurse needs to monitor lung sounds for dyspnea and crackles, which indicate pulmonary toxicity. The medication needs to be discontinued immediately if pulmonary toxicity occurs. Options 1, 2, and 3 are unrelated to the specific use of this medication.

70.) Oxybutynin chloride (Ditropan XL) is prescribed for a client with neurogenic bladder. Which sign would indicate a possible toxic effect related to this medication? 1. Pallor 2. Drowsiness 3. Bradycardia 4. Restlessness

4. Restlessness Rationale: Toxicity (overdosage) of this medication produces central nervous system excitation, such as nervousness, restlessness, hallucinations, and irritability. Other signs of toxicity include hypotension or hypertension, confusion, tachycardia, flushed or red face, and signs of respiratory depression. Drowsiness is a frequent side effect of the medication but does not indicate overdosage.

66.) Trimethoprim-sulfamethoxazole (TMP-SMZ) is prescribed for a client. A nurse should instruct the client to report which symptom if it developed during the course of this medication therapy? 1. Nausea 2. Diarrhea 3. Headache 4. Sore throat

4. Sore throat Rationale: Clients taking trimethoprim-sulfamethoxazole (TMP-SMZ) should be informed about early signs of blood disorders that can occur from this medication. These include sore throat, fever, and pallor, and the client should be instructed to notify the health care provider if these symptoms occur. The other options do not require health care provider notification.

67.) Phenazopyridine hydrochloride (Pyridium) is prescribed for a client for symptomatic relief of pain resulting from a lower urinary tract infection. The nurse reinforces to the client: 1. To take the medication at bedtime 2. To take the medication before meals 3. To discontinue the medication if a headache occurs 4. That a reddish orange discoloration of the urine may occur

4. That a reddish orange discoloration of the urine may occur Rationale: The nurse should instruct the client that a reddish-orange discoloration of urine may occur. The nurse also should instruct the client that this discoloration can stain fabric. The medication should be taken after meals to reduce the possibility of gastrointestinal upset. A headache is an occasional side effect of the medication and does not warrant discontinuation of the medication.

177.) A nurse is assisting in caring for a pregnant client who is receiving intravenous magnesium sulfate for the management of preeclampsia and notes that the client's deep tendon reflexes are absent. On the basis of this data, the nurse reports the finding and makes which determination? 1. The magnesium sulfate is effective. 2. The infusion rate needs to be increased. 3. The client is experiencing cerebral edema. 4. The client is experiencing magnesium toxicity.

4. The client is experiencing magnesium toxicity. Rationale: Magnesium toxicity can occur as a result of magnesium sulfate therapy. Signs of magnesium sulfate toxicity relate to the central nervous system depressant effects of the medication and include respiratory depression; loss of deep tendon reflexes; sudden decrease in fetal heart rate or maternal heart rate, or both; and sudden drop in blood pressure. Hyperreflexia indicates increased cerebral edema. An absence of reflexes indicates magnesium toxicity. The therapeutic serum level of magnesium for a client receiving magnesium sulfate ranges from 4 to 7.5 mEq/L (5 to 8 mg/dL).

232.) A client in the mental health unit is administered haloperidol (Haldol). The nurse would check which of the following to determine medication effectiveness? 1. The client's vital signs 2. The client's nutritional intake 3. The physical safety of other unit clients 4. The client's orientation and delusional status

4. The client's orientation and delusional status Rationale: Haloperidol is used to treat clients exhibiting psychotic features. Therefore, to determine medication effectiveness, the nurse would check the client's orientation and delusional status. Vital signs are routine and not specific to this situation. The physical safety of other clients is not a direct assessment of this client. Monitoring nutritional intake is not related to this situation.

99.) The client with acquired immunodeficiency syndrome and Pneumocystis jiroveci infection has been receiving pentamidine isethionate (Pentam 300). The client develops a temperature of 101° F. The nurse does further monitoring of the client, knowing that this sign would most likely indicate: 1. The dose of the medication is too low. 2. The client is experiencing toxic effects of the medication. 3. The client has developed inadequacy of thermoregulation. 4. The result of another infection caused by leukopenic effects of the medication.

4. The result of another infection caused by leukopenic effects of the medication. Rationale: Frequent side effects of this medication include leukopenia, thrombocytopenia, and anemia. The client should be monitored routinely for signs and symptoms of infection. Options 1, 2, and 3 are inaccurate interpretations.

121.) A client who is taking hydrochlorothiazide (HydroDIURIL, HCTZ) has been started on triamterene (Dyrenium) as well. The client asks the nurse why both medications are required. The nurse formulates a response, based on the understanding that: 1. Both are weak potassium-losing diuretics. 2. The combination of these medications prevents renal toxicity. 3. Hydrochlorothiazide is an expensive medication, so using a combination of diuretics is cost-effective. 4. Triamterene is a potassium-sparing diuretic, whereas hydrochlorothiazide is a potassium-losing diuretic.

4. Triamterene is a potassium-sparing diuretic, whereas hydrochlorothiazide is a potassium-losing diuretic. Rationale: Potassium-sparing diuretics include amiloride (Midamor), spironolactone (Aldactone), and triamterene (Dyrenium). They are weak diuretics that are used in combination with potassium-losing diuretics. This combination is useful when medication and dietary supplement of potassium is not appropriate. The use of two different diuretics does not prevent renal toxicity. Hydrochlorothiazide is an effective and inexpensive generic form of the thiazide classification of diuretics. **It is especially helpful to remember that hydrochlorothiazide is a potassium-losing diuretic and triamterene is a potassium-sparing diuretic**

160.) Meperidine hydrochloride (Demerol) is prescribed for the client with pain. Which of the following would the nurse monitor for as a side effect of this medication? 1. Diarrhea 2. Bradycardia 3. Hypertension 4. Urinary retention

4. Urinary retention Rationale: Meperidine hydrochloride (Demerol) is an opioid analgesic. Side effects of this medication include respiratory depression, orthostatic hypotension, tachycardia, drowsiness and mental clouding, constipation, and urinary retention.

140.) The client has a new prescription for metoclopramide (Reglan). On review of the chart, the nurse identifies that this medication can be safely administered with which condition? 1. Intestinal obstruction 2. Peptic ulcer with melena 3. Diverticulitis with perforation 4. Vomiting following cancer chemotherapy

4. Vomiting following cancer chemotherapy Rationale: Metoclopramide is a gastrointestinal (GI) stimulant and antiemetic. Because it is a GI stimulant, it is contraindicated with GI obstruction, hemorrhage, or perforation. It is used in the treatment of emesis after surgery, chemotherapy, and radiation.

When initially teaching a patient the supraglottic swallow following a radical neck dissection, with which of the following foods should the nurse begin? A. Cola B. Applesauce C. French fries D. White grape juice

A. ColaWhen learning the supraglottic swallow, it may be helpful to start with carbonated beverages because the effervescence provides clues about the liquid's position. Thin, watery fluids should be avoided because they are difficult to swallow and increase the risk of aspiration. Nonpourable pureed foods, such as applesauce, would decrease the risk of aspiration, but carbonated beverages are the better choice to start with.

104.) Disulfiram (Antabuse) is prescribed for a client who is seen in the psychiatric health care clinic. The nurse is collecting data on the client and is providing instructions regarding the use of this medication. Which is most important for the nurse to determine before administration of this medication? 1. A history of hyperthyroidism 2. A history of diabetes insipidus 3. When the last full meal was consumed 4. When the last alcoholic drink was consumed

4. When the last alcoholic drink was consumed Rationale: Disulfiram is used as an adjunct treatment for selected clients with chronic alcoholism who want to remain in a state of enforced sobriety. Clients must abstain from alcohol intake for at least 12 hours before the initial dose of the medication is administered. The most important data are to determine when the last alcoholic drink was consumed. The medication is used with caution in clients with diabetes mellitus, hypothyroidism, epilepsy, cerebral damage, nephritis, and hepatic disease. It is also contraindicated in severe heart disease, psychosis, or hypersensitivity related to the medication.

180.) A health care provider (HCP) writes a prescription for digoxin (Lanoxin), 0.25 mg daily. The nurse teaches the client about the medication and tells the client that it is important to: 1. Count the radial and carotid pulses every morning. 2. Check the blood pressure every morning and evening. 3. Stop taking the medication if the pulse is higher than 100 beats per minute. 4. Withhold the medication and call the HCP if the pulse is less than 60 beats per minute.

4. Withhold the medication and call the HCP if the pulse is less than 60 beats per minute. Rationale: An important component of taking this medication is monitoring the pulse rate; however, it is not necessary for the client to take both the radial and carotid pulses. It is not necessary for the client to check the blood pressure every morning and evening because the medication does not directly affect blood pressure. It is most important for the client to know the guidelines related to withholding the medication and calling the HCP. The client should not stop taking a medication.

185.) Which of the following herbal therapies would be prescribed for its use as an antispasmodic? Select all that apply. 1.Aloe 2.Kava 3.Ginger 4.Chamomile 5.Peppermint oil

4.Chamomile 5.Peppermint oil Rationale: Chamomile has a mild sedative effect and acts as an antispasmodic and anti-inflammatory. Peppermint oil acts as an antispasmodic and is used for irritable bowel syndrome. Topical aloe promotes wound healing. Aloe taken orally acts as a laxative. Kava has an anxiolytic, sedative, and analgesic effect. Ginger is effective in relieving nausea.

When admitting a patient with the diagnosis of asthma exacerbation, the nurse will assess for which of the following potential triggers? (Select all that apply.) A. Exercise B. Allergies C. Emotional stress D. Decreased humidity

A,B,C Although the exact mechanism of asthma is unknown, there are several triggers that may precipitate an attack. These include allergens, exercise, air pollutants, respiratory infections, drug and food additives, psychologic factors, and GERD.

During admission of a patient diagnosed with non-small cell carcinoma of the lung, the nurse questions the patient related to a history of which of the following risk factors for this type of cancer? (Select all that apply.) A. Asbestos exposure B. Cigarette smoking C. Exposure to uranium D. Chronic interstitial fibrosis

A,B,C Non-small carcinoma is associated with cigarette smoking and exposure to environmental carcinogens, including asbestos and uranium. Chronic interstitial fibrosis is associated with the development of adenocarcinoma of the lung.

Which content about self-care should the nurse include in the teaching plan of a client who has genital herpes? (Select all that apply.) A. Encourage annual physical and Pap smear. B. Take antiviral medication as prescribed. C. Use condoms to avoid transmission to others. D. Warm sitz baths may relieve itching. E. Use Nystatin suppositories to control itching. F. Douche with weak vinegar solutions to decrease itching.

A,B,C,D. (E) is specific for Candida infections and (F) is used to treat Trichomonas.

When assessing a patient's sleep-rest pattern related to respiratory health, the nurse would ask if the patient: (Select all that apply.) A. Has trouble falling asleep B. Awakens abruptly during the night C. Sleeps more than 8 hours per night D. Has to sleep with the head elevated

A,B,D The patient with sleep apnea may have insomnia and/or abrupt awakenings. Patients with cardiovascular disease (e.g., heart failure that may affect respiratory health) may need to sleep with the head elevated on several pillows (orthopnea). Sleeping more than 8 hours per night is not indicative of impaired respiratory health.

When admitting a 45-year-old female with a diagnosis of pulmonary embolism, the nurse will assess the patient for which of the following risk factors? (Select all that apply.) A. Obesity B. Pneumonia C. Hypertension D. Cigarette smoking

A,C,D Research has demonstrated an increased risk of pulmonary embolism in women associated with obesity, heavy cigarette smoking, and hypertension. Other risk factors include immobilization, surgery within the last 3 months, stroke, history of DVT, and malignancy.

To promote airway clearance in a patient with pneumonia, the nurse instructs the patient to do which of the following? (Select all that apply.) A. Splint the chest when coughing B. Maintain a semi-Fowler's position C. Maintain adequate fluid intake D. Instruct patient to cough at end of exhalation

A,C,D The nurse should instruct the patient to splint the chest while coughing. This will reduce discomfort and allow for a more effective cough. Maintaining adequate fluid intake liquefies secretions, allowing easier expectoration. Coughing at the end of exhalation promotes a more effective cough. The patient should be positioned in an upright sitting position (high-Fowler's) with head slightly flexed.

When caring for a patient with metastatic cancer, the nurse notes a hemoglobin level of 8.7 g/dl and hematocrit of 26%. The nurse would place highest priority on initiating interventions that will reduce which of the following? A. Fatigue B. Thirst C. Headache D. Abdominal pain

A. Fatigue The patient with a low hemoglobin and hematocrit (normal values approximately 13.5% to 17% and 40% to 54%, respectively) is anemic and would be most likely to experience fatigue. This symptom develops because of the lowered oxygen-carrying capacity that leads to reduced tissue oxygenation to carry out cellular functions.

The physician has prescribed salmeterol (Serevent) for a patient with asthma. In reviewing the use of dry powder inhalers (DPIs) with the patient, the nurse should provide which of the following instructions? A. "Close lips tightly around the mouthpiece and breathe in deeply and quickly." B. "To administer a DPI, you must use a spacer that holds the medicine so that you can inhale it." C. "Hold the inhaler several inches in front of your mouth and breathe in slowly, holding the medicine as long as possible." D. "You will know you have correctly used the DPI when you taste or sense the medicine going into your lungs."

A. "Close lips tightly around the mouthpiece and breathe in deeply and quickly." Dry powder inhalers do not require spacer devices. The patient should be instructed to breathe in deeply and quickly to ensure medicine moves down deeply into lungs. The patient may not taste or sense the medicine going into the lungs.

Which of the following instructions are most appropriate in the home management of a patient who has undergone surgery for oral cancer? A. "You should drink plenty of fluids and eat foods you enjoy." B. "It is normal to have some leakage of saliva from the suture line." C. "Lying in a prone position helps decrease swelling at the suture line." D. "You should avoid foods high in protein while your suture line is healing."

A. "You should drink plenty of fluids and eat foods you enjoy." For patients who have undergone treatment for head and neck cancers, maintaining adequate nutrition is a challenge. The nurse encourages the patient to increase fluids to prevent dehydration and liquefy secretions. These patients are more likely to eat foods that they enjoy and can tolerate.

A male client with arterial peripheral vascular disease (PVD) complains of pain in his feet. Which instruction should the nurse give to the UPA to quickly relieve the client's pain? A. Help the client to dangle his legs. B. Apply compression stockings. C. Assist with passive leg exercises. D. Ambulate three times daily.

A. A client who has arterial PVD may benefit from a dependent position which can be achieved by dangling by improving blood flow and relieving pain. (B) is indicated for venous insufficiency and (C) is indicated for bed rest. (D) is indicated to facilitate collateral circulation and may improve long term complaints of pain.

Which of the following test results identify that a patient with an asthma attack is responding to treatment? A. A decreased exhaled nitric oxide B. An increase in CO2 levels C. A decrease in white blood cell count D. An increase in serum bicarbonate levels

A. A decreased exhaled nitric oxide. Nitric oxide levels are increased in the breath of people with asthma. A decrease in the exhaled nitric oxide concentration suggests that the treatment may be decreasing the lung inflammation associated with asthma.

A patient is having inspiratory stridor (crowing respiration) and the nurse suspects he is experiencing a laryngospasm. Which of the following would be most appropriate to implement for a patient experiencing a laryngospasm? A. Administer 100% oxygen. B. Position the patient in high Fowler's position. C. Insert a 16-gauge (large-bore) IV needle. D. Activate the emergency response team (code blue team) to the patient's room.

A. Administer 100% oxygen.A nurse should immediately administer 100% oxygen to the patient until the airway is fully reestablished, the larynx relaxes, and the spasms stop. Activating the emergency response team is not an immediate nursing action at this time because the nurse can administer the oxygen without the assistance of others. Positioning the patient in high Fowler's will not address the patient's need for immediate reoxygenation because of the patient's compromised respiratory state. Insertion of an IV device is not the first priority response but should be implemented after the nurse has assessed that the airway is stable.

When caring for a patient who is 3 hours postoperative laryngectomy, the nurse's highest priority assessment would be: A. Airway patency B. Patient comfort C. Incisional drainage D. Blood pressure and heart rate

A. Airway patency Remember ABCs with prioritization. Airway patency is always the highest priority and is essential for a patient undergoing surgery surrounding the upper respiratory system.

The patient has an order for each of the following inhalers. Which of the following should the nurse offer to the patient at the onset of an asthma attack? A. Albuterol (Proventil) B. Beclomethasone (Beclovent) C. Ipratropium bromide (Atrovent) D. Salmeterol (Serevent)

A. Albuterol (Proventil) Albuterol is a short-acting bronchodilator that should initially be given when the patient experiences an asthma attack.

A 45-year-old man with asthma is brought to the emergency department by automobile. He is short of breath and appears frightened. During the initial nursing assessment, which of the following clinical manifestations might be present as an early symptom during an exacerbation of asthma? A. Anxiety B. Cyanosis C. Hypercapnia D. Bradycardia

A. Anxiety An early symptom during an asthma attack is anxiety because he is acutely aware of the inability to get sufficient air to breathe. He will be hypoxic early on with decreased PaCO2 and increased pH as he is hyperventilating.

The nurse is performing hourly neurological check for a client with a head injury. Which new assessment finding warrants the most immediate intervention by the nurse? A. A unilateral pupil that is dilated and nonreactive to light. B. Client cries out when awakened by a verbal stimulus. C. Client demonstrates a loss of memory to the events leading up to the injury. D. Onset of nausea, headache, and vertigo.

A. Any changes in pupil size and reactivity is an indication of increasing ICP and should be reported immediately. (B) is normal for being awakened. (C & D) are common manifestations of head injury and less of an immediacy than (A).

Following a patient's bone marrow aspiration, which of the following nursing interventions should a nurse anticipate? A. Application of firm pressure to the site B. Positioning the patient in a prone position C. Positioning the patient in a supine position D. Application of a warm, moist compress to the site

A. Application of firm pressure to the site After a bone marrow aspiration procedure, a nurse should apply pressure to the aspiration site until bleeding stops. Application of a warm, moist compress will not alter the potential for bleeding. Positioning the patient to assume a supine or prone position also will not address the need to control bleeding from the aspiration site.

The nurse evaluates that a patient is experiencing the expected beneficial effects of ipratropium (Atrovent) after noting which of the following assessment findings? A. Increased peak flow readings B. Increased level of consciousness C. Decreased sputum production D. Increased respiratory rate

A. Increased peak flow readings. Ipratropium is a bronchodilator that should lead to increased PEFRs.

When assessing a patient's nutritional-metabolic pattern related to hematologic health, the nurse would: A. Inspect the skin for petechiae. B. Ask the patient about joint pain. C. Assess for vitamin C deficiency. D. Determine if the patient can perform ADLs.

A. Inspect the skin for petechiae. Any changes in the skin's texture or color should be explored when assessing the patient's nutritional-metabolic pattern related to hematologic health. The presences of petechiae or ecchymotic areas could be indicative of hematologic deficiencies related to poor nutritional intake or related causes.

The nurse who has administered a first dose of oral prednisone (Deltasone) to the patient with asthma writes on the care plan to begin monitoring which of the following patient parameters? A. Intake and output B. Bowel sounds C. Apical pulse D. Deep tendon reflexes

A. Intake and output Corticosteroids such as prednisone can lead to fluid retention. For this reason, it is important to monitor the patient's intake and output.

The nurse reviews pursed lip breathing with a patient newly diagnosed with emphysema. The nurse reinforces that this technique will assist respiration by which of the following mechanisms? A. Preventing bronchial collapse and air trapping in the lungs during exhalation B. Increasing the respiratory rate and giving the patient control of respiratory patterns C. Loosening secretions so that they may be coughed up more easily D. Promoting maximal inhalation for better oxygenation of the lungs

A. Preventing bronchial collapse and air trapping in the lungs during exhalation The focus of pursed lip breathing is to slow down the exhalation phase of respiration, which decreases bronchial collapse and subsequent air trapping in the lungs during exhalation.

If a nurse is caring for an 80-year-old patient with a temperature of 100.4° F, crackles at the right lung base, pain with deep inspiration, and dyspnea, which of the following orders is the nurse's priority? A. Sputum specimen for culture and sensitivity B. Codeine 15 mg orally every 6 hours as needed C. Incentive spirometer every 2 hours while awake D. Amoxicillin (Amoxil) 500 mg orally 4 times a day

A. Sputum specimen for culture and sensitivity The patient presents with signs of a respiratory infection. To initiate the most effective therapy, the health care prescriber must know the pathogen causing the infection. Therefore, the sputum specimen is the nurse's priority. If the antibiotic is administered before the specimen is obtained, the results of the culture might not be as accurate and could impair the effectiveness of therapy. After the specimen is obtained, the nurse can administer codeine for coughing and begin the incentive spirometry to mobilize secretions and improve the patient's ability to expectorate the secretions.

In assessing an older client with dementia for sundowning syndrome, what assessment technique is best for the nurse to use? A. Observe for tiredness at the end of the day. B. Perform a neurologic exam and mental status exam. C. Monitor for medication side effects. D. Assess for decreased gross motor movement.

A. Sundowning syndrome is a pattern of agitated behavior in the evening, believed to be associated with tiredness at the end of the day combined with fewer orienting stimuli, such as activities and interactions. (B, C, & D) with not provide information about this syndrome.

During assessment of a client in the intensive care unit, the nurse notes that the client's breath sounds are clear upon auscultation, but jugular vein distention and muffled heart sounds are present. Which intervention should the nurse implement? A. Prepare the client for a pericardial tap. B. Administer intravenous furosemide (Lasix). C. Assist the client to cough and deep breathe. D. Instruct the client to restrict oral fluid intake.

A. The client is exhibiting symptoms of cardiac tamponade that results in reduced cardiac output. Treatment is pericardial tap. (B) is not a treatment. (C) is not priority. (D) Fluids are frequently increased but this is not as priority as (A).

A client with cirrhosis states that his disease was cause by a blood transfusion. What information should the nurse obtain first to provide effective client teaching? A. The year the blood transfusion was received B. The amount of alcohol the client drinks C. How long the client has had cirrhosis D. The client's normal coping mechanisms

A. The nurse should first verify the clients explanation (A) since it may be accurate due to prior to 1990 blood was not screened for Hep C and hep C can cause cirrhosis. Not all cirrhosis is caused is caused by alcoholism (B) (C & D) provide useful but less relevant information.

Before beginning a transfusion of RBCs, which of the following actions by the nurse would be of highest priority to avoid an error during this procedure? A. Check the identifying information on the unit of blood against the patient's ID bracelet. B. Select new primary IV tubing primed with lactated Ringer's solution to use for the transfusion. C. Add the blood transfusion as a secondary line to the existing IV and used the IV controller to maintain correct flow. D. Remain with the patient for 60 minutes after beginning the transfusion to watch for signs of a transfusion reaction. The patient's identifying information (name, date of birth, medical record number) on the identification bracelet should exactly match the information on the blood bank tag that has been placed on the unit of blood. If any information does not match, the transfusions should not be hung because of possible error and risk to the patient.

A. The patient's identifying information (name, date of birth, medical record number) on the identification bracelet should exactly match the information on the blood bank tag that has been placed on the unit of blood. If any information does not match, the transfusions should not be hung because of possible error and risk to the patient.

A client is placed on a mechanical ventilator following a cerebral hemorrhage, and vecuronium bromide (Norcuron) 0.04 mg/kg every 12 hours IV is prescribed. What is the priority nursing diagnosis for this client? A. Impaired communication related to paralysis of skeletal muscles. B. Hight risk or infection related to increased ICP. C. Potential for injury related to impaired lung expansion. D. Social isolation related to inability to communicate.

A. To increase the client's tolerance of the endotracheal intubation and/or mechanical ventilation, a skeletal-muscle relaxant such as vecuronium is usually prescribed. (A) is a serious outcome because the client cannot communicate his/her needs. (D) is not as much of a priority. (B) infection is not related to ICP. (C) is incorrect because the ventilator will ensure that the lungs are expanded.

A nurse is working on a respiratory care unit where many of the patients are affected by asthma. Which of the following actions by the nurse would most likely increase respiratory difficulty for the patients? A. Wearing perfume to work B. Encouraging patients to ambulate daily C. Allowing the patients to eat green leafy vegetables D. Withholding antibiotic therapy until cultures are obtained

A. Wearing perfume to work People with asthma should avoid extrinsic allergens and irritants (e.g., dust, pollen, smoke, certain foods, colognes and perfumes, certain types of medications) because their airways become inflamed, producing shortness of breath, chest tightness, and wheezing. Many green leafy vegetables are rich in vitamins, minerals, and proteins, which incorporate healthy lifestyle patterns into the patients' daily living routines. Routine exercise is a part of a prudent lifestyle, and for patients with asthma the physical and psychosocial effects of ambulation can incorporate feelings of well-being, strength, and enhancement of physical endurance. Antibiotic therapy is always initiated after cultures are obtained so that the sensitivity to the organism can be readily identified.

A patient is admitted to the hospital with fever, chills, a productive cough with rusty sputum, and pleuritic chest pain. Pneumococcal pneumonia is suspected. An appropriate nursing diagnosis for the patient based on the patient's manifestations is A. hyperthermia related to acute infectious process. B. chronic pain related to ineffective pain management. C. risk for injury related to disorientation and confusion. D. ineffective airway clearance related to retained secretions.

A. hyperthermia related to acute infectious process. The patient with pneumococcal pneumonia is acutely ill with fever and the systemic manifestations of fever, such as chills, thirst, headache, and malaise. Interventions that monitor temperature and aid in lowering body temperature are appropriate. Ineffective airway clearance would be manifested by adventitious breath sounds and difficulty producing secretions. Disorientation and confusion are not noted in this patient and are not typical unless the patient is very hypoxemic. Pleuritic pain is an acute pain that is due to inflammation of the pleura.

To ensure the correct amount of oxygen delivery for a patient receiving 35% oxygen via a Venturi mask, it is most important that the nurse A. keep the air-entrainment ports clean and unobstructed. B. apply an adaptor to increase humidification of the oxygen. C. drain moisture condensation from the oxygen tubing every hour. D. keep the flow rate high enough to keep the bag from collapsing during inspiration.

A. keep the air-entrainment ports clean and unobstructed. Oxygen is delivered to a small jet in the center of a wide-based cone. Air is entrained (pulled through) openings in the cone as oxygen flows through the small jet. The degree of restriction or narrowness of the jet determines the amount of entrainment and the dilution of pure oxygen with room air and thus the concentration of oxygen. Although applying an adaptor can increase the humidification with the Venturi mask, it is not the best answer, because an open port is essential to proper functioning. Draining moisture condensation from the oxygen tubing is performed as often as needed, not on an hourly schedule. A plastic face mask with a reservoir bag needs to have sufficient flow rate to keep the bag inflated.

While caring for a patient with respiratory disease, a nurse observes that the oxygen saturation drops from 94% to 85% when the patient ambulates. The nurse should determine that A. supplemental oxygen should be used when the patient exercises. B. ABG determinations should be done to verify the oxygen saturation reading. C. this finding is a normal response to activity and that the patient should continue to be monitored. D. the oximetry probe should be moved from the finger to the earlobe for an accurate oxygen saturation measurement during activity.

A. supplemental oxygen should be used when the patient exercises.An oxygen saturation lower than 90% indicates inadequate oxygenation. If the drop is related to activity of some type, supplemental oxygen is indicated.

-mycin

Aminoglycoside (Antimicrobials) TX: pneumonia, meningitis, septicemia NI: high risk for ototoxicity, nephrotoxicity, monitor creatinine & BUN Common meds- gentamicin sulfate (garamycin) therapeutic range: 4-12mcg/dL

523. A burn client is receiving treatments of topical mafenide acetate to the site of injury. The nurse monitors the client, knowing that which finding indicates that a systemic effect has occurred? 1. Hyperventilation 2. Local rash at the burn site 3. Elevated blood pressure 4. Local pain at the burn site

Answer: 1 Mafenide acetate is a carbonic anhydrase inhibitor and can suppress renal excretion of acid, thereby causing acidosis. Clients receiving this treatment should be monitored for signs of an acid base imbalance (hyperventilation). If this occurs, the medication will probably be discontinued for 1 to 2 days. Options 2 and 4 describe local rather than systemic effects. An elevated blood pressure may be expected from the pain that occurs with a burn injury.

520. Salicyclic acid is prescribed for a client with a diagnosis of psoriasis. The nurse monitors the client, knowing that which finding indicates the presence of systemic toxicity from this medication? 1. Tinnitus 2. Diarrhea 3. Constipation 4. Decreased respirations

Answer: 1 Salicyclic acid is absorbed readily through the skin and systemic toxicity (salicylism) can result. Symptoms include tinnitus, dizziness, hyperpnea, and psychological disturbances. Constipation and diarrhea are not associated with salicylism.

524. Isotretinoin is prescribed for a client with severe acne. Before the administration of this medication, the nurse anticipates that which laboratory test will be prescribed? 1. Platelet count 2. Triglyceride level 3. Complete blood count 4. White blood cell count

Answer: 2 Isotretinoin can elevate triglyceride levels. Blood triglyceride levels hould be measured before treatment and periodically thereafter until the effect on the triglycerides has been evaluated. Options 1, 3, and 4 do not need to be monitored specifically during this treatment.

526. The nurse is applying a topical corticosteroid to a client with eczema. The nurse should monitor for the potential for increased systemic absorption if the medication were being applied to which body area? 1. Back 2. Axilla 3. Soles of the feet 4. Palms of the hands

Answer: 2 Topical corticosteroids can be absorbed into the systemic circulation. Absorption is higher from regions where the skin is especially permeable (scalp, axilla, face, eyelids, neck, perineum, genitalia), and lower from regions where permeability is poor (back, palms, soles).

521. The health education nurse provides instructions to a group of clients regarding measures that will assist in preventing skin cancer. Which instructions should the nurse provide? Select all that apply. 1. Sunscreen should be applied every 8 hours. 2. Use sunscreen when participating in outdoor activities 3. Wear a hat, opaque clothing, and sunglasses when in the sun 4. Avoid sun exposure in the late afternoon and early evening hours 5. Examine your body monthly for any lesions that may be suspicious

Answer: 2, 3, 5 The client should be instructed to avoid sun exposure between the hours of 10 am and 4 pm. Sunscreen, a hat, opaque clothing, and sunglasses should be worn for outdoor activities. The client should be instructed to examine the body monthly for the appearance of any possible cancerous or any precancerous lesions. Sunscreen should be reapplied every 2 to 3 hours and after swimming or sweating otherwise, the duration of protection is reduced.

522. Mafenide acetate is prescribed for a client with a burn injury. When applying the medication, the client complains of local discomfort and burning. The nurse should take which most appropriate action? 1. Discontinue the medication 2. Notify the health care provider 3. Inform the client that this is expected 4. Apply a thinner film than prescribed to the burn site

Answer: 3 Mafenide acetate is bacteriostatic for gram-negative and gram-positive organisms and is used to treat burns to reduce bacteria present in avascular tissues. The client should be informed that the medication will cause local discomfort and burning and that this is a normal reaction. Therefore, options 1, 2, and 4 are incorrect.

528. Silver sulfadiazine is prescribed for a client with a partial thickness burn and the nurse provides teaching about the medication. Which statement made by the client indicates a need for further teaching about the treatments? 1. "The medication is an antibacterial" 2. "The medication will help heal the burn" 3. "The medication will permanently stain my skin" 4. "The medication should be applied directly to the wound"

Answer: 3 Silver sulfadiazine is an antibacterial that has a broad spectrum of activity against gram-negative bacteria, gram-positive bacteria, and yeast. It is applied directly to the wound to assist in healing. It does not stain the skin.

529. The camp nurse asks the children preparing to swim in the lake if they have applied sunscreen. The nurse reminds the children that chemical sunscreens are most effective when applied at which times? 1. Immediately before swimming 2. 5 minutes before exposure to the sun 3. Immediately before exposure to the sun 4. At least 30 minutes before exposure to the sun

Answer: 4 Sunscreens are most effective when applied at least 30 minutes before exposure to the sun so that they can penetrate the skin. All sunscreens should be reapplied after swimming or sweating.

-zine

Antihistamine S/S: anticholenergic effects (cant see, spit, pee, poop), drowsiness NI: use cautiously pts w/ HTN, PUD, urinary retention, assess hypokalemia, BP, Advise to take @ night Common meds- diphenhydramine (benadryl), loratadine (claratin), cetirizine (zyrtec), fexofenadrine (allegra)

-tidine

Antiulcer S/S: lethargy, depression, confusion, decreased libido Common meds- ranitidine hydrochloride (zantac), cimetidine (tagamet), famotidine (pepcid)

The nurse is planning the care for a client who is admitted with the syndrome of inappropriate antidiuretic hormone secretion (SIADH). Which interventions should the nurse include in this client's plan of care? (Select all that apply.) A. Salt-free diet B. Quiet environment C. Deep tendon reflex assessments D. Neurologic checks E. Daily weights F. Unrestricted intake of free water

B, C, D, E. SIADH results in water retention and dilutional hyponatremia, which causes neurologic change when serum sodium levels are less than 115 mEq/L. The nurse should maintain a quiet environment (B) to prevent overstimulation that can lead to periods of disorientation, assess deep tendon reflexes (C) and neurologic checks (D) to monitor for neurologic deterioration. Daily weights (E) should be monitored to assess for fluid overload: 1 kg weight gain equals 1 L of fluid retention, which further dilutes serum sodium levels. (A and F) contribute to dilutional hyponatremia.

A patient is being discharged from the emergency department after being treated for epistaxis. In teaching the family first aid measures in the event the epistaxis would recur, which of the following measures would the nurse suggest? (Select all that apply.) A. Tilt patients head backwards B. Apply ice compresses to the nose C. Pinch the entire soft lower portion of the nose D. Partially insert a small gauze pad into the bleeding nostril

B,C,D First aid measures to control epistaxis includes placing the patient in a sitting position, leaning forward. Tilting the head back does not stop the bleeding, but rather allows the blood to enter the nasopharynx, which could result in aspiration or nausea/vomiting from swallowing blood. All of the other options are appropriate first aid treatment of epistaxis.

The nurse is teaching a patient how to self-administer ipratropium (Atrovent) via a metered dose inhaler. Which of the following instructions given by the nurse is most appropriate to help the patient learn proper inhalation technique? A. "Avoid shaking the inhaler before use." B. "Breathe out slowly before positioning the inhaler." C. "After taking a puff, hold the breath for 30 seconds before exhaling." D. "Using a spacer should be avoided for this type of medication."

B. "Breathe out slowly before positioning the inhaler." It is important to breathe out slowly before positioning the inhaler. This allows the patient to take a deeper breath while inhaling the medication thus enhancing the effectiveness of the dose.

A patient has been receiving oxygen per nasal cannula while hospitalized for COPD. The patient asks the nurse whether oxygen use will be needed at home. Which of the following would be the most appropriate response by the nurse? A. "Long-term home oxygen therapy should be used to prevent respiratory failure." B. "Oxygen will be needed when your oxygen saturation drops to 88% and you have symptoms of hypoxia. C. "Long-term home oxygen therapy should be used to prevent heart problems related to emphysema." D. "Oxygen will not be needed until or unless you are in the terminal stages of this disease."

B. "Oxygen will be needed when your oxygen saturation drops to 88% and you have symptoms of hypoxia.Long-term oxygen therapy in the home should be considered when the oxygen saturation is 88% or less and the patient has signs of tissue hypoxia, such as cor pulmonale, erythrocytosis, or impaired mental status.

If a health care provider is planning to transfuse a patient with a unit of packed red blood cells, which of the following solutions should the health care provider hang with the transfusion? A. 5% dextrose in water B. 0.9% sodium chloride C. 5% dextrose in 0.9% sodium chloride D. 5% dextrose in lactated Ringer's solution

B. 0.9% sodium chloride The only solution appropriate for administration with whole blood or blood products is 0.9% sodium chloride. The other options are not appropriate for use with blood products.

The nurse notes a physician's order written at 10:00 AM for 2 units of packed red blood cells to be administered to a patient who is anemic secondary to chronic blood loss. If the transfusion is picked up at 11:30, the nurse should plan to hang the unit no later than which of the following times? A. 11:45 AM B. 12:00 noon C. 12:30 PM D. 3:30 PM

B. 12:00 noon The nurse must hang the unit of packed red blood cells within 30 minutes of signing them out from the blood bank

The nurse is caring for a patient with COPD and pneumonia who has an order for arterial blood gases to be drawn. Which of the following is the minimum length of time the nurse should plan to hold pressure on the puncture site? A. 2 minutes B. 5 minutes C. 10 minutes D. 15 minutes

B. 5 minutes Following obtaining an arterial blood gas, the nurse should hold pressure on the puncture site for 5 minutes by the clock to be sure that bleeding has stopped. An artery is an elastic vessel under higher pressure than veins, and significant blood loss or hematoma formation could occur if the time is insufficient.

The nurse is interviewing a client who is taking interferon-alfa-2a (Roferon-A) and ribavirin (Virazole) combination therapy for hepatitis C. The client reports experiencing overwhelming feelings of depression. What action should the nurse implement first? A. Recommend mental health counseling. B. Review the medications actions and interactions. C. Assess for the client's daily activity level. D. Provide information regarding a support group.

B. Alpha-interferon and ribavirin combination therapy can cause severe depression. (A, B, C) may be implemented after physiological aspect of the situation are assessed.

The nurse is caring for a patient who is to receive a transfusion of two units of packed red blood cells. After obtaining the first unit from the blood bank, the nurse would ask which of the following health team members in the nurses' station to assist in checking the unit before administration? A. Unit secretary B. Another registered nurse C. A physician's assistant D. A phlebotomist

B. Another registered nurseBefore hanging a transfusion, the registered nurse must check the unit with another RN or with a licensed practical (vocational) nurse, depending on agency policy.

A 43-year-old homeless, malnourished female client with a history of alcoholism is transferred to the ICU. She is placed on telemetry, and the rhythm strip shown is obtained. The nurse palpates a heart rate of 160 beats/min, and the client's blood pressure is 90/54. Based on these finding, which IV medication should the nurse administer? A. Amiodarone (Cordarone) B. Magnesium sulfate C. Lidocaine (Xylocaine) D. Procainamide (Pronestyl)

B. Because the client has chronic alcoholism, she is likely to have hypomagnesium. (B) is the recommended drug for torsades de pointes (AHA, 2005), which is a form of polymorphic ventricular tachycardia (VT), usually associated with a prolonged QT interval that occurs with hypomagnesemia. (A and D) increase the QT interval, which can cause the torsades to worsen. (C) is the antiarrhythmic of choice in most cases of drug-induced monomorphic VT, not torsades.

A client with chronic asthma is admitted to postanesthesia complaining of pain at level 8 of 10, with a BP of 124/78, pulse of 88 beats/min, and respirations of 20 breaths/min. The postanesthesia recovery prescription is, "Morphine 2 to 4 mg IV push while in recovery for pain level over 5." What intervention should the nurse implement? A. Give the medication as prescribed to decrease the client's pain. B. Call the anesthesia provider for a different medication for pain. C. Use nonpharmacologic techniques before giving the medication. D. Reassess pain level in 30 Minutes and medicate if it remains elevated.

B. Call for a different medication because morphine and meperidine (Demerol) have histamine-releasing narcotics and should be avoided when a client has asthma. (A) puts the client at risk for asthma attack. (C & D) disregard the clients prescription and pain relief.

The nurse is caring for a 73-year-old patient who underwent a left total knee arthroplasty. On the third postoperative day, the patient complains of shortness of breath, slight chest pain, and that "something is wrong." Temperature is 98.4o F, blood pressure 130/88, respirations 36, and oxygen saturation 91% on room air. Which of the following should the nurse first suspect as the etiology of this episode? A. Septic embolus from the knee joint B. Pulmonary embolus from deep vein thrombosis C. New onset of angina pectoris D. Pleural effusion related to positioning in the operating room

B. Pulmonary embolus from deep vein thrombosis The patient presents the classic symptoms of pulmonary embolus: acute onset of symptoms, tachypnea, shortness of breath, and chest pain.

A nurse is preparing to establish oxygen therapy for a patient with COPD, and the physician's prescription reads "oxygen per nasal cannula at 5 L per minute." Which of the following actions should the nurse take? A. Administer the oxygen as prescribed. B. Call the physician and question the correct flow rate of the oxygen. C. Establish the oxygen as prescribed and obtain an ABG. D. Change the delivery device from a nasal cannula to a simple oxygen mask.

B. Call the physician and question the correct flow rate of the oxygen. The nurse should call the physician immediately and question the flow rate for delivery of the oxygen before implementation. Oxygen is used cautiously in patients with COPD because of longstanding hypoxemia serving as the respiratory drive mechanism. If high levels of oxygen are administered, the respiratory drive can be obliterated. Changing the device to a simple oxygen mask may alter the oxygen concentration being delivered to the patient and will further enhance the obliteration of the patient's respiratory drive. Obtaining an ABG sample is not a priority at this time, and the action does not address the validity of the prescribed oxygen dosing for the patient.

The nurse is reviewing the routine medications taken by a client with chronic angle closure glaucoma. Which medication prescription should the nurse question? A. An antianginal with a therapeutic effect of vasodilation. B. An anticholinergic with a side effect of pupillary dilation. C. An antihistamine with a side effect of sedation. D. A corticosteroid with a side effect of hyperglycemia.

B. Clients with angle closure glaucoma should not take medications that dilate the pupil (B) because this can precipitate acute and severely increased intraocular pressure. (A, C, D) do not cause increased intraocular pressure, which is the primary concern.

If a patient states, "It's hard for me to breathe and I feel short-winded all the time," what is the most appropriate terminology to be applied in documenting this assessment by a nurse? A. Apnea B. Dyspnea C. Tachypnea D. Respiratory fatigue

B. Dyspnea Dyspnea is a subjective description reflective of the patient's statement indicating difficulty in breathing. Apnea refers to absence of breath or breathing. Tachypnea refers to an increased rate of breathing, usually greater than 20 breaths per minute. Respiratory fatigue is subjective and usually refers to the patient exhibiting signs and symptoms associated with a comprehensive respiratory assessment including laborious breathing, use of accessory muscles, and slowing of respirations.

A client diagnosed with chronic kidney disease (CDK) 2 years ago is regularly treated at a community hemodialysis facility. In assessing the client before his scheduled dialysis treatment, which electrolyte imbalance should the nurse anticipate? A. Hypophosphatemia B. Hypocalcemia C. Hyponatremia D. Hypokalemia

B. Hypocalcemia develops in CKD due to chronic hyperphosphatemia not (A). (C & D) incorrect you would find hypernatremia and hyperkalemia

A 25-year-old client was admitted yesterday after a motor vehicle collision. Neurodiagnostic studies showed a basal skull fracture in the middle fossa. Assessment on admission revealed both halo and Battle signs. Which new symptom indicates that the client is likely to be experiencing a common life-threatening complication associated with basal skull fracture? A. Bilateral jugular vein distention. B. Oral temperature of 102 degrees F. C. Intermittent focal motor seizures. D. Intractable pain in the cervical region.

B. Increased temp indicates meningitis. (C & D) these symptoms may be exhibited but are not life threatening. (A) JVD is not a typical complication of basal skull fractures.

Which condition should the nurse anticipate as a potential problem in a female client with a neurogenic bladder? A. Stress incontinence. B. Infection. C. Painless, gross hematuria. D. Peritonitis.

B. Infection is the major complication resulting from stasis of urine and subsequent catheterization. (A) is the involuntary loss of urine through an intact urethra as a result of suddenly increased pressure. (C) is the most common symptom of bladder cancer. (D) is the most common and serious complication of peritoneal dialysis.

The nurse receives a physician's order to transfuse fresh frozen plasma to a patient suffering from an acute blood loss. Which of the following procedures is most appropriate for infusing this blood product? A. Hand the fresh frozen plasma as a piggyback to a new bag of primary IV solution without KCl. B. Infuse the fresh frozen plasma as rapidly as the patient will tolerate. C. Hang the fresh frozen plasma as a piggyback to the primary IV solution. D. Infuse the fresh frozen plasma as a piggyback to a primary solution of normal saline.

B. Infuse the fresh frozen plasma as rapidly as the patient will tolerate. The fresh frozen plasma should be administered as rapidly as possible and should be used within 2 hours of thawing. Fresh frozen plasma is infused using any straight-line infusion set. Any existing IV should be interrupted while the fresh frozen plasma is infused, unless a second IV line has been started for the transfusion.

When assigning clients on a medical-surgical floor to a RN and a LPN, it is best for the charge nurse to assign which client to the LPN? A. A child with bacterial meningitis with recent seizures. B. An older adult client with pneumonia and viral meningitis. C. A female client in isolation wiht meningococcal meningitis. D. A male client 1 day post-op after drainage of a brain abscess.

B. Is the most stable. A, C, D have an increased risk for elevated ICP.

A 58-year-old client, who has no health problems, asks the nurse about taking the pneumococcal vaccine (Pneumovax). Which statement give by the nurse would offer the client accurate information about this vaccine? A. "The vaccine is given annually before the flue season to those over 50 years of age." B. "The immunization is administered once to older adults or persons with a history of chronic illness." C. "The vaccine is for all ages and is given primarily to those person traveling overseas to infected areas." D. "The vaccine will prevent the occurrence of pneumococcal pneumonia for up to 5 years."

B. It is usually recommended that persons over 65 years of age and those with a history of chronic illness should receive the vaccine once in a lifetime. (A) the influenza vaccine is given annually. (C) travel is not the main rationale for the vaccine. (D) The vaccine is usually given once in a lifetime.

Which of the following is a factor significant in the development of anemia in men? A. Condom use B. Large hemorrhoids C. A diet high in cholesterol D. Smoking one pack of cigarettes daily

B. Large hemorrhoids Gastrointestinal (GI) tract bleeding is a common etiologic factor in men and may result from peptic ulcers, hiatal hernia, gastritis, cancer, hemorrhoids, diverticula, ulcerative colitis, or salicylate poisoning.

An older female client with dementia is transferred from a long term care unit to an acute care unit. The client's children express concern that their mother's confusion is worsening. How should the nurse respond? A. "It is to be expected that older people will experience progressive confusion." B. "Confusion in an older person often follows relocation to new surroundings." C. "The dementia is progressing rapidly, but we will do everything we can to keep your mother safe." D. "The acute care staff is not as experienced as the long-term care staff at dealing with dementia."

B. Relocation often results in confusion among older clients and is stressful to clients of all ages. (A) is an inaccurate stereotype. (C) is most likely false there are many factors that cause increased temporary confusion. (D) may be true but does not offer the family a sense of security about the care.

An older male client comes to the geriatric screening clinic complaining of pain in his left calf. The nurse notices a reddened area on the calf of his right leg that is warm to touch and the nurse suspects that the client may have thrombophlebitis. Which addition assessment is most important for the nurse to perform? A. Measure calf circumference. B. Auscultate the client's breath sounds. C. Observe for ecchymosis and petechiae. D. Obtain the client's blood pressure.

B. Since the client may have a pulmonary embolus secondary to the thrombophlebitis. A. Would support the nurses assessment. C. Least helpful since bruising is not associated with thrombophlebitis. D. Less important then auscultation.

The nurse is assigned to care for a patient in the emergency department admitted with an exacerbation of asthma. The patient has received a β-adrenergic bronchodilator and supplemental oxygen. If the patient's condition does not improve, the nurse should anticipate which of the following is likely to be the next step in treatment? A. Pulmonary function testing B. Systemic corticosteroids C. Biofeedback therapy D. Intravenous fluids

B. Systemic corticosteroids Systemic corticosteroids speed the resolution of asthma exacerbations and are indicated if the initial response to the β-adrenergic bronchodilator is insufficient.

The nurse assesses a postoperative client. Oxygen is being administered at 2 L/min and a saline lock is in place. Assessment shows cool, pale, moist skin. The client is very restless and has scant urine in the urinary drainage bag. What intervention should the nurse implement first. A. Measure urine specific gravity. B. Obtain IV fluids for infusion protocol. C. Prepare for insertion of a central venous catheter. D. Auscultate the client's breath sounds.

B. The client is at risk for hypovolemic shock and is exhibiting early signs. Start IV to restore tissue perfusion. (A, C, D) are all important but less of a priority.

A female client with a nasogastric tube attached to low suction states that she is nauseated. The nurse assesses that there has been no drainage through the nasogastric tube in the last 2 hours. Which action should the nurse take first? A. Irrigate the nasogastric tube with sterile normal saline. B. Reposition the client on her side. C. Advance the nasogastric tube 5 cm. D. Administer an intravenous antiemetic as prescribed.

B. The priority is to determined if the tube is functioning correctly, which would relieve the client's nausea. The least invasive intervention is to reposition the client (B), should be attempted first, followed by (A & C) if these are unsuccessful then (D).

A 55-year-old male client is admitted to the coronary care unit having suffered an acute myocardial infarction (MI). Within 24 hours of the occurrence, the nurse can expect to find which systemic sign? A. Elevated serum amylase level B. Elevated CM-MB level C. Prolonged prothrombin time (PT) D. Elevated serum BUN and creatinine

B. Tissue damage in the myocardium causes the release of cardiac enzymes into the blood system. An elevated CM-MB is a recognized indicator of an MI. It peaks 12 - 24 hours and returns to normal within 48 - 78 hours. (A) would indicate pancreatitis or a gastric disorder. (D) Although an elevated BUN might be related to an acute MI it is usually associated with dehydration, high protein intake or gastrointestinal bleeding and creatine levels indicate renal damage. (C) Indicates effective anticoagulation therapy.

Which description of symptoms is characteristic of a client with diagnosed with trigeminal neuralgia (tic douloureux)? A. Tinnitus, vertigo, and hearing difficulties. B. Sudden, stabbing, severe pain over the lip and chin. C. Unilateral facial weakness and paralysis. D. Difficulty in talking, chewing, and swallowing.

B. Trigeminal neuralgia is characterized by paroxysms of pain, similar to an electric shock, in the area innervated by one or more branches of the trigeminal nerve. A. Characteristic of Meniere's C. Characteristic of Bell palsey D. Characteristic of disorders of the hypoglossal (12th cranial nerve)

The nurse observes ventricular fibrillation on telemetry and upon entering the clients bathroom finds the client unconscious on the floor. What intervention should the nurse implement first? A. Administer an antidysrhythmic medication. B. Start cardiopulmonary resuscitation. C. Defibrillate the client at 200 joules. D. Assess the client's pulse oximetry.

B. Ventricular fibrillation is a life-threatening dysrhythmia and CPR should be started immediately. A & C are appropriate but B is the priority. D does not address the seriousness of the situation.

Before discharge, the nurse discusses activity levels with a 61-year-old patient with COPD and pneumonia. Which of the following exercise goals is most appropriate once the patient is fully recovered from this episode of illness? A. Slightly increase activity over the current level. B. Walk for 20 minutes a day, keeping the pulse rate less than 130 beats per minute. C. Limit exercise to activities of daily living to conserve energy. D. Swim for 10 min/day, gradually increasing to 30 min/day.

B. Walk for 20 minutes a day, keeping the pulse rate less than 130 beats per minute. The patient will benefit from mild aerobic exercise that does not stress the cardiorespiratory system. The patient should be encouraged to walk for 20 min/day, keeping the pulse rate less than 75% to 80% of maximum heart rate (220 minus patient's age).

Debilitating anginal pain can be decreased in some clients by the administration of beta-blocking agents such as nadolol (Corgard). Which client requires the nurse to use extreme caution when administering Corgard? A. A 56-year-old air traffic controller who had bypass surgery 2 years ago. B. A 47-year-old kindergarten teacher diagnosed with asthma 40 years ago C. A 52-year-old unemployed stock broker who refuses treatment for alcoholism D. A 60-year-old retired librarian who takes a diuretic daily for hypertension.

B. asthma must be carefully monitored because beta blockers because it can induce cardiogenic shock and reduce bronchodilation efforts. (A & D) this medication is indicated and (C) it is not contraindicated.

If a patient with an uncuffed tracheostomy tube coughs violently during suctioning and dislodges the tracheostomy tube, a nurse should first A. call the physician. B. attempt to reinsert the tracheostomy tube. C. position the patient in a lateral position with the neck extended. D. cover the stoma with a sterile dressing and ventilate the patient with a manual bag-mask until the physician arrives.

B. attempt to reinsert the tracheostomy tube.Retention sutures may be grasped (if present) and the tracheostomy opening spread, or a hemostat may be used to spread the opening. The obturator is inserted into the replacement tube (one size smaller than the original tube), lubricated with saline solution, and inserted into the stoma at a 45-degree angle to the neck. If the attempt is successful, the obturator tube should immediately be removed.

A nurse is reviewing the hematologic test results for a patient in whom the hematocrit (Hct) is reported at a reading of 30%. Based on this result, the nurse should interpret that the patient A. is susceptible to bleeding disorders. B. has fewer red blood cells than normal. C. is experiencing an inflammatory response. D. is experiencing an acute hemolytic crisis.

B. has fewer red blood cells than normal.The Hct is the measure of the volume of red blood cells in whole blood expressed as a percentage. This test is useful in the diagnosis of anemia, polycythemia, and abnormal hydration states. Patients who are susceptible to bleeding disorders likely will have a low platelet count. The inflammatory response may best be evaluated by examination of results that include the white blood cell count with differential analysis. Acute hemolytic crisis develops in patients receiving blood components in which incompatibility occurs or in patients with bleeding disorders or conditions that promote cellular damage, such as damage associated with shock.

Nursing interventions for the patient with aplastic anemia are directed toward the prevention of the complications of A. fatigue and dyspnea. B. hemorrhage and infection. C. thromboemboli and gangrene. D. cardiac arrhythmias and heart failure.

B. hemorrhage and infection. Hemorrhage from thrombocytopenia and infection from neutropenia are the greatest risks for the patient with aplastic anemia. The patient will experience fatigue from anemia, but bleeding and infection are the major causes of death in aplastic anemia.

A family member was taught to suction a client's tracheostomy prior to the client's discharge from the hospital. Which observation by the nurse indicates that the family member is capable of correctly performing the suctioning technique? A. Turns on the continuous wall suction to -190 mm Hg B. Inserts the catheter until resistance or coughing occurs C. Withdraws the catheter while maintaining suctioning D. Re-clears the tracheostomy after suctioning the mouth

B. indicates correct technique for performing suctioning. Suction pressure should be between -80 and -120 (A). The catheter should be withdrawn 1-2 cm at a time with intermittent suction (C). (D) introduces pathogens.

In preparing the preoperative teaching plan for a patient who is to undergo a total laryngectomy, a nurse should give highest priority to the A. tracheostomy being in place for 2 to 3 days. B. patient's not being able to speak normally again. C. insertion of a gastrostomy feeding tube during surgery. D. patient's not being able to perform deep-breathing exercises.

B. patient's not being able to speak normally again. Patients who have a total laryngectomy have a permanent tracheostomy and will need to learn how to speak using alternative methods, such as an artificial larynx. The tracheostomy will be permanent to allow normal breathing patterns and air exchange. After surgery, the patient's nutrition is supplemented with enteral feedings, and when the patient can swallow secretions, oral feedings can begin. Deep-breathing exercises should be performed with the patient at least every 2 hours to prevent further pulmonary complications.

A patient with hemophilia is hospitalized with acute knee pain and swelling. An appropriate nursing intervention for the patient includes A. wrapping the knee with an elastic bandage. B. placing the patient on bed rest and applying ice to the joint. C. gently performing range-of-motion (ROM) exercises to the knee to prevent adhesions. D. administering nonsteroidal anti-inflammatory drugs (NSAIDs) as needed for pain.

B. placing the patient on bed rest and applying ice to the joint. During an acute bleeding episode in a joint, it is important to totally rest the involved joint and slow bleeding with application of ice. Drugs that decrease platelet aggregation, such as aspirin or NSAIDs, should not be used for pain. As soon as bleeding stops, mobilization of the affected area is encouraged with range-of-motion (ROM) exercises and physical therapy.

The resurgence in TB resulting from the emergence of multidrug-resistant strains of Mycobacterium tuberculosis is primarily the result of A. a lack of effective means to diagnose TB. B. poor compliance with drug therapy in patients with TB. C. the increased population of immunosuppressed individuals with AIDS. D. indiscriminate use of antitubercular drugs in treatment of other infections.

B. poor compliance with drug therapy in patients with TB. Drug-resistant strains of TB have developed because TB patients' compliance to drug therapy has been poor and there has been general decreased vigilance in monitoring and follow-up of TB treatment. Antitubercular drugs are almost exclusively used for TB infections. TB can be effectively diagnosed with sputum cultures. The incidence of TB is at epidemic proportions in patients with HIV, but this does not account for drug-resistant strains of TB.

A patient admitted to the emergency department with tension pneumothorax and mediastinal shift following an automobile crash is most likely to exhibit A. bradycardia. B. severe hypotension. C. mediastinal flutter. D. a sucking chest wound.

B. severe hypotension. Mediastinal shift may cause compression of the lung in the direction of the shift and compression, traction, torsion, or kinking of the great vessels. Blood return to the heart is dangerously impaired and causes a subsequent decrease in cardiac output and blood pressure. Tachycardia is a clinical manifestation of tension pneumothorax. An uncovered opened pneumothorax is associated with a sucking chest wound and mediastinal flutter.

Select all that apply. Atelectasis can be caused by A. long-term smoking. B. inadequate surfactant. C. localized airway obstruction. D. an increase in lung expansion. E. an increase in elastic recoil.

BCE The collapse of lung tissue has several causes, including reduced lung expansion, localized airway obstruction, inadequate surfactant, and an increase in elastic recoil. Smoking, although harmful, does not in itself cause atelectasis.

Select all that apply. Which of the following are significant risk factors for leukemia? A. Being a longtime smoker B. Employment in an oil refinery C. History of hemophilia in parent D. Having Down syndrome E. Having a twin brother with leukemia F. Treatment with an alkylating agent = 3 years ago

BDEF Exposure to chemical agents, treatment with alkylating cancer drugs, leukemia in a sibling, and the patient's having Down syndrome are all risk factors for leukemia.

-pam, -lam

Benzodiazipines TX: Sedative-hypnotics for sleep, Adjuncts to anesthesia to induce relaxation and amnesia (procedural memory loss), To reduce anxiety (anxiolytic), Panic disorders, To treat or prevent seizures, For alcohol withdrawal, Muscle relaxant

catopril, lisinopril, enalapril (vastotec)

Block the conversion of angiotensin I to angiotensin II TX: HTN, HF, MI, diabetic nephropathy S/S: Anigoedema, Cough, Electrolyte imbalance (^k+) NI: Monitor K+ levels, BP

albuterol

Bronchodilator S/S: tachcardia, palpitations, tremors

-phylline, -terol

Bronchodilator S/S: tachcardia, palpitations, tremors Common meds- albeuterol

39. Select all that apply. Which of the following are clinical manifestations of tension pneumothorax? A. Midline trachea B. Severe hypertension C. Progressive cyanosis D. A loud bruit on affected side E. Asymmetrical chest wall movement F. Subcutaneous emphysema in the neck

C,E, F The indicators of tension pneumothorax are asymmetrical chest wall movement, severe hypotension, subcutaneous emphysema in the neck and upper chest, and progressive cyanosis.

Which of the following statements made by a patient with COPD indicates a need for further education regarding the use of an ipratropium inhaler? A. "I should rinse my mouth following the two puffs to get rid of the bad taste." B. "I should wait at least 1 to 2 minutes between each puff of the inhaler." C. "If my breathing gets worse, I should keep taking extra puffs of the inhaler until I can breathe more easily." D. "Because this medication is not fast-acting, I cannot use it in an emergency if my breathing gets worse.

C. "If my breathing gets worse, I should keep taking extra puffs of the inhaler until I can breathe more easily." The patient should not take extra puffs of the inhaler at will to make breathing easier. Excessive treatment could trigger paradoxical bronchospasm, which would worsen the patient's respiratory status.

Which of the following statements made by a nurse would indicate proper teaching principles regarding feeding and tracheostomies? A. "Follow each spoon of food consumed with a drink of fluid." B. "Thin your foods to a liquid consistency whenever possible." C. "Tilt your chin forward toward the chest when swallowing your food." D. "Make sure your cuff is overinflated before eating if you have swallowing problems."

C. "Tilt your chin forward toward the chest when swallowing your food." A nurse should instruct a patient to tilt the chin toward the chest, which will close the glottis and allow food to enter the normal passageway. Ideally, foods should be of a thick consistency to enable effective swallowing and reduce the risk of aspiration. Overinflation of the cuff causes swallowing difficulties. Fluids should be consumed in small amounts after swallowing to prevent the risk of aspiration.

When preparing to administer an ordered blood transfusion, the nurse selects which of the following intravenous solutions to use when priming the blood tubing? A. 5% dextrose in water B. Lactated Ringer's C. 0.9% sodium chloride D. 0.45% sodium chloride

C. 0.9% sodium chloride The blood set should be primed before the transfusion with 0.9% sodium chloride, also known as normal saline. It is also used to flush the blood tubing after the infusion is complete to ensure the patient receives blood that is left in the tubing when the bag is empty.

The home health nurse is assessing a male client being treated for Parkinson disease with levodopa-carbidopa (Sinemet). The nurse observes that he does not demonstrate any apparent emotions when speaking and rarely blinks. Which intervention should the nurse implement? A. Perform a complete cranial nerve assessment. B. Instruct the client that he may be experiencing medication toxicity. C. Document the presence of these assessment findings. D. Advise the client to seek immediate medical evaluation.

C. A mask-like expression and infrequent blinking are common clinical features of Parkinsonism. The nurse should document the findings. (A & D) are not necessary. Signs of toxicity (B) are dyskinesia, hallucinations, and psychosis.

A central venous catheter has been inserted via a jugular vein and a radiography has confirmed placement of the catheter. A prescription has been received for stat medication but IV fluids have not yet been started. What action should the nurse take prior to administering the prescribed medication? A. Assess for signs of jugular vein distention. B. Obtain the needed intravenous solution. C. Administer a bolus of normal saline solution. D. Flush the line with heparinized saline.

C. A medication can be administered central line without IV fluids, flush with normal saline to remove heparin that may counteract with the medication. (B) is used following the medication and a second saline bolus. (A) will not impact the the med administration and is not a priority. (B) Administration of the stat medication is more of a priority than (B).

The nurse evaluates that nursing interventions to promote airway clearance in a patient admitted with COPD are successful based on which of the following findings? A. Absence of dyspnea B. Improved mental status C. Effective and productive coughing D. PaO2 within normal range for the patient

C. Effective and productive coughing The issue of the question is airway clearance, which is most directly evaluated as successful if the patient can engage in effective and productive coughing.

Nursing assessment findings of jugular vein distention and pedal edema would be indicative of which of the following complications of emphysema? A. Acute respiratory failure B. Pulmonary edema caused by left-sided heart failure C. Fluid volume excess secondary to cor pulmonale D. Secondary respiratory infection

C. Fluid volume excess secondary to cor pulmonale Cor pulmonale is a right-sided heart failure caused by resistance to right ventricular outflow due to lung disease. With failure of the right ventricle, the blood emptying into the right atrium and ventricle would be slowed, leading to jugular venous distention and pedal edema.

During the change of shift report, the charge nurse reviews the infusions being received by the clients on the oncology unit. The client receiving which infusion should be seen first?

C. Has the highest risk for respiratory depression and therefor should be seen first. (A) Risk of hypotension. (B) Lowest risk. (D) Risk of nephrotoxicity and phlebitis.

Which of the following positions is most appropriate for the nurse to place a patient experiencing an asthma exacerbation? A. Supine B. Lithotomy C. High-Fowler's D. Reverse Trendelenburg

C. High-Fowler'sThe patient experiencing an asthma attack should be placed in high-Fowler's position to allow for optimal chest expansion and enlist the aid of gravity during inspiration.

The nurse know that normal lab values expected for an adult may vary in an older client. Which data would the nurse expect to find when reviewing laboratory values of an 80-year-old man who is in good overall health. A. Complet blood count reveals increased WBC and decreased RBC counts. B. Chemistries reveal an increased serum bilirubin with slightly increased liver enzymes. C. Urinalysis reveals slight protein in the urine and bacteriuria with pyuria. D. Serum electrolytes reveal a decreased sodium level with an increased potassium level.

C. In older adults the protein found in urine is slightly risen as a result of kidney changes or subclinical UTIs and the client frequently experiences asymptomatic bacteriuria and pyuria as a result of incomplete bladder emptying. (A, B, D) are not normal findings.

Which of the following clinical manifestations would the nurse expect to find during assessment of a patient admitted with pneumococcal pneumonia? A. Hyperresonance on percussion B. Fine crackles in all lobes on auscultation C. Increased vocal fremitus on palpation D. Vesicular breath sounds in all lobes

C. Increased vocal fremitus on palpation. A typical physical examination finding for a patient with pneumonia is increased vocal fremitus on palpation. Other signs of pulmonary consolidation include dullness to percussion, bronchial breath sounds, and crackles in the affected area.

While teaching a patient with asthma about the appropriate use of a peak flow meter, the nurse instructs the patient to do which of the following? A. Use the flow meter each morning after taking medications to evaluate their effectiveness. B. Empty the lungs and then inhale quickly through the mouthpiece to measure how fast air can be inhaled. C. Keep a record of the peak flow meter numbers if symptoms of asthma are getting worse. D. Increase the doses of the long-term control medication if the peak flow numbers decrease.

C. Keep a record of the peak flow meter numbers if symptoms of asthma are getting worse. It is important to keep track of peak flow readings daily and when the patient's symptoms are getting worse. The patient should have specific directions as to when to call the physician based on personal peak flow numbers. Peak flow is measured by exhaling into the meters and should be assessed before and after medications to evaluate their effectiveness.

What type of anemia is associated with folate deficiency? A. Microcytic B. Pernicious C. Megaloblastic D. Iron deficiency

C. Megaloblastic Megaloblastic anemia is nutritional anemia; large immature red blood cells with a decreased oxygen-carrying capacity can occur as a result of impaired DNA synthesis. Folic acid is used in the synthesis of DNA and helps convert B12 to coenzyme form. Folic acid is needed for growth and development of red blood cells. Microcytic anemia is anemia with abnormally small erythrocytes (red blood cells) in Hb. This anemia is associated with vitamin B6 (pyridoxine) deficiency. Pernicious anemia is caused by a deficiency of vitamin B12. Iron-deficiency anemia results from loss of blood or deficient intake of iron foods or disease states in which the body does not absorb or utilize iron as it should.

During assessment of a 45-year-old patient with asthma, the nurse notes wheezing and dyspnea. The nurse interprets that these symptoms are related to which of the following pathophysiologic changes? A. Laryngospasm B. Overdistention of the alveoli C. Narrowing of the airway D. Pulmonary edema

C. Narrowing of the airwayNarrowing of the airway leads to reduced airflow, making it difficult for the patient to breathe and producing the characteristic wheezing.

While ambulating a patient with metastatic lung cancer, the nurse observes a drop in oxygen saturation from 93% to 86%. Which of the following nursing interventions is most appropriate based upon these findings? A. Continue with ambulation as this is a normal response to activity. B. Move the oximetry probe from the finger to the earlobe for more accurate monitoring during activity. C. Obtain a physician's order for supplemental oxygen to be used during ambulation and other activity. D. Obtain a physician's order for arterial blood gas determinations to verify the oxygen saturation.

C. Obtain a physician's order for supplemental oxygen to be used during ambulation and other activity. An oxygen saturation level that drops below 90% with activity indicates that the patient is not tolerating the exercise and needs to have supplemental oxygen applied.

When assessing a patient's respiratory status, which of the following nonrespiratory data are most important for the nurse to obtain? A. Height and weight B. Neck circumference C. Occupation and hobbies D. Usual daily fluid intake

C. Occupation and hobbiesMany respiratory problems occur as a result of chronic exposure to inhalation irritants. Common occupational sources of inhalation irritants include mines, granaries, farms, lawn care companies, paint, plastics and rubber manufacture, and building remodeling. Hobbies associated with inhalation irritants include woodworking, metal finishing, furniture refinishing, painting, and ceramics. Daily fluids, height, and weight are more related to respiratory problems secondary to cardiac issues.

jaundice

serium amylase & lipase 2 times higher than normal indicate pancreatic injury frothy tea colored urine clay colored stools complaints of puritis

The nurse determines that a patient is experiencing common adverse effects from the inhaled corticosteroid beclomethasone (Beclovent) after noting which of the following? A. Adrenocortical dysfunction and hyperglycemia B. Elevation of blood glucose and calcium levels C. Oropharyngeal candidiasis and hoarseness D. Hypertension and pulmonary edema

C. Oropharyngeal candidiasis and hoarseness Oropharyngeal candidiasis and hoarseness are common adverse effects from the use of inhaled corticosteroids because the medication can lead to overgrowth of organisms and local irritation if the patient does not rinse the mouth following each dose.

The nurse is completing an admission inter for a client with Parkinson disease. Which question will provide addition information about manifestations the client is likely to experience? A. "Have you ever experienced and paralysis of your arms or legs?" B. " Do you have frequent blackout spells?" C. "Have you ever been 'frozen' in one spot, unable to move?" D. "Do you have headaches, especially ones with throbbing pain?"

C. Parkinson clients frequently experience difficulty in initiating, maintaining, and performing motor activities. They may even experience being rooted, unable to move. (A, B, D) Does not typically occur in Parkinson.

When reviewing the results of a 83-year-old patient's blood tests, which of the following findings would be of most concern to the nurse? A. Platelets of 150,000/µl B. Serum iron of 50 mcg/dl C. Partial thromboplastin time (PTT) of 60 seconds D. Erythrocyte sedimentation rate (ESR) of 35 mm in 1 hour

C. Partial thromboplastin time (PTT) of 60 seconds In aging, the partial thromboplastin time (PTT) is normally decreased, so an abnormally high PTT of 60 seconds is an indication that bleeding could readily occur. Platelets are unaffected by aging, and 150,000 is a normal count. Serum iron levels are decreased and the erythrocyte sedimentation rate (ESR) is significantly increased with aging, as are reflected in these values.

Which of the following conditions or factors in a 64-year-old patient diagnosed with head and neck cancer most likely contributed to this health problem? A. Patient's hobby is oil painting. B. Patient's father also had head and neck cancer. C. Patient uses chewing tobacco and drinks beer daily. D. Patient quit school at age 16 and has worked in a butcher shop for more than 40 years.

C. Patient uses chewing tobacco and drinks beer daily. Many environmental risk factors contribute to the development of head and neck cancer, although the actual cause is unknown. There does not appear to be a genetic predisposition to this type of cancer. The two most important risk factors are tobacco and alcohol use, especially in combination. Other risk factors include chewing tobacco, pipe smoking, marijuana use, voice abuse, chronic laryngitis, exposure to industrial chemicals or hardwood dust, and poor oral hygiene.

A 71-year-old patient is admitted with acute respiratory distress related to cor pulmonale. Which of the following nursing interventions is most appropriate during admission of this patient? A. Delay any physical assessment of the patient and review with the family the patient's history of respiratory problems. B. Perform a comprehensive health history with the patient to review prior respiratory problems. C. Perform a physical assessment of the respiratory system and ask specific questions related to this episode of respiratory distress. D. Complete a full physical examination to determine the effect of the respiratory distress on other body functions.

C. Perform a physical assessment of the respiratory system and ask specific questions related to this episode of respiratory distress.Because the patient is having respiratory difficulty, the nurse should ask specific questions about this episode and perform a physical assessment of this system. Further history taking and physical examination of other body systems can proceed once the patient's acute respiratory distress is being managed.

During discharge teaching for a 65-year-old patient with emphysema and pneumonia, which of the following vaccines should the nurse recommend the patient receive? A. S. aureus B. H. influenzae C. Pneumococcal D. Bacille Calmette-Guérin (BCG)

C. Pneumococcal The pneumococcal vaccine is important for patients with a history of heart or lung disease, recovering from a severe illness, age 65 or over, or living in a long-term care facility.

Which of the following conditions is manifested by unexplained shortness of breath and a high mortality rate? A. Bleeding ulcer B. Transient ischemia C. Pulmonary embolism D. MI

C. Pulmonary embolism A high mortality rate is associated with a pulmonary embolism. A pulmonary embolism is an obstruction of the pulmonary artery caused by an embolus. It presents with hypoxia, anxiety, restlessness, and shortness of breath. Bleeding ulcers, MI, and transient ischemia are not associated with such a high mortality rate.

The nurse is caring for a critically ill client with cirrhosis of the liver who has a nasogastric tube draining bright red blood. The nurse notes that the client's serum hemoglobin and hematocrit are decreased. What additional change in lab data should the nurse expect? A. Increased serum albumin B. Decreased serum creatinine C. Decreased serum ammonia D. Increased liver function tests

C. The breakdown of glutamine in the intestine and the increased activity of colonic bacteria from the digestion of proteins increases the ammonia levels in the clients with advanced liver disease, so removal of blood, a protein source, from the intestines results in reduced ammonia. (A, B, D) will not be significantly impacted by the removal of blood.

What is the correct procedure for performing an ophthalmoscopic examination on a client's right eye? A. Instruct the client to look at the examiner's nose and not move his/her eyes during the exam. B. Set ophthalmoscope on the plus 2 to 3 lens and hold it in front of the examiner's right eye. C. From a distance of 8 to 12 inches and slightly to the side, shine the light into the client's pupil. D. For optimum visualization, keep the ophthalmoscope at least 3 inches for the client's eye

C. The client should focus on a distant object in order to promote pupil dilation. The ophthalmoscope should be set on the 0 lens to begin (creates no correction) and should be held in front of the examiner's left eye when examining the client's right eye and kept 1" from the client's eye for optimum visualization. (A, B, D) are incorrect procedures.

During report, the nurse learns that a client with tumor lysis syndrome is receiving an IV infusion containing insulin. Which assessment should the nurse complete first? A. Review the client's history for diabetes mellitus. B. Observe the extremity distal to the IV site. C. Monitor the client's serum potassium and blood glucose. D. Evaluate the client's oxygen saturation and breath sounds.

C. The client with tumor lysis syndrome may experience hyperkalemia, therefor it is important to monitor serum potassium and blood glucose levels. (A, B, D) are not as priority.

If a nurse is assessing a patient whose recent blood gas determination indicated a pH of 7.32 and respirations are measured at 32 breaths/min, which of the following is the most appropriate nursing assessment? A. The rapid breathing is causing the low pH. B. The nurse should sedate the patient to slow down respirations. C. The rapid breathing is an attempt to compensate for the low pH. D. The nurse should give the patient a paper bag to breathe into to correct the low pH.

C. The rapid breathing is an attempt to compensate for the low pH. The respiratory system influences pH (acidity) through control of carbon dioxide exhalation. Thus, rapid breathing increases the pH. Breathing into a paper bag aids a patient who is hyperventilating; in respiratory alkalosis, it aids in lowering the pH. The use of sedation can cause respiratory depression and hypoventilation, resulting in an even lower pH.

A patient with an acute pharyngitis is seen at the clinic with fever and severe throat pain that affects swallowing. On inspection the throat is reddened and edematous with patchy yellow exudates. The nurse anticipates that collaborative management will include A. treatment with antibiotics. B. treatment with antifungal agents. C. a throat culture or rapid strep antigen test. D. treatment with medication only if the pharyngitis does not resolve in 3 to 4 days.

C. a throat culture or rapid strep antigen test. Although inadequately treated β-hemolytic streptococcal infections may lead to rheumatic heart disease or glomerulonephritis, antibiotic treatment is not recommended until strep infections are definitely diagnosed with culture or antigen tests. The manifestations of viral and bacterial infections are similar, and appearance is not diagnostic except when candidiasis is present.

An excess of carbon dioxide in the blood causes an increased respiratory rate and volume because CO2 A. displaces oxygen on hemoglobin, leading to a decreased PaO2. B. causes an increase in the amount of hydrogen ions available in the body. C. combines with water to form carbonic acid, lowering the pH of cerebrospinal fluid. D. directly stimulates chemoreceptors in the medulla to increase respiratory rate and volume.

C. combines with water to form carbonic acid, lowering the pH of cerebrospinal fluid. A combination of excess CO2 and H2O results in carbonic acid, which lowers the pH of the cerebrospinal fluid and stimulates an increase in the respiratory rate. Peripheral chemoreceptors in the carotid and aortic bodies also respond to increases in PaCO2 to stimulate the respiratory center. Excess CO2 does not increase the amount of hydrogen ions available in the body but does combine with the hydrogen of water to form an acid.

Following a supraglottic laryngectomy, the patient is taught how to use the supraglottic swallow to minimize the risk of aspiration. In teaching the patient about this technique, the nurse instructs the patient to A. perform Valsalva maneuver immediately after swallowing. B. breathe between each Valsalva maneuver and cough sequence. C. cough after swallowing to remove food from the top of the vocal cords. D. practice swallowing thin, watery fluids before attempting to swallow solid foods.

C. cough after swallowing to remove food from the top of the vocal cords. A supraglottic laryngectomy involves removal of the epiglottis and false vocal cords, and the removal of the epiglottis allows food to enter the trachea. Supraglottic swallowing requires performance of the Valsalva maneuver before placing food in the mouth and swallowing. The patient then coughs to remove food from the top of the vocal cords, swallows again, and then breathes after the food has been removed from the vocal cords.

If a patient has pernicious anemia, the nurse should provide information regarding A. frequent bouts of dyspnea. B. risks relative to dehydration. C. deficiency of intrinsic factor. D. lack of any effective treatment for this condition.

C. deficiency of intrinsic factor. Pernicious anemia is a type of anemia caused by failure of absorption of vitamin B12 (cobalamin). The most common cause is lack of intrinsic factor, a glucoprotein produced by the parietal cells of the gastric lining.

The chronic inflammation of the bronchi characteristic of chronic obstructive pulmonary disease (COPD) results in A. collapse of small bronchioles on expiration. B. permanent, abnormal dilation of the bronchi. C. hyperplasia of mucus-secreting cells and bronchial edema. D. destruction of the elastic and muscular structures of the bronchial wall.

C. hyperplasia of mucus-secreting cells and bronchial edema. Chronic bronchitis is characterized by chronic inflammation of the bronchial lining, with edema and increased mucus production. Collapse of small bronchioles on expiration is common in emphysema, and abnormal dilation of the bronchi because of destruction of the elastic and muscular structures is characteristic of bronchiectasis.

A person complains of fatigue and malaise and has a slight temperature elevation for 2 days before symptoms of influenza (fever, chest congestion, and productive cough) become noticeable. During the time immediately before the illness is diagnosed, the patient A. could avoid contracting the disease if treatment is begun with antibiotics. B. is unable to spread the disease because it is still in the incubation period. C. is in the prodromal stage and is highly contagious and able to spread the disease. D. has a nosocomial infection, which affects approximately two million individuals a year.

C. is in the prodromal stage and is highly contagious and able to spread the disease. The prodromal stage is a short period of time (hours to several days) immediately preceding the onset of an illness during which the patient is very contagious. Antibiotics are not effective against viral illnesses. The incubation period is the time from entry of the organism to the onset of symptoms and, in some viral illnesses, may be contagious. Nosocomial infections are those acquired in a hospital, and this scenario does not suggest the source of the infection.

During care of a patient with multiple myeloma, an important nursing intervention is A. limiting activity to prevent pathologic fractures. B. assessing for changes in size and characteristics of lymph nodes. C. maintaining a fluid intake of 3 to 4 L/day to dilute calcium load. D. administering narcotic analgesics continuously to control bone pain.

C. maintaining a fluid intake of 3 to 4 L/day to dilute calcium load. Adequate hydration must be maintained to minimize problems from hypercalcemia. The goal of a urinary output of 1.5 to 2 L/day requires an intake of 3 to 4 L/day.

Absorption of vitamin B12 may be decreased in older adults because of decreased A. intestinal motility. B. production of bile by the liver. C. production of intrinsic factor by the stomach. D. synthesis of cobalamin (vitamin B12) by intestinal bacteria.

C. production of intrinsic factor by the stomach. Older persons are at risk for deficiency of cobalamin (pernicious anemia) because of a naturally occurring reduction of the intrinsic factor by the stomach mucosa. Absorption of cobalamin relies on intrinsic factor. Both must be present for absorption. Megaloblastic anemia is related to folate dysfunction. Intestinal motility (peristalsis) is the motion that moves food down the GI tract. The rhythmic contractions of muscles cause wave-like motions. Lack of peristalsis is called "paralytic ileus." Bile is produced in the liver, is stored and concentrated in the gallbladder, and is released into the duodenum when fat is eaten. Bile emulsifies fats and prepares them for enzyme digestion in order for the nutrient to be absorbed into lymph and eventually into blood vessels to the liver. Vitamin K (the blood-clotting vitamin) is synthesized by intestinal bacteria.

In older adults, infection after exposure to respiratory illness is most likely to A. result in similar rates of infection as in the younger adult. B. be easily prevented with the use of antibiotics after being exposed. C. result in serious lower respiratory infection related to weakened respiratory muscles and fewer cilia. D. be less serious because the older adult has less contact with younger children who are most likely to carry serious infections.

C. result in serious lower respiratory infection related to weakened respiratory muscles and fewer cilia. Changes in the older adult respiratory system make older adults more susceptible to infections that can be very serious and life threatening. Use of antibiotics to "prevent" lung infections is not recommended and is ineffective for viral infections.

In teaching the patient with COPD about the need for physical exercise, the nurse informs the patient that A. all patients with COPD should be able to increase walking gradually up to 20 min/day. B. a bronchodilator inhaler should be used to relieve exercise-induced dyspnea immediately after exercise. C. shortness of breath is expected during exercise but should return to baseline within 5 minutes after the exercise. D. monitoring the heart rate before and after exercise is the best way to determine how much exercise can be tolerated.

C. shortness of breath is expected during exercise but should return to baseline within 5 minutes after the exercise.Shortness of breath usually increases during exercise, but the activity is not being overdone if breathing returns to baseline within 5 minutes after stopping. Bronchodilators can be administered 10 minutes before exercise but should not be administered for at least 5 minutes after activity to allow recovery. Patients are encouraged to walk 15 to 20 minutes a day with gradual increases, but actual patterns will depend on patient tolerance. Dyspnea most frequently limits exercise and is a better indication of exercise tolerance than is heart rate in the patient with COPD.

Upon entering the room of a patient who has just returned from surgery for total laryngectomy and radical neck dissection, a nurse should recognize a need for intervention when finding A. a gastrostomy tube that is clamped. B. the patient coughing blood-tinged secretions from the tracheostomy. C. the patient positioned in a lateral position with the head of the bed flat. D. 200 ml of serosanguineous drainage in the patient's portable drainage device.

C. the patient positioned in a lateral position with the head of the bed flat. After total laryngectomy and radical neck dissection, a patient should be placed in a semi-Fowler's position to decrease edema and limit tension on the suture line.

The nurse evaluates that discharge teaching for a patient hospitalized with pneumonia has been most effective when the patient states which of the following measures to prevent a relapse? A. "I will increase my food intake to 2400 calories a day to keep my immune system well." B. "I must use home oxygen therapy for 3 months and then will have a chest x-ray to reevaluate." C. "I will seek immediate medical treatment for any upper respiratory infections." D. "I should continue to do deep-breathing and coughing exercises for at least 6 weeks."

D. "I should continue to do deep-breathing and coughing exercises for at least 6 weeks." It is important for the patient to continue with coughing and deep breathing exercises for 6 to 8 weeks until all of the infection has cleared from the lungs. A patient should seek medical treatment for upper respiratory infections that persist for more than 7 days. Increased fluid intake, not caloric intake, is required to liquefy secretions. Home O2 is not a requirement unless the patient's oxygenation saturation is below normal.

The nurse evaluates that teaching for the patient with iron deficiency anemia has been effective when the patient states A. "I will need to take the iron supplements the rest of my life." B. "I will increase my dietary intake of milk and milk products." C. "I should increase my activity to increase my aerobic capacity." D. "I should take the iron for several months after my blood is normal."

D. "I should take the iron for several months after my blood is normal." To replace the body's iron stores, iron supplements should be continued for 2 to 3 months after the Hb level returns to normal, but if the cause of the iron deficiency is corrected, the supplements do not need to be taken for a lifetime. Milk and milk products are poor sources of dietary iron. Activity should be gradually increased as Hb levels return to normal because aerobic capacity can be increased when adequate Hb is available.

The nurse is teaching a patient who is to undergo bone marrow aspiration. Which of the following statements made by the nurse would indicate correct instruction regarding the site for the aspiration procedure? A. "The health care provider will perform the aspiration by needle to the femur." B. "The health care provider will perform the aspiration by needle to the scapula." C. "The health care provider will perform the aspiration by needle to the antecubital fossa." D. "The health care provider will perform the aspiration by needle to the posterior iliac crest."

D. "The health care provider will perform the aspiration by needle to the posterior iliac crest." Bone marrow samples are commonly taken from the posterior iliac crest or, as an alternative, the sternum may be aspirated. These sites provide relative ease in accessing the bone marrow via the biopsy needle. The antecubital fossa, femur, and scapula do not allow access to bone marrow while also providing reduced risk of harm to the patient.

Before starting a transfusion of packed red blood cells for an anemic patient, the nurse would arrange for a peer to monitor his or her other assigned patients for how many minutes when the nurse begins the transfusion? A. 60 B. 5 C. 30 D. 15

D. 15 As part of standard procedure, the nurse remains with the patient for the first 15 minutes after hanging a blood transfusion. Patients who are likely to have a transfusion reaction will more often exhibit signs within the first 15 minutes that the blood is infusing.

The arterial blood gas (ABG) readings that indicate compensated respiratory acidosis are a PaCO2 of A. 30 mm Hg and bicarbonate level of 24 mEq/L. B. 30 mm Hg and bicarbonate level of 30 mEq/L. C. 50 mm Hg and bicarbonate level of 20 mEq/L. D. 50 mm Hg and bicarbonate level of 30 mEq/L.

D. 50 mm Hg and bicarbonate level of 30 mEq/L. If compensation is present, carbon dioxide and bicarbonate are abnormal (or nearly so) in opposite directions (e.g., one is acidotic and the other alkalotic).

The nurse know that a client taking diuretics must be assessed for the development of hypokalemia, and that hypokalemia will create changes in the client's normal ECG tracing. Which ECG change would be an expected finding in the client with hypokalemia? A. Tall, spiked T waves B. A prolonged QT interval C. A widening QRS complex D. Presence of a U wave

D. A U wave is a positive deflection following the T wave and is often present with hypokalemia. A, B, C indicate hyperkalemia.

A patient with COPD is receiving oxygen at 2 L/min. While in the supine position for a bath, the patient complains of shortness of breath. What is the most appropriate first nursing action? A. Increase the flow of oxygen. B. Perform tracheal suctioning. C. Report this to the physician. D. Assist the patient to Fowler's position.

D. Assist the patient to Fowler's position. Breathing is easier in Fowler's position because it permits greater expansion of the chest cavity. If repositioning does not improve the situation, then oxygenation and physician reporting might be appropriate. The patient would not benefit from tracheal suctioning.

The nurse is scheduled to give a dose of ipratropium bromide by metered dose inhaler. The nurse would administer the right drug by selecting the inhaler with which of the following trade names? A. Vanceril B. Pulmicort C. AeroBid D. Atrovent

D. Atrovent The trade or brand name for ipratropium bromide, an anticholinergic medication, is Atrovent.

Which of the following physical assessment findings in a patient with pneumonia best supports the nursing diagnosis of ineffective airway clearance? A. Oxygen saturation of 85% B. Respiratory rate of 28 C. Presence of greenish sputum D. Basilar crackles

D. Basilar crackles The presence of adventitious breath sounds indicates that there is accumulation of secretions in the lower airways. This would be consistent with a nursing diagnosis of ineffective airway clearance because the patient is retaining secretions.

Respiratory acidosis is at highest risk in a patient with A. hypokalemia. B. pulmonary fibrosis. C. salicylate overdose. D. COPD.

D. COPD. Chronic respiratory acidosis is most commonly caused by COPD. Pulmonary fibrosis, hypokalemia, and salicylate overdose do not predispose a patient to respiratory acidosis. Hypokalemia can lead to cardiac dysrhythmias. Salicylate overdose results in central nervous system changes, and pulmonary fibrosis can result in respiratory arrest.

A client who is receiving an ACE inhibitor for hypertension calls the clinic and reports the recent onset of a cough to the nurse. What action should the nurse implement? A. Advise the client to come to the clinic immediately for further assessment. B. Instruct the client to discontinue use of the drug, and make an appointment at the clinic. C. Suggest that the client lear to accept the cough as a side effect to a necessary prescription. D. Encourage the client to keep taking the drug until seen by the HCP.

D. Cough is a common s/e of ACE inhibitors and is not an indication to discontinue the medication. (A) immediate evaluation is not needed. (B) an antihypertensive should not be stopped abruptly. (C) is demeaning since the cough may be disruptive to the client and other medications may produce results without the s/e.

The nurse is assigned to care for a patient who has anxiety and an exacerbation of asthma. Which of the following is the primary reason for the nurse to carefully inspect the chest wall of this patient? A. Observe for signs of diaphoresis B. Allow time to calm the patient C. Monitor the patient for bilateral chest expansion D. Evaluate the use of intercostal muscles

D. Evaluate the use of intercostal muscles The nurse physically inspects the chest wall to evaluate the use of intercostal (accessory) muscles, which gives an indication of the degree of respiratory distress experienced by the patient.

Which of the following foods is high in iron? A. Citrus fruits B. Milk products C. Yellow vegetables D. Green leafy vegetables

D. Green leafy vegetables Green leafy vegetables are high in iron. Foods cooked in iron pots and foods such as liver (the richest source), oysters, lean meats, kidney beans, whole wheat bread, kale, spinach, egg yolk, turnip tops, beet greens, carrots, apricots, and raisins are also high in iron.

When planning patient teaching about emphysema, the nurse understands that the symptoms of emphysema are caused by which of the following? A. Hypertrophy and hyperplasia of goblet cells in the bronchi B. Collapse and hypoventilation of the terminal respiratory unit C. An overproduction of the antiprotease alpha1-antitrypsin D. Hyperinflation of alveoli and destruction of alveolar walls

D. Hyperinflation of alveoli and destruction of alveolar walls In emphysema, there are structural changes that include hyperinflation of alveoli, destruction of alveolar walls, destruction of alveolar capillary walls, narrowing of small airways, and loss of lung elasticity.

A nurse is performing assessment for a patient diagnosed with chronic obstructive pulmonary disease (COPD). Which of the following findings should the nurse expect to observe? A. Nonproductive cough B. Prolonged inspiration C. Vesicular breath sounds D. Increased anterior-posterior chest diameter

D. Increased anterior-posterior chest diameter An increased anterior-posterior diameter is a compensatory mechanism experienced by patients with COPD and is caused by air-trapping. Patients with COPD have a productive cough, often expectorating copious amounts of sputum. Because of air-trapping, patients with COPD experience a prolonged expiration because the rate of gas on exhalation takes longer to escape. Chest auscultation for patients with COPD often reveals wheezing, crackles, and other adventitious breath sounds.

When assessing lab values on a patient admitted with septicemia, the nurse would expect to find: A. Increased platelets B. Decreased red blood cells C. Decreased erythrocyte sedimentation rate (ESR) D. Increased bands in the WBC differential (shift to the left)

D. Increased bands in the WBC differential (shift to the left) When infections are severe, such as in septicemia, more granulocytes are released from the bone marrow as a compensatory mechanism. To meet the increased demand, many young, immature polymorphonuclear neutrophils (bands) are released into circulation. WBCs are usually reported in order of maturity, with the less mature forms on the left side of a written report. Hence, the term "shift to the left" is used to denote an increase in the number of bands.

The blood bank notifies the nurse that the two units of blood ordered for an anemic patient are ready for pick up. The nurse should take which of the following actions to prevent an adverse effect during this procedure? A. Immediately pick up both units of blood from the blood bank. B. Regulate the flow rate so that each unit takes at least 4 hours to transfuse. C. Set up the Y-tubing of the blood set with dextrose in water as the flush solution. D. Infuse the blood slowly for the first 15 minutes of the transfusion.

D. Infuse the blood slowly for the first 15 minutes of the transfusion. Because a transfusion reaction is more likely to occur at the beginning of a transfusion, the nurse should initially infuse the blood at a rate no faster than 2 ml/min and remain with the patient for the first 15 minutes after hanging a unit of blood.

The nurse is evaluating whether a patient understands how to safely determine whether a metered dose inhaler is empty. The nurse interprets that the patient understands this important information to prevent medication underdosing when the patient describes which method to check the inhaler? A. Place it in water to see if it floats. B. Shake the canister while holding it next to the ear. C. Check the indicator line on the side of the canister. D. Keep track of the number of inhalations used.

D. Keep track of the number of inhalations used. It is no longer appropriate to see if a canister floats in water or not as research has demonstrated this is not accurate. The best method to determine when to replace an inhaler is by knowing the maximum puffs available per MDI and then replacing when those inhalations have been used.

When planning appropriate nursing interventions for a patient with metastatic lung cancer and a 60-pack-year history of cigarette smoking, the nurse recognizes that the smoking has most likely decreased the patient's underlying respiratory defenses because of impairment of which of the following? A. Reflex bronchoconstriction B. Ability to filter particles from the air C. Cough reflex D. Mucociliary clearance

D. Mucociliary clearance Smoking decreases the ciliary action in the tracheobronchial tree, resulting in impaired clearance of respiratory secretions, chronic cough, and frequent respiratory infections.

If a patient with arthritis develops iron-deficiency anemia, a nurse should ask about the patient's use of A. alcoholic beverages. B. stool softeners and laxatives. C. caffeinated foods and beverages. D. NSAIDs.

D. NSAIDs. NSAIDs decrease the level of vitamin C, which aids in the absorption of iron. These drugs also compete with folate and vitamin K and may cause gastritis. Excessive alcoholic beverage consumption can cause stomach irritation; alcohol would not be directly related to iron-deficiency anemia unless bleeding ulcers or gastritis were to occur. NSAID consumption, not stool softeners and laxative use, would be suspect for iron-deficiency anemia. Caffeinated foods and beverages can cause gastric irritation and discomfort but are not associated with iron-deficiency anemia.

The nurse is assessing a client who presents with jaundice. Which assessment finding is the most significant indication that further follow up is needed? A. Urine specific gravity of 1.03 with a urine output of 500 ml in 8 hours B. Frothy, tea-colored urine C. Clay-colored stools and complaints of pruritus D. Serum amylase and lipase levels that are twice their normal levels

D. Obstructive cholelithiasis and alcoholism are the two major causes of pancreatitis, and an elevated serum amylase and lipase indicate pancreatic injury. (A) is a normal finding. (B & C) are expected findings for jaundice.

When caring for a patient with COPD, the nurse identifies a nursing diagnosis of imbalanced nutrition less than body requirements after noting a weight loss of 30 lb. Which of the following would be an appropriate intervention to add to the plan of care for this patient? A. Teach the patient to use frozen meals at home that can be microwaved. B. Provide a high-calorie, high-carbohydrate, nonirritating, frequent feeding diet. C. Order fruits and fruit juices to be offered between meals. D. Order a high-calorie, high-protein diet with six small meals a day.

D. Order a high-calorie, high-protein diet with six small meals a day.Because the patient with COPD needs to use greater energy to breathe, there is often decreased oral intake because of dyspnea. A full stomach also impairs the ability of the diaphragm to descend during inspiration, interfering with the work of breathing. Finally, the metabolism of a high carbohydrate diet yields large amounts of CO2, which may lead to acidosis in patients with pulmonary disease. For these reasons, the patient with emphysema should take in a high-calorie, high-protein diet, eating six small meals per day.

Which of the following nursing interventions is most appropriate to enhance oxygenation in a patient with unilateral malignant lung disease? A. Positioning patient on right side. B. Maintaining adequate fluid intake C. Performing postural drainage every 4 hours D. Positioning patient with "good lung down"

D. Positioning patient with "good lung down" Therapeutic positioning identifies the best position for the patient assuring stable oxygenation status. Research indicates that positioning the patient with the unaffected lung (good lung) dependent best promotes oxygenation in patients with unilateral lung disease. For bilateral lung disease, the right lung down has best ventilation and perfusion. Increasing fluid intake and performing postural drainage will facilitate airway clearance, but positioning is most appropriate to enhance oxygenation.

The nurse plans to help an 18-year-old developmentally disabled female client ambulate on the first postoperative day. When the nurse tells her it is time to get out of bed, the client becomes angry and yells at the nurse. "Get out of here! I'll get up when I'm ready." Which response should the nurse provide? A. "Your healthcare provider has prescribed ambulation on the first postoperative day." B. "You must ambulate to avoid serious complications that are much more painful." C. "I know how you feel; you're angry about having to do this, but it is required." D. "I'll be back in 30 minutes to help you get out of bed and walk around the room."

D. Returning in 30 minutes provides a cooling off period, is firm, direct, nonthreatening, and avoids argument with the client. B is threatening. C. assumes what the client is feeling. A. avoids the nurse's responsibility to ambulate the client.

The nurse is assisting a patient to learn self-administration of beclomethasone two puffs inhalation q6hr. The nurse explains that the best way to prevent oral infection while taking this medication is to do which of the following as part of the self-administration techniques? A. Chew a hard candy before the first puff of medication. B. Ask for a breath mint following the second puff of medication. C. Rinse the mouth with water before each puff of medication. D. Rinse the mouth with water following the second puff of medication.

D. Rinse the mouth with water following the second puff of medication. The patient should rinse the mouth with water following the second puff of medication to reduce the risk of fungal overgrowth and oral infection.

The nurse is assessing a 75-year-old male client for symptoms of hyperglycemia. Which symptom of hyperglycemia is an older adult most likely to exhibit? A. Polyuria B. Polydipsia C. Weight loss D. Infection

D. S/Sx of hyperglycemia in older adults may include fatigue, infection, and neuropathy (such as sensory changes). (A, B, C) are classic symptoms and may be absent in the older adult.

A client with alcohol-related liver disease is admitted to the unit. Which prescription should the nurse question as possibly inappropriate for the client? A. Vitamin K1 (AquaMEPHYTON) 5 mg IM daily B. High-calorie, low-sodium diet C. Fluid restriction to 1500 ml/day D. Pentobarbital (Nembutal sodium) 50 mg at bedtime for rest

D. Sedatives such as Nembutal are contraindicated for clients with liver damage and can have dangerous consequences. (A) is often prescribed since normal clotting mechanism is damaged. (B) is needed to restore energy. (C) Fluids are restricted to decrease ascites which often accompanies cirrhosis, particularly in later stages of the disease.

In the case of pulmonary embolus from deep vein thrombosis, which of the following actions should the nurse take first? A. Notify the physician. B. Administer a nitroglycerin tablet sublingually. C. Conduct a thorough assessment of the chest pain. D. Sit the patient up in bed as tolerated and apply oxygen.

D. Sit the patient up in bed as tolerated and apply oxygen.The patient's clinical picture is consistent with pulmonary embolus, and the first action the nurse takes should be to assist the patient. For this reason, the nurse should sit the patient up as tolerated and apply oxygen before notifying the physician.

After admitting a patient to the medical unit with a diagnosis of pneumonia, the nurse will verify that which of the following physician orders have been completed before administering a dose of cefotetan (Cefotan) to the patient? A. Serum laboratory studies ordered for AM B. Pulmonary function evaluation C. Orthostatic blood pressures D. Sputum culture and sensitivity

D. Sputum culture and sensitivityThe nurse should ensure that the sputum for culture and sensitivity was sent to the laboratory before administering the cefotetan. It is important that the organisms are correctly identified (by the culture) before their numbers are affected by the antibiotic; the test will also determine whether the proper antibiotic has been ordered (sensitivity testing). Although antibiotic administration should not be unduly delayed while waiting for the patient to expectorate sputum, all of the other options will not be affected by the administration of antibiotics.

The nurse witnesses a baseball player receive a blunt trauma to the back of the head with a softball. What assessment data should the nurse collect immediately? A. Reactivity of deep tendon reflexes, comparing upper to lower extremities. B. Vital signs readings, excluding blood pressure if need equipment is unavailable. C. Memory of events that occurred before and after the blow to the head. D. Ability to spontaneously open the eyes before any tactile stimuli are given.

D. The LOC should be immediately established immediately after the head injury has occurred. Spontaneous eye opening (D) is a simple measure of LOC. (A) is not the best indicator of LOC. (B) is important but not the best indicator of LOC. (C) can be assessed after LOC has been established by assessing eye opening.

Twelve hours after chest tube insertion for hemothorax, the nurse notes that the client's drainage has decreased from 50 ml/hr to 5 ml/hr. What is the best inital action for the nurse to take? A. Document this expected decrease in drainage. B. Clamp the chest tube while assessing for air leaks. C. Milk the tube to remove any excessive blood clot build up. D. Assess for kinks or dependent loops in the tubing.

D. The least invasive action should be performed to assess the decrease in drainage. (A) is completed after assessing for and problems causing the decreased drainage. (B) is no longer protocol because the increased pressure may be harmful for the client. (C) is an appropriate nursing action after the tube has been assessed for kinks or dependent loops.

An older client is admitted with a diagnosis of bacterial pneumonia. The nurse's assessment of the client will most likely reveal which S/SX? A. Leukocytosis and febrile. B. Polycythemia and crackles. C. Pharyngitis and sputum production. D. Confusion and tachycardia.

D. The onset of pneumonia is the older may be signaled by general deterioration, confusion, increased heart rate or increased respiratory rate. (A, B, C) are often absent in the older with bacterial pneumonia.

The nurse identifies the nursing diagnosis of activity intolerance for a patient with asthma. The nurse assesses for which of the following etiologic factor for this nursing diagnosis in patients with asthma? A. Anxiety and restlessness B. Effects of medications C. Fear of suffocation D. Work of breathing

D. Work of breathingWhen the patient does not have sufficient gas exchange to engage in activity, the etiologic factor is often the work of breathing. When patients with asthma do not have effective respirations, they use all available energy to breathe and have little left over for purposeful activity.

After a posterior nasal pack is inserted by a physician, the patient is very anxious and states, "I don't feel like I'm breathing right." The immediate intervention the nurse should initiate is to A. monitor ABGs. B. reassure the patient that this is normal discomfort. C. cut the pack strings and pull the packing out with a hemostat. D. direct a flashlight into the patient's mouth and inspect the oral cavity.

D. direct a flashlight into the patient's mouth and inspect the oral cavity. The nurse should inspect the oral cavity for the presence of blood, soft palate necrosis, and proper placement of the posterior plug. If the posterior plug is visible, the physician should be notified for readjustment of the packing. Reassurance, cutting the strings, and ABGs are not top priority interventions. The nurse needs further data before intervening.

Anticoagulant therapy is used in the treatment of thromboembolic disease because anticoagulants can A. dissolve the thrombi. B. decrease blood viscosity. C. prevent absorption of vitamin K. D. inhibit the synthesis of clotting factors.

D. inhibit the synthesis of clotting factors. Anticoagulant therapy is based on the premise that the initiation or extension of thrombi can be prevented by inhibiting the synthesis of clotting factors or by accelerating their inactivation. The anticoagulants heparin and warfarin do not induce thrombolysis but effectively prevent clot extension.

A patient's ABGs include a PaO2 of 88 mm Hg and a PaCO2 of 38 mm Hg and mixed venous blood gases include a PvO2 of 40 mm Hg and PvCO2 of 46 mm Hg. These findings indicate that the patient has A. impaired cardiac output. B. unstable hemodynamics. C. inadequate delivery of oxygen to the tissues. D. normal capillary oxygen-carbon dioxide exchange.

D. normal capillary oxygen-carbon dioxide exchange. Normal venous blood gas values reflect the normal uptake of oxygen from arterial blood and the release of carbon dioxide from cells into the blood, resulting in a much lower PaO2 and an increased PaCO2. The pH is also decreased in mixed venous blood gases because of the higher PvCO2. Normal mixed venous blood gases also have much lower PvO2 and SvO2 than arterial blood bases. Mixed venous blood gases are used when patients are hemodynamically unstable to evaluate the amount of oxygen delivered to the tissue and the amount of oxygen consumed by the tissues.

134.) A nurse reinforces instructions to a client who is taking levothyroxine (Synthroid). The nurse tells the client to take the medication: 1. With food 2. At lunchtime 3. On an empty stomach 4. At bedtime with a snack

Rationale: Oral doses of levothyroxine (Synthroid) should be taken on an empty stomach to enhance absorption. Dosing should be done in the morning before breakfast. **Note that options 1, 2, and 4 are comparable or alike in that these options address administering the medication with food.**

206.) A client is receiving baclofen (Lioresal) for muscle spasms caused by a spinal cord injury. The nurse monitors the client, knowing that which of the following is a side effect of this medication? 1. Muscle pain 2. Hypertension 3. Slurred speech 4. Photosensitivity

Rationale: Side effects of baclofen include drowsiness, dizziness, weakness, and nausea. Occasional side effects include headache, paresthesia of the hands and feet, constipation or diarrhea, anorexia, hypotension, confusion, and nasal congestion. Paradoxical central nervous system excitement and restlessness can occur, along with slurred speech, tremor, dry mouth, nocturia, and impotence. **Option 3 is most closely associated with a neurological disorder**

lorazepam

TX: Sedative-hypnotics for sleep, Adjuncts to anesthesia to induce relaxation and amnesia (procedural memory loss), To reduce anxiety (anxiolytic), Panic disorders, To treat or prevent seizures, For alcohol withdrawal, Muscle relaxant

community-acquired pneumonia

a lower respiratory tract infection of the lung parenchyma with onset in the community or during the first 2 days of hospitalization.

tension pneumothorax

a pneumothorax with rapid accumulation of air in the pleural space causing severely high intrapleural pressures with resultant tension on the heart and great vessels.

lung abscess

a pus-containing lesion of the lung parenchyma that results in a cavity formed by necrosis of lung tissue.

chemoreceptor

a sensory nerve cell that responds to a change in the chemical composition (PaCO2 and pH) of the fluid around it.

chest physiotherapy

a series of maneuvers including percussion, vibration, and postural drainage designed to promote clearance of excessive respiratory secretions.

alpha 1-antitrypsin

a serum protein produced by the liver normally found in the lungs that inhibits proteolytic enzymes of white cells from lysing lung tissue; genetic deficiency of this protein can cause emphysema.

6. When a client is taking an antidepressant, what should the nurse do? (Select all that apply.) a. Monitor the client for suicidal tendencies. b. Observe the client for orthostatic hypotension. c. Teach the client to take the drug with food if GI distress occurs. d. Tell the client that the drug may not have full effectiveness for 1 to 2 weeks. e. Advise the client to maintain adequate fluid intake of 2 L/day.

a. Monitor the client for suicidal tendencies. b. Observe the client for orthostatic hypotension. c. Teach the client to take the drug with food if GI distress occurs. d. Tell the client that the drug may not have full effectiveness for 1 to 2 weeks.

3. When benztropine (Cogentin) is ordered for a client, the nurse acknowledges that this drug is an effective treatment for which condition? a. Parkinsonism b. Paralytic ileus c. Motion sickness d. Urinary retention

a. Parkinsonism

7. A client is to receive conscious sedation for a minor surgical procedure. Which drug administration should the nurse expect? (Select all that apply.) a. Propofol (Diprivan) to sustain natural sleep b. Lidocaine (Xylocaine) to provide local anesthesia c. Midazolam (Versed) to promote sedation and following of commands d. Ketamine (Ketalar) for rapid inductionand prolonged duration of action

a. Propofol (Diprivan) to sustain natural sleep c. Midazolam (Versed) to promote sedation and following of commands

6. A client who is prescribed pyridostigmine bromide (Mestinon) is being taught about the drug. Which statements should the nurse include in the teaching? (Select all that apply.) a. The drug must be taken on time. b. The drug must be taken two times per day. c. Underdosing can result in cholinergic crisis. d. Overdosing can result in cholinergic crisis. e. The client should report the adverse effects of tachycardia to the health care provider.

a. The drug must be taken on time. c. Underdosing can result in cholinergic crisis. d. Overdosing can result in cholinergic crisis.

6. What should the client who is taking anticholinergic therapy for parkinsonism be taught? (Select all that apply.) a. To avoid alcohol, cigarettes, and caffeine b. To relieve dry mouth with hard candy or ice chips c. To use sunglasses to reduce photophobia d. To urinate 2 hours after taking the drug e. To receive routine eye examinations

a. To avoid alcohol, cigarettes, and caffeine b. To relieve dry mouth with hard candy or ice chips c. To use sunglasses to reduce photophobia e. To receive routine eye examinations

7. When the client has a cholinergic overdose, the nurse anticipates administration of which drug as the antidote? a. atropine b. bethanechol c. ambenonium d. metoclopramide

a. atropine

4. An atypical antipsychotic is prescribed for a client with psychosis. The nurse understands that this category of medications includes which drug? a. clozapine (Clozaril) b. loxapine (Loxitane) c. haloperidol (Haldol) d. thiothixene (Navane)

a. clozapine (Clozaril)

5. A client is admitted to the emergency department with status epilepticus. Which drug should the nurse most likely prepare to administer to this client? (Select all that apply.) a. diazepam (Valium) b. midazolam (Versed) c. gabapentin (Neurontin) d. levetiracetam (Keppra)

a. diazepam (Valium) b. midazolam (Versed)

6. The nurse who is teaching the client to self-administer medications explains to the client that which drug treats narcolepsy? a. modafinil b. atomoxetine c. lisdexamfetamine d. methylphenidate

a. modafinil

5. Assessing a client following IV morphine administration, the nurse notes cold, clammy skin; a pulse of 40 beats/min; respirations of 10 breaths/min; and constricted pupils. Which medication will the client likely need next? a. naloxone (Narcan) b. meloxicam (Mobic) c. pentazocine (Talwin) d. propoxyphene (Darvon)

a. naloxone (Narcan)

3. A selective serotonin reuptake inhibitor (SSRI) is prescribed for a client. The nurse knows that which drug is an SSRI? a. paroxetine (Paxil) b. amitriptyline (Elavil) c. divalproex sodium (Depakote) d. bupropion HCl (Wellbutrin)

a. paroxetine (Paxil)

metered-dose inhaler

aerosolized drug delivered in a specific amount by activating the inhaler or by inhaling.

lovastatin (mevacor)

aid in lowering LDL & increasing HDL S/S: muscle aches, hepatotoxicity, myopathy, rhabdomyolysis, peripheral neruopathy NI: take in evening, monitor renal and liver function, low fat/high fiber diet, drug interactions: digoxin, warfarin, NSAIDs, etc.

absorption atelectasis

alveolar collapse that occurs when high concentrations of oxygen are given and oxygen replaces nitrogen in the alveoli; if airway obstruction occurs, the oxygen is absorbed into the bloodstream and the alveoli collapse.

pulmonary edema

an abnormal accumulation of fluid in the alveoli and interstitial spaces of the lungs caused most commonly by heart failure; an acute, life-threatening situation in which the lung alveoli become filled with serous or serosanguineous fluid caused most commonly by heart failure.

pleural effusion

an abnormal accumulation of fluid in the intrapleural spaces of the lungs.

hyperreactivity

an abnormal condition in which responses to stimuli are exaggerated.

apnea

an absence of spontaneous respirations.

pneumonia

an acute inflammation of the lungs, often caused by inhaled pneumococci of the species Streptococcus pneumoniae.

Prednisone report

sore throat

Chapter 27 1. The nurse realizes that facial grimacing, involuntary upward eye movement, and muscle spasms of the tongue and face are indicative of which condition? a. Akathisia b. Acute dystonia c. Tardive dyskinesia d. Pseudoparkinsonism

b. Acute dystonia

7.The nurse is administering atenolol (Tenormin) to a client. Which concurrent drug does the nurse expect to most likely cause an interaction? a. ginseng herb b. An NSAID, such as aspirin c. methyldopa (Aldomet) d. haloperidol (Haldol)

b. An NSAID, such as aspirin

Chapter 24 1. When the nurse explains the pathophysiology of myasthenia gravis to a client, which is the best explanation? a. Degeneration of cholinergic neurons and a deficit in acetylcholine leads to neuritic plaques and neurofibrillary tangles. b. Decreased amount of acetylcholine to cholinergic receptors produces weak muscles and reduced nerve impulses. c. Myelin sheaths of nerve fibers in brain and spinal cord develop lesions or plaques. d. Imbalance of dopamine and acetylcholine leads to degeneration of neurons in midbrain and extrapyramidal motor tracts.

b. Decreased amount of acetylcholine to cholinergic receptors produces weak muscles and reduced nerve impulses.

2. Phenytoin (Dilantin) has been prescribed for a client with seizures. The nurse should include which appropriate nursing intervention in the plan of care? a. Reporting an abnormal phenytoin level of 18 mcg/mL b. Monitoring CBC levels for early detection of blood dyscrasias c. Encouraging the client to brush teeth vigorously to prevent plaque buildup d. Teaching the client to stop the drug immediately when passing pinkish-red or reddish-brown urine

b. Monitoring CBC levels for early detection of blood dyscrasias

6. The nurse should monitor the client receiving phenytoin (Dilantin) for which adverse effect? a. Psychosis b. Nosebleeds c. Hypertension d. Gum erosion

b. Nosebleeds

6. A client is receiving haloperidol (Haldol). Which nursing intervention(s) should the nurse perform? (Select all that apply.) a. Monitor vital signs to detect bradycardia. b. Remain with the client until medication is swallowed. c. Monitor vital signs to detect orthostatic hypotension. d. Assess the client for evidence of neuroleptic malignant syndrome. e. Observe the client for acute dystonia, akathisia, and tardive dyskinesia.

b. Remain with the client until medication is swallowed. c. Monitor vital signs to detect orthostatic hypotension. d. Assess the client for evidence of neuroleptic malignant syndrome. e. Observe the client for acute dystonia, akathisia, and tardive dyskinesia.

Chapter 19 1. A client is receiving bethanechol (Urecholine). The nurse realizes that the action of this drug is to treat: a. Glaucoma b. Urinary retention c. Delayed gastric emptying d. Gastroesophageal reflux disease

b. Urinary retention

5. The nurse realizes that cholinergic agonists mimic which parasympathetic neurotransmitter? a. dopamine b. acetylcholine c. cholinesterase d. monoamine oxidase

b. acetylcholine

chapter 18 For the client taking epinephrine, the nurse realizes there is a possible drug interaction with which drug? a. albuterol (Proventil) b. metoprolol (Lopressor) c. bethanechol (Urecholine) d. tolterodine tartrate (Detrol)

b. metoprolol (Lopressor)

4. A client is admitted to the emergency department in respiratory depression following self-injection with hydromorphone. The admitting nurse knows that which drug will reverse respiratory depression caused by opioid overdose? a. fentanyl b. naloxone c. butorphanol d. sufenta

b. naloxone

nadolol (Corgard)

beta blocker dibilatating anginal pain bypass surgery patients use with diuretic for hypertension use extreme caution with respiratory problems (asthma) and congestive heart failure

3. When administering phenytoin (Dilantin), the nurse realizes more teaching is needed if the client makes which statement? a. "I must shake the oral suspension very well before pouring in the dose cup." b. "I cannot drink alcoholic beverages when taking phenytoin." c. "I should take phenytoin 1 hour before meals." d. "I will need to get periodic dental checkups."

c. "I should take phenytoin 1 hour before meals."

2. A client states during a medical history that he takes several acetaminophen tablets throughout the day. The nurse teaches the client that the dosage should not exceed which amount? a. 1 g/day b. 2 g/day c. 4 g/day d. 6 g/day

c. 4 g/day

3. A client has spasticity following a spinal cord injury. The nurse should expect which drug to be prescribed to treat this client's spasticity? a. Tacrine b. Ropinirole c. Carisoprodol d. Pyridostigmine

c. Carisoprodol

5.The nurse realizes that beta1 receptor stimulation is differentiated from beta2 stimulation in that stimulation of beta1 receptors leads to which condition? a. Increased bronchodilation b. Decreased uterine contractility c. Increased myocardial contractility d. Decreased blood flow to skeletal muscles

c. Increased myocardial contractility

2. A client taking lorazepam (Ativan) asks the nurse how this drug works. The nurse should respond by stating that it is a benzodiazepine that acts by which mechanism? a. Depressing the central nervous system (CNS), leading to a loss of consciousness b. Depressing the CNS, including the motor and sensory activities c. Increasing the action of the inhibitory neurotransmitter gamma-aminobutyric acid (GABA) to GABA receptors d. Creating an epidural block by placement of the local anesthetic in the outer covering of the spinal cord

c. Increasing the action of the inhibitory neurotransmitter gamma-aminobutyric acid (GABA) to GABA receptors

5. A client is taking carisoprodol (Soma). Which statement would the nurse include in teaching the client about this drug? a. It may cause hypertension. b. It may lead to bradycardia. c. It blocks interneuronal activity. d. Its action is decreased by antihistamines.

c. It blocks interneuronal activity.

5. The nurse teaches a client about which common side effect of analeptics? a. Bradycardia b. Constipation c. Nervousness d. Urinary retention

c. Nervousness

4. A client is taking rivastigmine (Exelon). The nurse should teach the client and family which information about rivastigmine? a. That hepatotoxicity may occur b. That the initial dose is 6 mg t.i.d c. That GI distress is a common side effect d. That weight gain may be a side effect

c. That GI distress is a common side effect

6. A client is taking triazolam (Halcion). Which instructions about this drug are important for the nurse to include? a. It may be used as a barbiturate for only 4 weeks. b. Use as a nonbenzodiazepine to reduce anxiety. c. This drug does not lead to vivid dreams or nightmares. d. Avoid alcohol and smoking to prevent rebound insomnia.

d. Avoid alcohol and smoking to prevent rebound insomnia.

5. Which statement is true concerning lithium? a. The maximum dose is 3.4 g/day. b. The therapeutic drug range is 2.5 to 3.5 mEq/L. c. Lithium increases receptor sensitivity to GABA. d. Concurrent NSAIDs may increase lithium levels.

d. Concurrent NSAIDs may increase lithium levels.

2. A client is receiving carbidopa-levodopa for parkinsonism. What should the nurse know about this drug? a. Carbidopa-levodopa may lead to hypertension. b. Carbidopa-levodopa may lead to excessive saliva. c. Dopaminergic and anticholinergic therapy may lead to drowsiness and sedation. d. Dopaminergics and anticholinergics are contraindicated in clients with glaucoma.

d. Dopaminergics and anticholinergics are contraindicated in clients with glaucoma.

6. A client is given epinephrine (Adrenalin), an adrenergic agonist (sympathomimetic). The nurse should monitor the client for which condition? a. Decreased pulse b. Pupil constriction c. Bronchial constriction d. Increased blood pressure

d. Increased blood pressure

6. The nurse is administering a cholinergic agonist and should know that the expected cholinergic effects include which of the following? a. Increased heart rate b. Decreased peristalsis c. Decreased salivation d. Increased pupil constriction

d. Increased pupil constriction

4. Dicyclomine (Bentyl) is an anticholinergic, which the nurse realizes is given to treat which condition? a. Mydriasis b. Constipation c. Urinary retention d. Irritable bowel syndrome

d. Irritable bowel syndrome

Chapter 20 1. When a 12-year-old child is prescribed methylphenidate, which is most important for the nurse to monitor? a. The child's temperature b. The child's respirations c. The child's intake and output d. The child's height and weight

d. The child's height and weight

Chapter 28 1. A client is admitted with bipolar affective disorder. The nurse acknowledges that which medication is used to treat this disorder for some clients in place of lithium? a. thiopental b. gingko biloba c. fluvoxamine (Luvox) d. divalproex (Depakote)

d. divalproex (Depakote)


Set pelajaran terkait

Radical Equations and Extraneous Roots

View Set

Education - Gender Differences in Education

View Set

Pathophysiology Exam 2 practice questions

View Set

Chapter 2: Summary of Learning Goals (pgs. 55-56)

View Set